195
B¸ GIÁO DC VÀ ĐÀO TO TRI HÈ HÙNG VƯƠNG ------------------ Chı biên: Nguy„n Văn M“u CÁC PHƯƠNG PHÁP GII TOÁN VÀ Đ D TUYN TRI HÈ HÙNG VƯƠNG NĂM 2012 K YU TRI HÈ, MÔN TOÁN H¯C CAO BNG - NĂM 2012 www.VNMATH.com

[Vnmath.com] Ky Yeu Trai He Hv 2012

Embed Size (px)

Citation preview

Page 1: [Vnmath.com] Ky Yeu Trai He Hv 2012

BỘ GIÁO DỤC VÀ ĐÀO TẠO

TRẠI HÈ HÙNG VƯƠNG

- - - - - - - - - - - - - - - - - -

Chủ biên: Nguyễn Văn Mậu

CÁC PHƯƠNG PHÁPGIẢI TOÁN VÀ ĐỀ DỰ TUYỂN

TRẠI HÈ HÙNG VƯƠNG NĂM 2012

KỶ YẾU TRẠI HÈ, MÔN TOÁN HỌC

CAO BẰNG - NĂM 2012

www.VNMATH.com

Page 2: [Vnmath.com] Ky Yeu Trai He Hv 2012

.

2

www.VNMATH.com

Page 3: [Vnmath.com] Ky Yeu Trai He Hv 2012

KỶ YẾUTRẠI HÈ HÙNG VƯƠNG 2012

Môn TOÁN HỌC

Cao Bằng, 01-04 tháng 8 năm 2012

www.VNMATH.com

Page 4: [Vnmath.com] Ky Yeu Trai He Hv 2012

Mục lục

Lời nói đầu 5

Đào Văn Lương

Phương pháp lượng giác và áp dụng 7

Nguyễn Việt Hà

Hệ phương trình không mẫu mực 17

Nguyễn Văn Xã

Chứng minh bất đẳng thức theo phương pháp suy luận 24

Nguyễn Thế Hiệp

Sử dụng hệ thức truy hồi trong bài toán đếm 35

Lê Hồ Quý

Tính toán một số lớp tích phân xác định dạng đặc biệt 55

Nguyễn Anh Tuấn

Một số phương pháp giải hệ ơhương trình 64

Trường THPT Chuyên Hạ Long

Chuyên đề dãy số nguyên 121

Nguyễn Ngọc Xuân, Bùi Thị Hương

Các phương pháp cơ bản khi giải hệ phương trình 137

Trần Ngọc Thắng

Sử dụng dãy số trong một số dạng bài tập về phương trình hàm trên tập Z 154

Đề thi chính thức

Môn Toán, Trại hè Hùng Vương 2011 155

3

www.VNMATH.com

Page 5: [Vnmath.com] Ky Yeu Trai He Hv 2012

Đáp án đề thi chính thức

Môn Toán, Trại hè Hùng Vương 2011 159

Đề thi dự bị

Môn Toán, Trại hè Hùng Vương 2011 160

Đáp án đề thi dự bị

Môn Toán, Trại hè Hùng Vương 2011 161

=====================

4

www.VNMATH.com

Page 6: [Vnmath.com] Ky Yeu Trai He Hv 2012

5Các chuyên đề Toán Trại hè Hùng Vương năm 2012, Cao Bằng 01-04/08/2012

Lời nói đầu

Hoạt động bồi dưỡng theo các cụm, khu vực theo địa hình và đặc thù văn hoá,

điển hình là các chương trình hoạt động của Trại hè Hùng Vương của khối các

trường THPT chuyên khu vực vùng núi phía bắc và trung du Bắc bộ liên kết với

một số trường khác như trường Vùng cao Việt Bắc, . . . đã thành truyền thống

và ngày càng đi vào nề nếp. Đó là khối các trường THPT Chuyên Hùng Vương

(Phú Thọ), THPT Chuyên Hoàng Văn Thụ (Hòa Bình), THPT Chuyên tỉnh Vĩnh

Phúc, THPT Chuyên tỉnh Bắc Giang, THPT Chuyên Chu Văn An (Lạng Sơn),

THPT Chuyên Thái Nguyên, THPT Chuyên Tuyên Quang, THPT Chuyên Hà

Giang, THPT Chuyên Lai Châu, THPT Chuyên Điện Biên, THPT Chuyên Bắc

Kạn, THPT Chuyên Bắc Giang, THPT Chuyên Hạ Long (Quảng Ninh), THPT

Chuyên Nguyễn Tất Thanh (Yên Bái) và Trường Vùng cao Việt Bắc.

Trại hè Hùng Vương hoạt động tuân thủ theo Bản điều lệ của Trại hè, nhờ đó,

các đơn vị chủ động xây dựng chương trình hành động và cách thức triển khai. Là

một nội dung hoạt động toàn diện của trường THPT Chuyên trên cơ sở hoàn toàn

tự nguyện tham gia của các trường THPT Chuyên khu vực miền núi - trung du

phía Bắc. Thông qua hoạt động của trại hè phát triển tư duy môn học cho những

học sinh có năng khiếu bộ môn văn hóa theo chương trình lớp 10 THPT Chuyên

của Bộ Giáo dục và Đào tạo. Đặc biệt trại hè hướng tới việc định hướng và rèn

luyện cho học sinh kỹ năng sống, kỹ năng giao tiếp xã hội, gắn lý thuyết với thực

tiễn xã hội ở địa phương và dất nước. Từ đó tăng cường chất lượng giáo dục văn

hóa, thể chất , hiểu biết xã hội qua những hoạt động ngoài nhà trường góp phần

nâng cao chất lượng giáo dục toàn diện cho học sinh. Gắn với các hoạt động trên

là việc bồi dưỡng chuyên môn nghiệp vụ và năng lực tổ chức các hoạt động xã hội

cho giáo viên đang giảng dạy ở các trường THPT Chuyên.

Các semina bộ môn góp phần giúp giáo viên và học sinh được tiếp cận với

những đơn vị kiến thức mới của bộ môn góp phần nâng cao chất lượng đào tạo

học sinh giỏi bộ môn của các trường tham gia trại hè.

Ban tổ chức trại hè đã xây dựng nội dung, chương trình kế hoạch cho các kỳ

trại hè ; liên hệ mời các giáo sư cố vấn khoa học; tổ chức và chủ trì các hội nghị

5

www.VNMATH.com

Page 7: [Vnmath.com] Ky Yeu Trai He Hv 2012

trù bị và trại hè chính thức.

Sản phẩm của trại hè: Mỗi bộ môn xây dựng một tập san (kỷ yếu) trong đó lưu

giữ các nội dung gồm các báo cáo khoa học bộ môn, các đề thi Olympic bộ môn,

các đề đề xuất của các trường kèm theo lời giải và một số bài làm có chất lượng

tốt của học sinh.

Trại hè Hùng Vương là hoạt động giao lưu gữa các trường THPT chuyên khu

vực Trung du - Miền núi phía bắc và trường Phổ thông Vùng cao Việt Bắc, trên cơ

sở hoàn toàn tự nguyện tham gia, nhằm bồi dưỡng và phát triển năng khiếu cho

học sinh về môn chuyên cho một số học sinh đạt kết quả xuất sắc của các trường

chuyên, tăng cường sự hiểu biết về văn hoá xã hội vùng miền cho học sinh qua các

hoạt động của trại hè, góp phần nâng cao chất lượng giáo dục toàn diện cho học

sinh; rèn luyện cho học sinh kỹ năng sống, kỹ năng giao tiếp xã hội, gắn lý thuyết

với thực tiễn xã hội ở địa phương và đất nước.

Trại hè Hùng Vương với mục tiêu bồi dưỡng và nâng cao năng lực tổ chức các

hoạt động xã hội cho giáo viên đang giảng dạy ở các trường THPT Chuyên, là nơi

dể trao đổi chuyên môn, kinh nghiệm và các nguồn lực khác giữa các trường nhằm

nâng cao chất lượng bồi dưỡng học sinh giỏi và hoạt động giáo dục toàn diện của

các trường chuyên tham gia trại hè.

Ngoài phần hoạt động chuyên môn, Trại hè Hùng Vương còn tổ chức giao lưu

văn hóa, văn nghệ, thăm quan học tập đối với học sinh và giáo viên các trường

tham gia trại hè, thi Olympic các môn học cho học sinh.

Năm nay Trại hè Hùng Vương được tổ chức tại trường THPT Chuyên Cao

Bằng từ ngày 01.08 đến 04.08. Nhằm bồi dưỡng và trao đổi chuyên môn, chúng tôi

giới thiệu một số tư liệu phục vụ trại hè này do các giáo viên và Ban chuyên môn

biên soạn, nhằm tổ chức tốt các semina giữa các giáo viên tham gia trại hè, trao

đổi tài liệu và các kinh nghiệm trong công tác giảng dạy.

Vì thời gian rất gấp gáp, nên các khâu chế bản và nội dung chắc chắn còn nhiều

khiếm khuyết. Mong nhận được sự góp ý của các thầy, cô và các em học sinh

Trưởng ban cố vấn khoa học

GS TSKH Nguyễn Văn Mậu

6

www.VNMATH.com

Page 8: [Vnmath.com] Ky Yeu Trai He Hv 2012

7Các chuyên đề Toán Trại hè Hùng Vương năm 2012, Cao Bằng 01-04/08/2012

Phương pháp lượng giác và áp dụng

Đào Văn Lương, Trường THPT Chuyên Lào Cai

Tóm tắt nội dung

Lượng giác là phần kiến thức quan trọng trong chương trình toán THPT, ngoàinhững bài toán liên quan trực tiếp đến lượng giác, như biến đổi lượng giác, hệ thứclượng giác trong tam giác, tứ giác, phương trình lượng giác, tích phân của các hàmsố lượng giác,..., thì một lượng không nhỏ các bài toán lại được chuyển về làm việcvới đối tượng là lượng giác thông qua các phép đặt lượng giác. Trong bài viết nayta sẽ áp phương pháp lượng giác vào giải quyết một số dạng toán cơ bản là các bàitoán về phương trình, hệ phương trình, các bài toán chứng minh bất đẳng thức vàmột số bài toán liên quan đến dãy số.

1 Các kết quả cơ bản

1.1 Một số đẳng thức lượng giác

1) sin2 x+ cos2 x = 1, ∀x ∈ R2) 1 + tan2 x =

1

cos2 x, ∀x 6= π

2+ kπ

3) 1 + cot2 x =1

sin2 x, ∀x 6= kπ

4) cotx+1

sinx= cot

x

2,∀x 6= kπ

5) Với α;β; γ 6= π

2+ kπ, k ∈ Z, ta có:

tanα+ tanβ + tan γ = tanα. tanβ. tan γ ⇐⇒ α+ β + γ = nπ(n ∈ Z)

6) Với α;β; γ 6= π

2+ kπ, k ∈ Z, ta có:

tanα. tanβ + tanβ. tan γ + tan γ. tanα = 1⇐⇒ α+ β + γ =π

2+ nπ(n ∈ Z)

1.2 Một số phép đặt lượng giác

7) Nếu x2 + y2 = 1 thì, đặt

x = sin t

y = cos t

7

www.VNMATH.com

Page 9: [Vnmath.com] Ky Yeu Trai He Hv 2012

8) Nếu x2 + y2 = a2 thì, đặt

x = a sin t

y = a cos t

9) Nếu x2 + y2 + z2 = 1 thì, đặt

x = cosϕ

y = sinϕ. cos θ

z = sinϕ. sin θ

10) Nếu x2 + y2 + z2 = a2 thì, đặt

x = a cosϕ

y = a sinϕ. cos θ

z = a sinϕ. sin θ

11) Trong mọi trường hợp ta đều có thể lượng giác hóa theo hàm tanx hoặc cotx

2 Áp dụng trong giải phương trình, hệ phương

trình

Ví dụ 2.1. Giải phương trình: x =

√2 +

√2−√2 + x

Giải. Điều kiện: 0 < x ≤ 2.Đặt x = 2. cos t, điều kiện: −π

2< t <

π

2(∗). Khi đó phương trình trở thành:

2 cos t =

√2 +

√2−

√2(1 + cos t)

⇐⇒ 2 cos t =

√2 +

√2(1− cos

t

2)

⇐⇒ 2 cos t =

√2 + 2 sin

t

4

⇐⇒ 2 cos t =√2(sin

t

8+ cos

t

8)

⇐⇒ sin(π

2− t) = sin(

π

4+t

8)(∗∗)

Giải phương trình (**) và kết hợp với điều kiện (*), ta nhận được 2 giá trị của t

thỏa mãn là: t1 =2π

9; t2 = −

7

Vậy phương trình đã cho có 2 nghiệm: x = 2 cos2π

9và x = 2 cos

7.

Ví dụ 2.2. Cho (x; y; z) là nghiệm của hệ phương trình:

x = y(4− y)

y = z(4− z)

z = x(4− x)Tìm tất cả các giá trị mà tổng S = x+ y + z có thể nhận được.

Giải.

Giả sử (x; y; z) là nghiệm của hệ phương trình đã cho. Cộng vế với vế tương ứngcác phương trình của hệ, ta nhận được:3S = x2+y2+z2 ≥ 0 =⇒ S ≥ 0. Vì S ≥ 0 nên trong 3 số x, y, z phải có ít nhất một

8

www.VNMATH.com

Page 10: [Vnmath.com] Ky Yeu Trai He Hv 2012

số không âm, không mất tính tổng quát, ta giả sử x ≥ 0, từ phương trình (1) củahệ ta suy ra 0 ≤ x ≤ 4. Bằng phép hoán vị vòng quanh ta có: 0 ≤ x, y, z ≤ 4. Đặtx = 4 sin2 α, với 0 ≤ α ≤ π. Khi đó từ PT(3) suy ra: z = 4. sin2 2α, thay vào PT(2),suy ra: y = 4. sin2 4α, thay trở lại phương trình đầu, ta nhận được x = 4. sin2 8α.Như vậy α là nghiệm của phương trình: sin2 8α = sin2 α ⇐⇒ cos 16α = cos 2α

⇐⇒ α =kπ

7;α =

9(k ∈ Z)

Trường hợp 1: α =kπ

7vì 0 ≤ α ≤ π, nên k ∈ 0; 1; 2; 3; 4; 5; 6; 7, tuy nhiên với

k ∈ 4; 5; 6; 7 hay k ∈ 0; 1; 2; 3 đều cho cùng một giá trị của x, y, z.*) Với k = 0 =⇒ α = 0 =⇒ S = 0

*) Với k ∈ 1; 2; 3 thì S có cùng một giá trị bằng: S = 4 sin2π

7+ 4 sin2

7+

4 sin23π

7= 7

Trường hợp 2: α =kπ

9vì 0 ≤ α ≤ π, nên k ∈ 0; 1; 2; 3; 4.

*) Với k = 0 =⇒ α = 0 =⇒ S = 0.

*) Với k ∈ 1; 2; 4 thì S có cùng một giá trị bằng: S = 4 sin2π

9+ 4 sin2

9+

4 sin24π

7= 6.

*) Với k = 3 thì S = 4 sin2π

3+ 4 sin2

3+ 4 sin2

3= 9.

Kết luận: S ∈ 0; 6; 7; 9.

Ví dụ 2.3. Tìm các số thực x, y, z thỏa mãn

x6+y6+z6−6(x4+y4+z4)+10(x2+y2+z2)−2(x3y+y3z+z3x)+6(xy+yz+zx) = 0

Giải. Biến đổi vế trái thành: P = (x3 − 3x− y)2 + (y3 − 3y− z)2 + (z3 − 3z − x)2

Do đó P = 0⇐⇒

y = x3 − 3x

x = z3 − 3z(∗)

z = y3 − 3y

*) Nếu x > 2 thì y = x(x2 − 3) > 2 =⇒ z = y(y2 − 3) > 2. Từ đó cộng theo vế baphương trình của hệ (*) ta có:0 = x3 + y3 + z3 − 4x− 4y − 4z = x(x2 − 4) + y(y2 − 4) + z(z2 − 4) > 0 (Vô lý).*) Lập luận hoàn toàn tương tự với x < −2, ta cũng có mâu thuẫn. Vậy |x| ≤ 2

Đặt x = 2 cos t, 0 ≤ t ≤ π, suy ra:

y = 2(4 cos3 t− 3 cos t) = 2. cos 3t

x = 2(4 cos3 3t− 3 cos 3t) = 2. cos 9t

z = 2(4 cos3 9t− 3 cos 9t) = 2. cos 27t

Dẫn đến, cos t = cos 27t⇐⇒ t =kπ

13(k = 0, 1, 2, ..., 13); t =

14(l = 0, 1, 2, ..., 14)

Ngược lại, dễ dàng kiểm tra được rằng, nếu cos t = cos 27t thì bộ(x; y; z) = (2 cos t; 2 cos 3t; 2 cos 9t), thỏa mãn hệ (*). Thành thử các bộ (x; y; z)

thỏa mãn đề bài là: (2 cos t; 2 cos 3t; 2 cos 9t), với t =kπ

13(k = 0, 1, 2, ..., 13) hoặc

t =lπ

14(l = 0, 1, 2, ..., 14).

9

www.VNMATH.com

Page 11: [Vnmath.com] Ky Yeu Trai He Hv 2012

3 Áp dụng trong chứng minh bất đẳng thức

Ví dụ 3.1. Xét các số thực dương a, b, c, thỏa mãn điều kiện abc+a+c = b. Chứngminh rằng:

2

a2 + 1− 2

b2 + 1+

3

c2 + 1≤ 10

3

Giải. Đặt P =2

a2 + 1− 2

b2 + 1+

3

c2 + 1Ta có abc+ a+ c = b⇐⇒ a+ c = b(1− ac)Nếu ac = 1 thì a+ c = 0 (vô lý), do a, b > 0. Vậy ac 6= 1 Viết lại điều kiện đã cho

dưới dạng: b =a+ c

1− ac, điều này gợi ý cho ta đến phép đặt:

a = tanA

b = tanB

c = tanC

Do a, b, c > 0, nên tồn tại A,B,C ∈ (0;π

2), thỏa mãn tanB =

tanA+ tanC

1− tanA. tanCTừ đây ta nhận được mối liên hệ giữa A,B,C là: A + B + C = π. Khi đó ta biếnđổiP = 2. cos2A− 2 cos2(A+ C) + 3. cos2C = cos 2A− cos(2A+ 2C) + 3 cos2C

= 2 sin(2A+ C). sinC + 3 cos2C ≤ 2 sinC + 3 cos2C = 2 sinC + 3− 3. sin2C

=10

3− 3(sinC − 1

3)2 ≤ 10

3

Dấu "=" xảy ra ⇐⇒

sinC =1

3

sin(2A+ C) = 1

Khi đó, (a; b; c) = (

√2

2;√2;

√2

4)

Ví dụ 3.2. Tìm giá trị lớn nhất của biểu thức Q =a

a+ bc+

b

b+ ca+

√abc

c+ ab. Với

a, b, c > 0 và thỏa mãn: a+ b+ c = 1

Giải.

Viết Q dưới dạng: Q =1

1 +bc

a

+1

1 +ca

b

+

√ab

c

1 +ab

c

Từ giả thiết: a+ b+ c = 1⇐⇒√ab

c

√ca

b+

√ab

c

√bc

a+

√bc

a

√ca

b= 1(∗)

Đặt

√bc

a= tan

A

2√ac

b= tan

B

2; (A,B,C ∈ (0;π))√

ab

c= tan

C

2Từ (*), ta có: A+B + C = π. Vậy A,B,C là 3 góc của một tam giác.

10

www.VNMATH.com

Page 12: [Vnmath.com] Ky Yeu Trai He Hv 2012

Khi đó, Q =1

1 + tan2 A2

+1

1 + tan2B

2

+tan

C

2

1 + tan2C

2

= cos2A

2+ cos2

B

2+

sinC

2

= 1 +1

2(cosA+ cosB + sinC).

Mặt khác, ta có: cosA+ cosB + sinC + sinπ

3≤ 2. cos

A+B

2+ 2 cos

C − π

32

≤ 4. cosA+B + C − π

34

= 4. cosπ

6= 2√3.

Suy ra: Q ≤ 1+1

2(2√3−√3

2) = 1+

3√3

4. Dấu "=" xảy ra⇐⇒

cosA−B

2= 1

sinC +

π

32

= 1

A+B

2=C − π

32

⇐⇒

A = B =π

6

C =2π

3

⇐⇒

a = b = 2√3− 3

c = 7− 4√3

Vậy MaxQ = 1 +3√3

4, đạt được ⇐⇒

a = b = 2√3− 3

c = 7− 4√3

Ví dụ 3.3. Các số dương x, y, z thỏa mãn điều kiện

x2 + y2 + z2 =1− 16xyz

4(∗). Tìm giá trị nhỏ nhất của biểu thức

S =x+ y + z + 4xyz

1 + 4xy + 4yz + 4zx

Giải. Từ giả thiết suy ra 0 < 2x < 1, 0 < 2y < 1, 0 < 2z < 1. Đặt 2x = a =

cosA; 2y = b = cosB; 2z = c, A,B ∈(0;π

2

). Khi đó, điều kiện (*) trở thành:

cos2A+ cos2B + c2 + 2c cosA cosB = 1

⇐⇒ (c+ cosA cosB)2 − (1− cos2A)(1− cos2B) = 0

⇐⇒ (c+ cosA cosB)2 − sin2A sin2B = 0

⇐⇒ [c+ cos(A+B)][c+ cos(A−B) = 0]

⇐⇒ c = − cos(A+B), vì (cos(A−B) > 0)

⇐⇒ c = cosC, trong đó A,B,C là ba góc của một tam giác nhọn. Mặt khác

tanA

2tan

B

2+ tan

B

2tan

C

2+ tan

C

2tan

A

2= 1, suy ra tan2

A

2tan2

B

2tan2

C

2≤ 1

27.

⇐⇒ cos2A

2cos2

B

2cos2

C

2≥ 27. sin2

A

2sin2

B

2sin2

C

2⇐⇒ (1 + a)(1 + b)(1 + c) ≥ 27(1− a)(1− b)(1− c)⇐⇒ 28(a+ b+ c+ abc) ≥ 26(1 + ab+ bc+ ca)

⇐⇒ 28(x+ y + z + 4xyz) ≥ 13(1 + 4xy + 4yz + 4zx)

Suy ra S ≥ 13

28. Vậy minS =

13

28. Đạt được khi x = y = z =

1

4

11

www.VNMATH.com

Page 13: [Vnmath.com] Ky Yeu Trai He Hv 2012

4 Dãy số và giới hạn

Trước hết ta xét ví dụ quen thuộc sau đây.

Ví dụ 4.1. Chứng minh rằng:√2 +

√2 +

√2 + · · ·+

√2︸ ︷︷ ︸

n dấu căn

+

√2−

√2 +

√2 + · · ·+

√2︸ ︷︷ ︸

n dấu căn

= 2√2. cos

2n−1 − 1

2n+1π

Giải. √2 +

√2 + · · ·+

√2︸ ︷︷ ︸

n-dấu căn

+

√2−

√2 + · · ·+

√2︸ ︷︷ ︸

n-dấu căn

= 2√2. cos

2n−1 − 1

2n+1π

⇐⇒ √2 +

√2 + · · ·+

√2︸ ︷︷ ︸

n-dấu căn

+

√2−

√2 + · · ·+

√2︸ ︷︷ ︸

n-dấu căn

= 2 cosπ

2n+1+ 2 sin

π

2n+1

Ta có các kết quả:

i)

√2 +

√2 + · · ·+

√2︸ ︷︷ ︸

n-dấu căn

= 2 cosπ

2n+1, ∀n ∈ N∗

ii)

√2−

√2 + · · ·+

√2︸ ︷︷ ︸

n-dấu căn

= 2 sinπ

2n+1, ∀n ∈ N∗

Hai kết quả trên được chứng minh bằng phương pháp qui nạpChứng minh (i).*) Với n = 1 thì (i) trở thành:

√2 = 2. cos

π

4(luôn đúng).

*) Giả sử (i) đúng với n = k(k > 1, k ∈ N∗), ta có√2 +

√2 + · · ·+

√2︸ ︷︷ ︸

k dấu căn

= 2 cosπ

2k+1

*) Xét√2 +

√2 + · · ·+

√2︸ ︷︷ ︸

k+1 dấu căn

=

√√√√√√2 +

√2 +

√2 + · · ·+

√2︸ ︷︷ ︸

k dấu căn

=

√2 + 2 cos

π

2k+1

=2| cos π

2k+2|, với 0 < π

2k+2<π

2⇒ | cos π

2k+2| = cos

π

2k+2

Vậy

√2 +

√2 + · · ·+

√2︸ ︷︷ ︸

k+1 dấu căn

= 2 cosπ

2k+2, ta có(i) đúng với n = k+1. Theo nguyên lý

qui nạp thì (i) đúng. Chứng minh tương tự ta có (ii). Từ đây ta nhận được đpcm.

12

www.VNMATH.com

Page 14: [Vnmath.com] Ky Yeu Trai He Hv 2012

Ví dụ 4.2. Cho dãy số xn thỏa mãn:

x1 =

1

2

xn+1 =1

2

(xn +

√x2n +

1

4n

) Chứng minh

rằng dãy số có giới hạn, tìm giới hạn đó?

Giải. Ta có: x1 =1

2=

cotπ

42

=1

2cot

π

22, suy ra x2 =

1

2

(x1 +

√x21 +

1

4

)=

1

22cot

π

23

Dự đoán, số hạng tổng quát xn =1

2ncot

π

2n+1, ∀n ∈ N∗(∗). Ta dễ dàng kiểm tra

lại khẳng định (∗) bằng phương pháp qui nạp toán học. Vậy xn =1

2ncot

π

2n+1.

Do đó limxn = lim1

2ncot

π

2n+1=

2

πlim

π

2n+1.cos

π

2n+1

sinπ

2n+1

=2

π

Ví dụ 4.3. Cho dãy số xn, được xác định bởi:

x1 =

√2

xn+1 =

√2xnxn + 1

, n = 2, 3, ...

Tìm limn∏

i=1xi.

Giải. Ta có: x1 =√2 =

11√2

=1

cosπ

22

;x2 =

√2x1x1 + 1

=1

cosπ

23

.

Bằng qui nạp ta chứng minh được: xk =1

cosπ

2k+1

. Từ đó,n∏

i=1xi = 2n. sin

π

2n+1

Suy ra: limn∏

i=1xi =

π

2.

Nhận xét 4.1. Qua các ví dụ trên ta thấy, việc đưa ra công thức số hạng tổngquát của dãy số, hoàn toàn phụ thuộc vào việc biến đổi và đánh dấu (coi) số hạngban đầu x1 là giá trị lượng giác của một góc đặc biệt, sau đó dựa vào các phép biếnđổi lượng giác, ta dự đoán được qui luật xác định của số hạng tổng quát, cuối cùnglà chứng minh công thức dự đoán bằng phương pháp qui nạp, các bài toán về giớihạn của dãy cũng từ đó được giải quyết. Ở một số bài toán việc chỉ ra các liên hệlà không hề đơn giản, ta xét tiếp các ví dụ sau.

Ví dụ 4.4. Cho dãy số (xn) được xác định:

x1 =

√3

3

xn+1 =xn + 2−

√3

1 + (√3− 2)xn

;n = 2, 3, ...

Chứng minh rằng: x12n+2 = 1

Giải. Ta có: tanπ

12= tan(

π

3− π

4) =

√3− 1√3− 1

= 2−√3.

Viết lại xn+1, dưới dạng: xn+1 =xn + tan

π

12

1− tanπ

12xn

. Lại có x1 = tanπ

6, suy ra: x2 =

13

www.VNMATH.com

Page 15: [Vnmath.com] Ky Yeu Trai He Hv 2012

tan(π

6+

π

12) và x3 = tan(

π

6+

π

12+

π

12) = tan(

π

6+ 2

π

12)

Dự đoán: xn = tan[π

6+(n−1)

π

12)]. Kết quả này được chứng minh bằng qui nạp

toán học. Vậy, ta có số hạng tổng quát: xn = tan[π

6+ (n− 1)

π

12)].

Khi đó: x12n+2 = tan(π

6+ (12n+ 1)

π

12) = tan(

π

6+ nπ +

π

12) = tan

π

4= 1 (đpcm)

Ví dụ 4.5. Cho hai dãy số dương (xn); (yn), n = 1, 2, ..., được xác định:x1 = y1 =

1√2

xn+1 =xn

4y2n+1 − 1, n = 1, 2, 3, ...

yn+1 =yn

−4x2n+1 + 1

Tìm giới hạn của dãy (xn) và (yn)?

Giải. Trước hết ta chứng minh bằng qui nạp rằng: x2n + y2n = 1(1), ∀n ∈ N∗.Thật vậy:Với n = 1 thì x21 + y21 = 1, hệ thức (1) đúng. Giả sử (1) đúng với n = k, tức làx2k + y2k = 1. Ta đi chứng minh (1), còn đúng với n = k + 1.Ta có: 1 = x2k + y2k = [xk+1(4y

2k+1 − 1)]2 + [yk+1(4x

2k+1 − 1)]2

⇐⇒ x2k+1(16y4k+1 − 8y2k+1 + 1) + y2k+1(16x

4k+1 − 8x2k+1 + 1)− 1 = 0

⇐⇒ (x2k+1 + y2k+1 − 1) + (16x2k+1y4k+1 − 16x2k+1y

2k+1 + 16x4k+1y

2k+1) = 0

⇐⇒ (x2k+1 + y2k+1 − 1)(16x2k+1y2k+1 + 1) = 0

⇐⇒ x2k+1 + y2k+1 = 1. Vậy (1) đúng với n = k + 1. Vậy x2n + y2n = 1(1), ∀n ∈ N∗.Đặt xn = sinαn; yn = cosαn, từ hệ thức truy hồi yn+1 =

yn−4x2n+1 + 1

, ta có:

cosαn+1 =cosαn

−4 sinα2n+1 + 1

=⇒ cosαn+1(1− 4 sinα2n+1) = cosαn

⇐⇒ cosαn+1(4 cosα2n+1 − 3) = cosαn

⇐⇒ cos 3αn+1 = cosαn =⇒ 3αn+1 = αn, do αn > 0, ∀n ∈ N∗

Từ đây, suy ra: αn =α1

3n−1, mặt khác x1 =

1√2=⇒ sinα1 =

1√2

=⇒ α1 =π

4=⇒ αn =

π

4.3n−1. Vậy xn = sin

π

4.3n−1; yn = cos

π

4.3n−1

Suy ra: limxn = lim sinπ

4.3n−1= sin(lim

π

4.3n−1) = 0

lim yn = lim cosπ

4.3n−1= cos(lim

π

4.3n−1) = 1

BÀI TẬP

Bài toán 4.1. Giải phương trình:a) 4x3 + 2

√1− x2 − 3x− 1 = 0

b) x+3x√

25x2 − 9=

7

4

14

www.VNMATH.com

Page 16: [Vnmath.com] Ky Yeu Trai He Hv 2012

Bài toán 4.2. Giải các hệ phương trình:

a)

2x+ x2y = y

2y + y2z = z

2z + z2x = x

b)

x3 − 3x = y(3x2 − 1)

y3 − 3y = z(3y2 − 1)

z3 − 3z = x(3z2 − 1)

c)

3(x+1

x) = 4(y +

1

y) = 5(z +

1

z)

xy + yz + zx = 1

Bài toán 4.3. Tìm nghiệm x, y, z, t của hệ:

x2 + y2 = 9

z2 + t2 = 16

xt+ yz ≥ 12

sao cho biểu thức

A = x+ z đạt giá trị lớn nhất.

Bài toán 4.4. Cho x, y, z > 0 và thỏa mãn: x + y + z = 1. Chứng minh rằng:√xy

z + xy+

√yz

x+ yz+

√zx

y + zx≤ 3

2

Bài toán 4.5. Cho x, y, z ∈ R, thỏa mãn: x2 + y2 + z2 = 1. Chứng minh rằng:

2xy + yz + zx ≤ 1 +√3

2

Bài toán 4.6. Cho x, y, z ∈ R, thỏa mãn: x2y2 + 2yx2 + 1 = 0. Tìm giá trị nhỏnhất và giá trị lớn nhất của biểu thức:

f(x, y) =2

x2+

1

x+ y(y +

1

x+ 2)

Bài toán 4.7. Cho dãy số (an) được xác định bởi:

a1 =

√2

2

an+1 =

√2

2

√1−

√1− a2n, ∀n ∈ N∗

Tìm số hạng tổng quát của an

Bài toán 4.8. Cho dãy: :

a1 =√2

an+1 =√2− an, ∀n ∈ N∗

Tìm số hạng tổng quát của an

Bài toán 4.9. Cho dãy số (an) được xác định bởi:

a1 =

√2

an+1 =

√2anan + 1

Tìm limn∏

i=1ai

15

www.VNMATH.com

Page 17: [Vnmath.com] Ky Yeu Trai He Hv 2012

Tài liệu

[1] Nguyễn Văn Mậu, Chuyên đề chọn lọc lượng giác và áp dụng, NXB Giáo dục2008.

[2] Nguyễn Văn Mậu, Chuyên đề chọn lọc dãy số và áp dụng, NXB Giáo dục 2008.

[3] Kỷ yếu hội nghị khoa học các chuyên đề toán học trong hệ THPT chuyên 2005.

[4] Kỷ yếu toán học trại hè Hùng Vương.

[5] Các wedsite toán học http://www.mathscope.org, http://www.mathlinks.ro

[6] Tạp trí toán học và tuổi trẻ.

16

www.VNMATH.com

Page 18: [Vnmath.com] Ky Yeu Trai He Hv 2012

17Các chuyên đề Toán Trại hè Hùng Vương năm 2012, Cao Bằng 01-04/08/2012

Hệ phương trình không mẫu mực

Nguyễn Việt Hà, Trường THPT Chuyên Lào Cai

1 Giới thiệu

Trong bài viết này, tôi xin được trao đổi về một số dạng hệ phương trình hoán

vị, đối xứng thường gặp trong các kì thi học sinh giỏi.

2 Hệ hoán vị

Trong một tài liệu về hệ phương trình của thầy Phạm Kim Chung (THPT

Đặng Thúc Hứa-Nghệ An) đã nói rất kĩ về hệ hoán vị vòng quanh. Cho hàm số

y = f(x), y = g(x) xác định trong miền D, xét hệ phương trìnhf(x1) = g(x2)

f(x2) = g(x3)...f(xn) = g(x1)

Ta có định lý sau:

Định lý 2.1. Nếu hàm số y = f(x) và y = g(x) cùng đồng biến trong miền D và

(x1;x2; . . . xn); là một nghiệm của hệ đã cho thì x1;x2 = · · · = xn; .

Chứng minh. Không mất tính tổng quát, giả sử x1 = minx1;x2; . . . ;xn. Khi

đó, x1 ≤ x2. Do f(x) đồng biến nên f(x1) ≤ f(x2), hay g(x2) ≤ g(x3). Tương tự như

vậy ta có x1 ≤ x2 ≤ x3 ≤ · · · ≤ xn ≤ x1. Do đó x1;x2 = · · · = xn;.

Ta xét một ví dụ

Bài toán 2.1. Giải hệ phương trình x3 + 11x2 + 41x+ 30 = y

y3 + 11y2 + 41y + 30 = z

z3 + 11z2 + 41z + 30 = x

Giải. Xét hàm f(x) = x3 + 11x2 + 41x + 30 và hàm g(x) = x, đây là hai hàm số

đồng biến trên R. Nên nếu hệ trên có nghiệm thì x = y = z.

17

www.VNMATH.com

Page 19: [Vnmath.com] Ky Yeu Trai He Hv 2012

Xét phương trình x3 + 11x2 + 41x + 30 = x có nghiệm duy nhất x = −1. Do đó hệ

đã cho có nghiệm duy nhất (−1;−1;−1).Trên đây là ví dụ mà hai hàm đang xét đồng biến trên cả tập xác định. Tuy

nhiên cũng có những trường hợp ta phải để ý một chút, giới hạn lại miền giá trị

thì hàm đang xét mới đồng biến.

Bài toán 2.2. Giải hệ phương trình y3 − 9x2 + 27x− 27 = 0

z3 − 9y2 + 27y − 27 = 0

x3 − 9z2 + 27z − 27 = 0

Giải. Hệ trên được viết lại dưới dạng y3 = 9x2 − 27x+ 27

z3 = 9y2 − 27y + 27

x3 = 9z2 − 27z + 27

Xét hàm số f(t) = 9t2 − 27t+ 27. Ta có: f(t) ≥ 27

4(Dấu bằng xảy ra khi t =

3

2). Do

đó y3 ≥ 27

4, suy ra y >

3

2. Tương tự, ta suy ra x, y, z >

3

2.

Do đó, ta chỉ xét với t >3

2thì cả hai hàm số f(t) = 9t2 − 27t + 27, g(t) = t3 đều là

các hàm số đồng biến. Làm tương tự như bài toán 1, ta có nghiệm của hệ đã cho

là (3; 3; 3).

Hay một ví dụ khác mà việc đánh giá không còn đơn giản như bài toán trên

nữa.

Bài toán 2.3 (Nguyễn Anh Hoàng-Nguyễn Đức Tấn: Phương trình và hơn thế

nữa). Giải hệ phương trình y3 − 6x2 + 12x− 8 = 0

z3 − 6y2 + 12y − 8 = 0

x3 − 6z2 + 12z − 8 = 0

Giải. Cộng ba phương trình ta có:

(x− 2)3 + (y − 2)3 + (z − 2)3 = 0

Nếu x > 2 thì từ phương trình đầu của hệ ta có y3 = 6x(x − 2) + 8 > 8, do đó

y > 2. Làm tương tự cho phương trình thứ hai ta thu được z > 2. Vậy ta thu được

x > 2, y > 2, z > 2 do đó

(x− 2)3 + (y − 2)3 + (z − 2)3 > 0,

18

www.VNMATH.com

Page 20: [Vnmath.com] Ky Yeu Trai He Hv 2012

vô lý. Do đó, x ≤ 2. Tương tự y ≤ 2, z ≤ 2. Do đó

(x− 2)3 + (y − 2)3 + (z − 2)3 ≤ 0,

và dấu bằng phải xảy ra, hay x = y = z = 2 là nghiệm duy nhất của hệ đã cho.

Bây giờ ta xét một số hệ hoán vị khác.

Bài toán 2.4. Tìm tất cả các nghiệm thực không âm của hệ sau

x22 + x1x2 + x41 = 1

x23 + x2x3 + x42 = 1

x24 + x3x4 + x43 = 1

. . .

x21999 + x1998x1999 + x41998 = 1

x21 + x1999x1 + x41 = 1

Giải. Đặt k =√√

2− 1 . Khi đó k thỏa mãn k4 + 2k2 = 1, do vậy xi = k là một

nghiệm.

Giả sử x1 < k, từ phương trình đầu tiên của hệ suy ra x2 > k. Tương tự, từ phương

trình tiếp theo ta thu được x3 < k và cứ tiếp tục như vậy ta lại thu được x1 < k,

mâu thuẫn.

Tương tự, nếu x1 < k ta cũng đi tới mâu thuẫn. Do đó, x1 = k. Tương tự cho các

ẩn còn lại, ta thu được nghiệm duy nhất của hệ là (x1;x2; . . . ;x1999) = (k; k; . . . ; k)

Bài toán 2.5. Tìm tất cả các nghiệm thực của hệx = 4z2

1+4z2

y = 4x2

1+4x2

z = 4y2

1+4y2

Giải. Từ hệ đã cho ta suy ra 0 ≤ x, y, z < 1. Viết lại hệ trên dưới dạng4z2 = x

1−x4x2 = y

1−y4y2 = z

1−z

Nhân các phương trình lại ta thu được (xyz)2(1− x)(1− y)(1− z) = xyz

64. Nếu một

trong ba số x, y, z bằng 0, không mất tính tổng quát ta giả sử x = 0 thì từ phương

trình dầu tiên của hệ, ta thu được z = 0 và do vậy y = 0. Do đó (0; 0; 0) là một

nghiệm.

Trong trường hợp xyz 6= 0, từ (xyz)2(1 − x)(1 − y)(1 − z) = xyz

64ta thu được x(1 −

19

www.VNMATH.com

Page 21: [Vnmath.com] Ky Yeu Trai He Hv 2012

x)y(1− y)z(1− z) = 1

64.

Nhưng x(1 − x) ≤ 1

4(dấu bằng xảy ra khi x =

1

2). Làm tương tự cho y, z, ta thu

được nghiệm còn lại của hệ là x = y = z =1

2.

Vậy hệ đã cho có hai nghiệm x = y = z = 0 và x = y = z =1

2.

Bài toán 2.6. Tìm tất cả các nghiệm (x, y, z) với x > 0, y > 0, z > 0 của hệ xy = z

yz = x

zx = y

Giải. Nếu x > 1 thì z = xy > 1 và y = zx > 1. Do đó z = xy > x, x = yz > y, và

y = zx > z, mâu thuẫn.

Tương tự, nếu x < 1, thì y < 1 và z < 1. Vậy, z = xy > x, x = yz > y, và y = zx > z,

mâu thuẫn.

Nếu x = 1 thì z = zy = 1 và y = zx = 1.

Do đó, hệ đã cho có nghiệm duy nhất (1; 1; 1).

3 Hệ đối xứng

Trong mục này, chúng ta xem xét một số hệ đối xứng.

Bài toán 3.1. Tìm tất cả các nghiệm thực dương của hệx3 + y3 + z3 = x+ y + z

x2 + y2 + z2 = xyz

Giải. Ta có xyz = x2 + y2 + z2 > y2 + z2 ≥ 2yz. Do đó x > 2 và x3 > x. Tương tự,

y3 > y và z3 > z, vô lý. Vậy hệ đã cho vô nghiệm.

Bài toán 3.2. Tìm nghiệm thực (x, y, z) của hệx+ y + z = 5

xy + yz + zx = 3

sao cho giá trị lớn nhất của z.

Giải. Ta có

25 = (x+ y + z)2 = (x2 + y2 + z2) + 2(xy + yz + zx) = (x2 + y2 + z2) + 6.

Vậy x2 + y2 + z2 = 19. Do đó(x− 1

3

)2+(y − 1

3

)2+(z − 1

3

)2= (x2+y2+z2)−2

(x+ y + z)

3+1

3= 19− 10

3+1

3= 16.

20

www.VNMATH.com

Page 22: [Vnmath.com] Ky Yeu Trai He Hv 2012

Vậy giá trị lớn nhất có thể của z là 41

3(với nghiệm tương ứng là x =

1

3; y =

1

3; z = 4

1

3)

Bài toán 3.3. Tìm tất cả các nghiệm thực của hệx21 + x22 + x23 + x24 = 4

x1x3 + x2x4 + x3x2 + x4x1 = 0

x1x2x3 + x1x2x4 + x1x3x4 + x2x3x4 = −2x1x2x3x4 = −1

Giải. Phương trình thứ hai của hệ tương đương (x1+x2)(x3+x4) = 0. Giả sử x1 =

−x2 = a. Khi đó các phương trình khác là x23+x24 = 4− 2a2, x3+x4 =2

a2;x3x4 =

1

a2.

Do đó,4

a2= (x3 + x4)

2 = 4− 2a2 +2

a2, vậy

(a2 − 2)(a2 + 1)(a2 − 1) = 0.

Nếu a = ±1 thì x3 + x4 = 2, x3x4 = 1, vậy x3 = x4 = 1. Nếu a2 = 2 thì x3 + x4 =

1, x3x4 =1

2và x3, x4 không phải số thực.

Vậy nghiệm của hệ đã cho là (1; 1; 1;−1) và các hoán vị.

Bài toán 3.4. Tìm tất cả các nghiệm dương của hệ x+ y2 + z3 = 3

y + z2 + x3 = 3

z + x2 + y3 = 3

Giải. Nếu x, y ≥ 1 thì x+ y2 ≤ x2+ y3 (dấu bằng xảy ra khi và chỉ khi x = y = 1).

Nhưng z3 = 3 − (x + y2) ≥ 3 − (x2 + y3) = z, vậy z ≥ 1. Từ đó, cả ba số x, y, z ≥ 1

nên x = y = z = 1. Tương tự, nếu x, y ≤ 1, khi đó z ≤ 1, và do đó (x; y; z) = (1; 1; 1).

Vậy nếu (x; y; z) 6= (1; 1; 1) thì trong hai số x, y thì một số lớn hơn 1 và một số nhỏ

hơn 1. Nhưng bằng lập luận tương tự ta có thể chỉ ra rằng trong hai số y, z thì một

số lớn hơn 1 và một số nhỏ hơn 1, trong hai số z, x thì một số lớn hơn 1 và một số

nhỏ hơn 1 , mâu thuẫn. Vậy hệ đã cho chỉ có duy nhất một nghiệm dương (1; 1; 1).

Bài toán 3.5. Tìm tất cả các nghiệm dương của hệw + x+ y + z = 12

wxyz = wx+ wy + wz + xy + xz + yz + 27

Giải. Đăt p =√wxyz. Áp dụng bất đẳng thức AM −GM cho wx,wy, wz, xy, xz, yz

ta có

p ≤ (wx+ wy + wz + xy + xz + yz)

6=p2

6− 27

6

, vậy (p− 3)2 ≥ 36. Do đó ‖p− 3‖ ≥ 6 nhưng p dương nên p ≥ 9. Do vậy wxyz ≥ 81.

Nhưng áp dụng bất đẳng thức AM −GM cho w, x, y, z ta có√p ≤ 3, dấu bằng xảy

ra khi và chỉ khi w = x = y = z. Vậy wxyz ≤ 81

Từ đó,wxyz = 81, và do đó w = x = y = z = 3.

21

www.VNMATH.com

Page 23: [Vnmath.com] Ky Yeu Trai He Hv 2012

4 Bài tập rèn luyện

Bài toán 4.1. Giải hệ phương trình x−√y = 1

y −√z = 1

z −√x = 1

Bài toán 4.2. Giải hệ phương trình x4 − 2y = −12

y4 − 2z = −12

z4 − 2x = −12

Bài toán 4.3. Giải hệ phương trìnhx1 =

√39 cos πx2

x2 =√39 cos πx3

x3 =√39 cos πx4

x4 =√39 cos πx1

Bài toán 4.4. Tìm tất cả các nghiệm dương của hệ bất phương trình (x1, x2, x3, x4, x5 >

0) (x21 − x3x5)(x22 − x3x5) ≤ 0

(x22 − x4x1)(x23 − x4x1) ≤ 0

(x23 − x5x2)(x24 − x5x2) ≤ 0

(x24 − x1x3)(x25 − x1x3) ≤ 0

(x25 − x2x4)(x21 − x2x4) ≤ 0

Bài toán 4.5. Chứng minh hệ phương trình sau có đúng một nghiệm thực dương:xy + yz + zx = 12

xyz − x− y − z = 2

Chứng minh rằng tồn tại nghiệm thực của hệ trên mà x, y, z phân biệt.

Bài toán 4.6. Tìm tất cả các số thực x, y thỏa mãn(1 + x)(1 + x2)(1 + x4) = 1 + y7

(1 + y)(1 + y2)(1 + y4) = 1 + x7

22

www.VNMATH.com

Page 24: [Vnmath.com] Ky Yeu Trai He Hv 2012

Tài liệu

[1] Nguyễn Văn Mậu, Phương pháp giải phương trình và bất phương trình. NXBGD,

1993.

[2] Phạm Kim Chung, Hệ phương trình bồi dưỡng HSG. Nghệ An, 2007.

[3] Lê Văn Đính, Tiểu luận của sinh viên khoa Toán, ĐHSPHN, 2010.

[4] Nguyễn Anh Hoàng-Nguyễn Đức Tấn, Hệ phương trình không mẫu mực.

(Phương trình và hơn thế nữa) Kỷ yếu bồi dưỡng giáo viên, HCM-Hè 2010,

pp. 13-18;

23

www.VNMATH.com

Page 25: [Vnmath.com] Ky Yeu Trai He Hv 2012

24Các chuyên đề Toán Trại hè Hùng Vương năm 2012, Cao Bằng 01-04/08/2012

Chứng minh bất đẳng thức theophương pháp suy luận

Nguyễn Văn Xã, Trường THPT Chuyên Cao Bằng

Trong các kì thi học sinh giỏi Toán quốc gia, Olympic Toán các khu vực trong

nước cũng như quốc tế ta thường thấy xuất hiện các bài toán về chứng minh bất

đẳng thức. Có những bất đẳng thức để chứng minh được ta phải sử dụng các bất

đẳng thức AM-GM, Cauchy-Schwarz, Holder, Cheuyshev, Schur, Jensen,sử dụng

đạo hàm vv... ở mức độ phức tạp. Tuy nhiên có một lớp các bài toán việc suy luận

trong khi chứng minh lại đóng vai trò chủ chốt, còn các bất đẳng thức kể trên hay

đạo hàm chỉ đóng vai trò hỗ trợ ở mức độ đơn giản. Lớp các bất đẳng thức được

chứng minh theo cách này tôi xin được gọi là các bất đẳng thức được chứng minh

bằng phương pháp suy luận.

1 Một số dạng bất đẳng thức sắp thứ tự

Bài toán 1.1. Cho a, b, c là các số thức dương thoả mãn điều kiện a ≥ b ≥ c. Chứng

minh rằnga2 − b2

c+b2 − c2

a+c2 − a2

b≥ 3a− 4b+ c.

Lời giải. Do a ≥ b ≥ c nêna+ b

c≥ 2,

c+ b

a≤ 2,

a+ c

b≥ 1.

Từ đó suy ra

a2 − b2

c≥ 2(a− b), c2 − b2

a≥ 2(c− b), a2 − c2

c≥ a− c.

Cộng vế với vế ba bất đẳng thức trên ta được

a2 − b2

c+b2 − c2

a+c2 − a2

b≥ 3a− 4b+ c.

Đây là điều phải chứng minh.

Dấu bằng xảy ra khi a = b = c.

24

www.VNMATH.com

Page 26: [Vnmath.com] Ky Yeu Trai He Hv 2012

Bài toán 1.2. Cho x, y, z là các số thức dương thoả mãn điều kiện

2

x+

1

y≤ 1 ,

4

z+ y ≤ 2.

Chứng minh rằng x+ 9y + z ≥ 26.

Lời giải. Do x, y, z > 0 nên có 1 ≥ 2

x+

1

y>

1

y⇒ y > 1

2 ≥ 4

z+ y > y ⇒ y < 2

Từ đó suy ra y ∈ (1 ; 2)

Ta có2

x≤ 1− 1

y=y − 1

y⇒ x ≥ 2y

y − 1= 2 +

2

y − 14

z≤ 2− y ⇒ z ≥ 4

2− yDo đó, Áp dung bất đẳng thức AM-GM ta có

x+ 9y + z ≥ 2

y − 1+

4

2− y+ 9y + 2

= 2[ 1

y − 1+ 9(y − 1)

]+[ 4

2− y+ 9(2− y)

]+ 2 ≥ 2 · 6 + 12 + 2 = 26

Đây là điều phải chứng minh.

Dấu bằng xảy ra khi

x =2y

y − 1

z =4

2− y1

y − 1= 9(y − 1)

4

2− y= 9(2− y)

x = 8

y =4

3z = 6

Bài toán 1.3. Cho x, y, z là các số thực dương thoả mãn điều kiện

x2 + y2 + z2 = xyz. Chứng minh rằng

xy + yz + zx+ 9 ≥ 4(x+ y + z).

Lời giải. Ta có xyz = x2 + y2 + z2 > 2yz ⇒ x > 2

Tương tự, ta cũng chứng minh được y ≥ 2 và z ≥ 2.

Ta có xy + yz + zx+ 9 ≥ 4(x+ y + z)

⇔ (x− 2)(y − 2) + (y − 2)(z − 2) + (z − 2)(x− 2) ≥ 3

Đặt a = x− 2 , b = y − 2 , c = z − 2 (a > 0, b > 0, c > 0).

Ta được bất đẳng thức ab+ bc+ ca ≥ 3.

Ta có x2 + y2 + z2 = xyz

⇔ (a+ 2)2 + (b+ 2)2 + (c+ 2)2 = (a+ 2)(b+ 2)(c+ 2)

25

www.VNMATH.com

Page 27: [Vnmath.com] Ky Yeu Trai He Hv 2012

⇔ a2 + b2 + c2 + 4 = abc+ 2(ab+ bc+ ca)

⇔ abc+ ab+ bc+ ca = a2 + b2 + c2 − (ab+ bc+ ca) + 4 ≥ 4.

Đặt t =

√ab+ bc+ ca

3≥ 3√abc⇒ t3 ≥ abc

Từ đó, suy ra t3 + 3t2 ≥ 4⇔ (t− 1)(t+ 2)2 ≥ 0⇒ t ≥ 1

⇒ ab+ bc+ ca ≥ 3. Đây là điều phải chứng minh.

Dấu bằng xảy ra khi a = b = c = 1⇔ x = y = z = 3.

Bài toán 1.4. Cho x, y, z là các số thức dương thoả mãn điều kiện x(x+y+z) = 3yz.

Chứng minh rằng

(x+ y)3 + (x+ z)3 + 3(x+ y)(x+ z)(y + z) ≤ 5(y + z)3.

Lời giải. Đặt a = x+ y , b = x+ z , c = y + z (a > 0, b > 0, c > 0).

Điểu kiện x(x+y+z) = 3yz trở thành c=a2+b2−ab = (a+b)2−3ab ≥ (a+b)2−3

4(a+b)2 =

1

4(a+ b)2

⇒ a+ b ≥ 2c (1)

Bất đẳng thức cần chứng minh tương đương với

a3 + b3 + 3abc ≤ 5c3 ⇔ (a+ b)(a2 + b2 − ab) + 3abc ≤ 5c3)

⇔ (a+ b)c2 + 3ab ≤ 5c2)

Từ (1) cho ta: (a+ b)c ≤ 2c2 và 3ab ≤ 3

4(a+ b)2 ≤ 3c2.

Từ đây ta suy ra điều phải chứng minh. Dấu bằng xảy ra khi a = b = c⇔ x = y = z.

Bài toán 1.5. Cho 0 ≤ x ≤ y ≤ 1 và 2x+ y ≤ 2.

Chứng minh rằng

2x2 + y2 ≤ 3

2.

Lời giải. Do 2x+ y ≤ 2 và x ≥ 0 nên 2x2 + xy ≤ 2x. (1)

Do y ≤ 1 và y − x ≥ 0 nên y(y − x) ≤ y − x⇔ y2 − xy ≤ y − x. (2)

Cộng vế với vế của (1) và (2) ta được 2x2 + y2 ≤ x+ y. (3)

Theo bất đẳng thức Cauchy-Schwarz ta có

(x+ y)2 =( 1√

2.√2x+ 1.y

)2 ≤ (12+ 1)(2x2 + y2

). (4).

Từ (3) và (4) suy ra (2x2 + y2)2 ≤ (x+ y)2 ≤ 3

2(2x2 + y2)⇒ 2x2 + y2 ≤ 3

2.

Đây là điều phải chứng minh.

Dấu bằng xảy ra khi x =1

2; y = 1.

26

www.VNMATH.com

Page 28: [Vnmath.com] Ky Yeu Trai He Hv 2012

Bài toán 1.6. Cho a2 + b2 + c2 = 1 .Chứng minh rằng

abc+ 2(1 + a+ b+ c+ ab+ bc+ ca) ≥ 0.

Lời giải. Từ a2 + b2 + c2 = 1⇒ a, b, c ∈ [−1 ; 1⇒ (1 + a)(1 + b)(1 + c) ≥ 0

⇒ 1 + a+ b+ c+ ab+ bc+ ca+ abc ≥ 0. (1)

Vì (1 + a+ b+ c)2 ≥ 0⇒ 1 + a2 + b2 + c2 + 2(a+ b+ c+ ab+ bc+ ca) ≥ 0

⇒ 2 + 2(a+ b+ c+ ab+ bc+ ca) ≥ 0⇒ 1 + a+ b+ c+ ab+ bc+ ca ≥ 0. (2)

Cộng vế với vế của (1) và (2) ta được

abc+ 2(1 + a+ b+ c+ ab+ bc+ ca) ≥ 0.

Đây là điều phải chứng minh.

Dấu bằng xảy ra khi a = −1; b = c = 0 và các hoán vị.

Bài toán 1.7. Cho a2 + b2 + c2 = 2 và ab+ bc+ ca = 1.

Chứng minh rằng

−4

3≤ a, b, c ≤ 4

3.

Lời giải. Coi c là tham số còn a, b là ẩn thì ta cóa2 + b2 + c2 = 2

ab+ bc+ ca = 1⇔

(a+ b)2 − 2ab = 2− c2c(a+ b) + ab = 1

Đặt S = a+ b, P = ab, thì phải có điều kiện S2 ≥ 4P . Ta đượcS2 − 2P = 2− c2cS + p = 1

⇔S2 + 2cS + c2 − 4 = 0

p = 1− cS ⇔S = −c± 2

p = 1− cSTrường hợp 1: P = 1 − cS, S = −c + 2 ⇒ P = c2 − 2c + 1. Do S2 ≥ 4P nên

(2− c)2 ≥ 4(c2 − 2c+ 1)⇔ 4c− 5c2 ≤ 0⇔ 0 ≤ c ≤ 4

3.

Trường hợp 2: P = 1 − cS, S = −c − 2. Tương tự trường hợp 1 ta suy ra đuợc

−4

3≤ c ≤ 0.

Như vậy, ta được −4

3≤ c ≤ 4

3.

Do vai trò của a, b, c giống nhau nên −4

3≤ a, b, c ≤ 4

3.

Đây là điều phải chứng minh.

Bài toán 1.8. Cho ba số x, y, z ∈ [0; 1]

Chứng minh rằng

2(x3 + y3 + z3 − (x2y + y2z + z2x) ≤ 3.

Lời giải. Do x, y, z ∈ [0; 1] nên ta có (1−x2)(1−y)+(1−y2)(1−z)+(1−z2)(1−x) ≥ 0

⇔ 3 ≥ (x2 + y2 + z2) + (x+ y + z)− (x2y + y2z + z2x). (1)

Vì x, y, z ∈ [0; 1] ⇒ x ≥ x2 ≥ x3, y ≥ y2 ≥ y3, z ≥ z2 ≥ z3 nên từ (1) ta có

27

www.VNMATH.com

Page 29: [Vnmath.com] Ky Yeu Trai He Hv 2012

3 ≥ (x3 + y3 + z3) + (x3 + y3 + z3)− (x2y + y2z + z2x)

⇒ 2(x3 + y3 + z3)− (x2y + y2z + z2x) ≤ 3.

Dấu bằng xảy ra khi x = y = z = 1 hoặc x = y = 1, z = 0 và các hoán vị.

Bài toán 1.9. Cho a, b, c ∈ (0; 1].

Chứng minh rằnga

1 + bc+

b

1 + ca+

c

1 + ab< 2.

Lời giải. Nhận xét:

Nếux

y≤ 1 (x > 0, y > 0) thì

x

y≤ x+m

y +m, nếu m > 0

Nếux

y≥ 1 (x > 0, y > 0) thì

x

y≥ x+m

y +m, nếu m > 0,

Do đó, ta cóa

1 + bc<

a+ a

1 + bc+ a=

2a

1 + bc+ a<

2a

a+ b+ c.(1)

(Vì 1 + bc ≥ b+ c⇔ (1− b)(1− c) ≥ 0 )

Tương tự, ta cóa

1 + ca<

2b

a+ b+ c(2)

vàc

1 + ab<

2c

a+ b+ c. (3)

Cộng vế với vế ba bất đẳng thức (1), (2) và (3) ta được

a

1 + bc+

b

1 + ca+

c

1 + ab< 2.

Đây là điều phải chứng minh.

Lí luận tương tự ta có thể giải được bài toán sau

Bài toán 1.10. Cho a, b, c > 0.Chứng minh rằng

a

a+ b+

b

b+ c+

c

c+ a<

√a

b+ c+

√b

c+ a+

√c

a+ b.

Lời giải. Ta cóa

a+ b<

a+ c

a+ b+ c,

b

b+ c<

b+ a

a+ b+ c,

c

c+ a<

c+ b

a+ b+ c

Từ đó, suy raa

a+ b+

b

b+ c+

c

c+ a<

2(a+ b+ c)

a+ b+ c= 2. (1)

Mặt khác, Theo bất đẳng thức AM-GM ta có√a

b+ c=

a√a(b+ c)

≥ a

a+ b+ c

2

=2a

a+ b+ c.

28

www.VNMATH.com

Page 30: [Vnmath.com] Ky Yeu Trai He Hv 2012

Tương tự, ta có

√b

c+ a≥ 2b

a+ b+ cvà

√c

a+ b≥ 2c

a+ b+ c.

Từ đó suy ra

√a

b+ c+

√b

c+ a+

√c

a+ b≥ 2(a+ b+ c)

a+ b+ c= 2. (2)

Từ (1) và(2) suy ra điều phải chứng minh.

Bài toán 1.11. Cho a, b, c là ba số dương. Chứng minh rằng

5b3 − a3

ab+ 3b2+

5c3 − b3

bc+ 3c2+

5a3 − c3

ca+ 3a2≤ a+ b+ c.

Lời giải. Vì a > 0, b > 0 nên suy ra a3 + b3 = (a+ b)(a2 + b2 − ab) ≥ (a+ b)ab

⇒ a3 − 5b3 ≥ ab(a+ b)− 6b3

⇒ a3 − 5b3 ≥ b(a2 + ab− 6b2) = b(a+ 3b)(a− 2b)

⇒ 5b3 − a3 ≥ b(a+ 3b)(b− 2a)

⇒ 5b3 − a3

ab+ 3b2≤ 2b− a. (1)

Dấu bằng xảy ra trong (1) khi a = b

Tương tự, ta có5c3 − b3

bc+ 3c2≤ 2c− b (2)

và5a3 − c3

ac+ 3a2≤ 2a− c. (3)

Cộng vế với vế của (1),(2) và(3) suy ra

5b3 − a3

ab+ 3b2+

5c3 − b3

bc+ 3c2+

5a3 − c3

ca+ 3a2≤ a+ b+ c.

Điều phải chứng minh.

Dấu bằng xảy ra khi a = b = c.

Bài toán 1.12. Cho 4ABC vuông tại A. Chứng minh rằng

cos4B

B+cos4C

C≥ 2

π.

Lời giải. Hiển nhiên ta có bất đẳng thức(B − π

4

)(tan4B − 1

)≥ 0

⇒(B − π

4

)(sin4Bcos4B

− 1)≥ 0

⇒(B − π

4

)(sin4B − cos4B

)≥ 0

⇒ Bsin4B +π

4cos4B −Bcos4B − π

4sin4B ≥ 0

⇒ Bsin4(π

2− C) + π

2cos4B − π

4cos4B − (

π

2− C)cos4B − π

4sin4B ≥ 0

29

www.VNMATH.com

Page 31: [Vnmath.com] Ky Yeu Trai He Hv 2012

⇒ Bcos4C + Ccos4B ≥ π

4(cos4B + sin4B)

⇒ Bcos4C + Ccos4B ≥ π

4(1− 1

2sin22B) ≥ π

8

⇒ cos4B

B+cos4C

C≥ π

8BC=

π

2(B + C)2≥ 2

π.

Vậycos4B

B+cos4C

C≥ 2

π. Điều phải chứng minh

Dấu bằng xảy ra khi

B = C

sin22B = 1⇔ B = C =

π

4

Bài toán 1.13. Cho a, b, c ∈ [0; 1] . Chứng minh rằng

a

b+ c+ 1+

b

c+ a+ 1+

c

a+ b+ 1+ (1− a)(1− b)(1− c) ≤ 1.

Lời giải. Do bất đẳng thức là đối xứng với ba biến a, b, c nên không mất tính

tổng quát ta có thể giả sử 0 ≤ a ≤ b ≤ c < 1.

Từ đó dễ thấya

b+ c+ 1+

b

c+ a+ 1+

c

a+ b+ 1≤ a

a+ b+ 1+

b

a+ b+ 1+

c

a+ b+ 1=a+ b+ c

a+ b+ 1. Bây

giờ ta chỉ cần chứng minha+ b+ c

a+ b+ 1+ (1− a)(1− b)(1− c) ≤ 1

⇔ (1− a)(1− b)(1− c) ≤ 1− ca+ b+ 1

⇔ (1− a)(1− b)(a+ b+ 1) ≤ 1

Ta có (1− a)(1− b)(a+ b+1) ≤ (1− a)(1− b)(1 + a)(1 + b) = (1− a2)(1− b2) ≤ 1. Suy

ra điều phải chứng minh.

Đẳng thức xảy ra khi a = b = c hoặc a = b = 0, c = 1 và các hoán vị.

Bài toán 1.14. Cho a, b, c ∈ [0; 1] . Chứng minh rằng

a2 + b2 + c2 ≤ a2b+ b2c+ c2a+ 1.

Lời giải. Vì a, b, c ∈ [0; 1] nên ta có

a2(1− b) ≤ a(1− b)

b2(1− c) ≤ b(1− c)

c2(1− a) ≤ c(1− a).

Cộng vế với vế ba bất đẳng thức trên lại ta được

a2(1− b) + b2(1− c) + c2(1− a) ≤ a(1− b) + b(1− c) + c(1− a)= (a− 1)(b− 1)(c− 1) + 1− abc ≤ 1

⇒ a2 + b2 + c2 ≤ a2b+ b2c+ c2a+ 1 . Điều phải chứng minh.

30

www.VNMATH.com

Page 32: [Vnmath.com] Ky Yeu Trai He Hv 2012

Bài toán 1.15. Cho x1, x2, . . . xn ∈ Rthoả mãn điều kiện | xi − xi+1 |< 1 với mọi

i = 1, 2, . . . , n, (x− i+ 1 = x1).

Chứng minh rằngx1x2

+x2x3

+ . . .xnx1

< 2n− 1.

Lời giải. Dox1x2

x2x3. . .

xnx1

= 1 nên tồn tại chỉ số k sao choxkxk+1

≤ 1. (1)

Từ giả thiết suy ra xi < xi+1 + 1 < 2xi ⇒xixi+1

< 2,∀i (2)

Từ (1) và (2) suy rax1x2

+x2x3

+ . . .xnx1

=∑

i6=k

xixi+1

+xkxk+1

< 2(n − 1) + 1 = 2n − 1.

Vậyx1x2

+x2x3

+ . . .xnx1

< 2n− 1.

Bài toán 1.16. Cho x1, x2, x3 ∈ [−1; 1] và y1, y2, y3 ∈ [0; 1) . Chứng minh rằng

1− x11− x2y3

+1− x21− x3y1

+1− x31− x1y2

≤ 8.

Lời giải. Vì 1− x1y2 = (1− x1)y2 + (1− y2) > 0, 1+ x1 ≥ 0 và 1− y2 > 0 nên ta có1− x11− x1y2

− 2

1 + y2= − (1 + x1)(1− y2)

(1 + y2)(1− x1y2)≤ 0

⇒ 0 ≤ 1− x11− x1y2

≤ 2

1 + y2≤ 2.

Tương tự ta cũng chứng minh được

0 ≤ 1− x21− x2y3

≤ 2 và 0 ≤ 1− x31− x3y1

≤ 2.

Suy ra1− x11− x1y2

1− x21− x2y3

1− x31− x3y1

=1− x11− x2y3

+1− x21− x3y1

+1− x31− x1y2

≤ 8. Dấu bằng xày ra khi

x1 = x2 = x3 = −1, y1 = y2 = y3 = 0

Bài toán 1.17. Cho a, b, c ∈ (0;π

2). Chứng minh rằng

∑ sina.sin(a− b).sin(a− c)sin(b+ c)

≥ 0.

Lời giải. Đặt x = sina, y = sinb, z = sinc. Khi đó x, y, z > 0.

Ta thấy sina.sin(a − b).sin(a − c)sin(a + b)sin(a + c) = sina(sin2a − sin2b)(sin2a −sin2c) = x(x2 − y2)(x2 − z2)Do đó cần chứng minh x(x2−y2)(x2−z2)+y(y2−z2)(y2−x2)+z(z2−x2)(z2−y2) ≥ 0

Đặt x =√u, y =

√v, z =

√w. Bất đẳng thức trở thành

sum√u(u− v)(u− w) ≥ 0 đúng theo bất đẳng thức Schur.

31

www.VNMATH.com

Page 33: [Vnmath.com] Ky Yeu Trai He Hv 2012

Bài toán 1.18. Cho x, y, z ∈ [1

2; 2] và a, b, c là một hoán vị tuỳ ý của chúng .

Chứng minh rằng60a2 − 1

4xy + 5z+

60b2 − 1

4yz + 5x+

60c2 − 1

4zx+ 5y≥ 12.

Lời giải. Do x, y, z ∈ [1

2; 2]nên (x− 2)(2y − 1) + (y − 2)(2x− 1) ≤ 0

⇒ 4xy ≤ 5(x+ y)− 4⇒ 0 < 4xy + 5z ≤ 5(x+ y + x)− 4

Do 60a2 − 1 > 0 nên60a2 − 1

4xy + 5z≥ 60a2 − 1

5(x+ y + z)− 4.

Tương tự ta cũng chứng minh được60b2 − 1

4yz + 5x≥ 60b2 − 1

5(x+ y + z)− 4và

60c2 − 1

4zx+ 5y≥

60c2 − 1

5(x+ y + z)− 4.

Từ đó suy ra60a2 − 1

4xy + 5z+

60b2 − 1

4yz + 5x+

60c2 − 1

4zx+ 5y≥ 60(a2 + b2 + c2)− 3

5(x+ y + z)− 4.

Do đó, ta chỉ cần chứng minh20(a2 + b2 + c2)− 1 ≥ 20(x+ y + z)− 16

Do a2 + b2 + c2 = x2 + y2 + z2 nên bất đẳng thức này tương đương với

(x2 + y2 + z2)− 20(x+ y + z) + 15 ≥ 0

⇔ 5(2x− 1)2 + 5(2y − 1)2 + 5(2z − 1)2 ≥ 0 luôn đúng.

Dấu bằng xảy ra khi x = y = z =1

2.

Bài toán 1.19. Cho a, b là các số thực dương thoả mãn điều kiện ab+ a+ b = 3 .

Chứng minh rằng3a

b+ 1+

3b

a+ 1+

ab

a+ b≤ a2 + b2 +

3

2.

Lời giải. Từ giả thiết a, b > 0 và ab+ a+ b = 3 ta suy ra ba điều sau:

(i) 3 = ab + a + b ≤ (a+b)2

4 + a + b ⇒ (a + b)2 + 4(a + b) − 12 ≥ 0 ⇒ a + b ≥ 2

hoặc a+ b ≤ −6. Do a+ b ≤ −6 không thể xảy ra nên a+ b ≥ 2 (1)

(ii) ab+ a+ b = 3⇒ ab

a+ b+ 1 =

3

a+ b⇒ ab

a+ b= −1 + 3

a+ b(2)

(iii) ab+ a+ b = 3⇒ (a+ 1)(b+ 1) = 4 (3)

Sử dụng (2) và (3) để biến đổi bất đẳng thức cần chứng minh ta được

a2 + b2 +3

2≥ 3a

b+ 1+

3b

a+ 1+

ab

a+ b= 3a

a+ 1

4+ 3b

b+ 1

4+

3

a+ b− 1

=3

4(a2 + b2) +

3

4(a+ b) +

3

a+ b− 1

hay 4(a2 + b2) + 6 ≥ 3(a2 + b2) + 3(a+ b) +12

a+ b− 4

⇔ a2 + b2 ≥ 3(a+ b) +12

a+ b− 10. (4)

Để ý rẳng a2+b2 ≥ (a+ b)2

2nên (4) sẽ được chứng minh nếu bất đẳng thức sau đúng.

32

www.VNMATH.com

Page 34: [Vnmath.com] Ky Yeu Trai He Hv 2012

(a+ b)2

2≥ 3(a+ b) +

12

a+ b− 10. (5)

Đặt s = a+ b. Từ (1) suy ra s ≥ 2. khi đó bất đẳng thức (5) trở thành

s2 − 6s− 42

s+ 20 ≥ 0⇔ (s− 2)(s2 − 4s+ 12) ≥ 0∀s ≥ 2 (luôn đúng).

Dấu bằng xảy ra khi s = 2⇔ a = b = 1 .

2 Một số bài toán tương tự

Bài toán 2.1. Cho x, y, z là các số thực thoả mãn điều kiện

x2 + y2 + z2 = 2. Chứng minh rằng

x+ y + z ≤ xyz + 2.

Bài toán 2.2. Cho x1, x2, . . . , x2012 ∈ (0; 1) Chứng minh rằng

2011√x1, x2, . . . , x2012 +

2011√

(1− x1)(1− x2), . . . , (1− x2012) < 1.

Bài toán 2.3. Cho∑1990

i=1 | xi − xi+1 |= 1991. Đặt sn =x1 + x2 + · · ·+ xn

n.

Chứng minh rằng

| s1 − s2 | + | s2 − s3 | + · · ·+ | s1990 − s1991 |≥ 1990.

Bài toán 2.4. Cho b3 + b ≤ a− a3 .

Tìm giá trị lớn nhất của T = a+ b.

Bài toán 2.5. Cho x, y, z là các số thức dương thoả mãn điều kiện x2+2x(y+ z) =

5yz.

Chứng minh rằng

(x+ y)3 + (x+ z)3 + (x+ y)(x+ z)(y + z) ≤ (y + z)3.

Bài toán 2.6. Cho a, b, c là các số thức dương thoả mãn điều kiện abc = 1.

Chứng minh rằng

(1

a+

1

b+)(

1

a+ 1+

1

b+ 1+

1

c+ 1) ≥ 9

2.

Bài toán 2.7. Cho a, b, c là các số thực thoả mãn điều kiện ab+ bc+ ca ≥ 0.

Chứng minh rằngab

a2+ b2 +

bc

b2 + c2≥ −1

2.

33

www.VNMATH.com

Page 35: [Vnmath.com] Ky Yeu Trai He Hv 2012

Bài toán 2.8. Cho hai tam giácA1B1C1 và A2B2C2 với diện tích S1, S2 và các cạnh

tương ứng a1, b1, c1 và a2.b2, c2 .Chứng minh rằng

a21(c22 + b22 − a22) + b21(a

22 + c22 − b22) + c21(b

22 + a22 − c22) ≥ 16S1S2.

Bài toán 2.9. Cho a, b, c là ba cạnh của một tam giác.

Chứng minh rằng

a2b(a− b) + b2c(b− c) + c2a(c− a) ≥ 0.

Bài toán 2.10. Cho a, b, c là các số thực không âm thoả mãn điều kiện a+b+c = 1.

Chứng minh rằng

0 ≤ xy + yz + zx ≤ 7

27.

Bài toán 2.11. Cho a, b, c là các số thực thoả mãn điều kiện

a+ b+ c = 0 và a2 + b2 + c2 = 6.

Chứng minh rằng

0 ≤ x2y + y2z + z2x ≤ 6.

34

www.VNMATH.com

Page 36: [Vnmath.com] Ky Yeu Trai He Hv 2012

35Các chuyên đề Toán Trại hè Hùng Vương năm 2012, Cao Bằng 01-04/08/2012

Sử dụng hệ thức truy hồi trong bàitoán đếm

Nguyễn Thế Hiệp, Lớp 11 Toán, Trường THPT Chuyên Bắc Giang

Bài toán đếm trong tổ hợp là một bài toán khá thông dụng đặc biệt với học sinh chuyên

toán. Việc đếm trực tiếp trong đa số các trường hợp là ”bất khả thi”, điều này hướng chúng

ta tới một phương thức đếm khác là đếm gián tiếp. Cùng với các phương pháp khác như:

thiết lập song ánh hay sử dụng hàm Sinh, vận dụng số phức, ... thì phương pháp thiết lập

quan hệ truy hồi cũng là một phương pháp khá hữu dụng và áp dụng rộng rãi, phổ biến

đối với nhiều bài toán khác nhau. Trong bài viết này chúng ta sẽ đề cập đến vấn đề đó.

1 Thiết lập quan hệ đơn truy hồi - một dãy truy hồi

Chúng ta sẽ mở đầu bằng một ví dụ đơn giản và khá quen thuộc từ bậc THCS. Đó

là bài toán tháp cổ Hà Nội.

Ví dụ 1.1 (Bài toán tháp cổ Hà Nội). Lúc khai thiên lập địa, Phật tổ Như

Lai đặt 64 chiếc đĩa có kích thước khác nhau lên một cái cọc, đĩa ở trên bé hơn đĩa

ở dưới. Các nhà sư được yêu cầu chuyển đĩa từ cọc 1 sang cọc 2 theo nguyên tắc:

Mỗi lần chỉ chuyển một đĩa; trong quá trình chuyển, đĩa lớn không được đặt lên

trên đĩa nhỏ (cần sử dụng thêm một cọc nữa để thực hiện nguyên tắc này). Tìm

số lần chuyển đĩa để có thể chuyển tất cả đĩa ở cọc 1 sang cọc 2.

Lời giải. Gọi an là số lần chuyển n chiếc đĩa từ cọc 1 sang cọc số 2 theo các

nguyên tắc trên.

Để chuyển n chiếc đĩa từ cọc 1 sang cọc 2 ta phải thực hiện các công đoạn sau:

1. Chuyển n− 1 chiếc đĩa bên trên chiếc đĩa lớn nhất sang cọc số 3 sang nguyên

tắc trên. Cần thực hiện an−1 lần chuyển như vậy.

2. Chuyển chiếc đĩa lớn nhất sang cọc số 2

3. Chuyển n − 1 chiếc đĩa từ cọc số 3 sang cọc số 2. Cần thực hiện an−1 lần

chuyển.

35

www.VNMATH.com

Page 37: [Vnmath.com] Ky Yeu Trai He Hv 2012

Như vậy ta thu được hệ thức an = 2an−1 + 1

Với chú ý rằng a1 = 1 ta có an = 2n − 1. Như vậy a64 = 264 − 1.

Ví dụ 1.2 (Bài toán chia kẹo Euler). Có bao nhiêu cách chia m cái kẹo cho n

đứa trẻ?

Lời giải. Bài toán tương đương với bài toán tìm số nghiệm tự nhiên của

phương trình

m = x1 + x2 + · · ·+ xn. (1)

Hiển nhiên,

(1)⇔ m− x1 = x2 + · · ·+ xn. (2)

Gọi S (m;n) là số nghiệm của phương trình (1). Khi đó với mỗi giá trị của x1; (0 ≤ x1 ≤ m)

ta có số nghiệm của phương trình (2) là S (m− x1;n− 1).

Từ đó ta thu được hệ thức S (m;n) =∑m

x=0 S (m− x;n− 1). Chú ý rằng

S (m; 1) = 1;S (m; 2) = m+ 1;S (m; 3) =(m+ 2) (m+ 1)

2

Bằng quy nạp ta có S (m;n) = Cn−1m+n−1.

Thật vậy, giả sử mệnh đề đúng với n, ta chứng minh nó đúng với n+ 1

S (m;n+ 1) =

m∑x=0

S (m− x;n) =

m∑x=0

Cn−1m−x+n−1

=

m∑x=0

Cn−1x+n−1 =

m∑x=0

(Cnx+n − Cn

x−1+n) = Cnm+n ()

Nhận xét 1.1. Đây là một bài toán khá đơn giản nhưng cũng có nhiều ứng dụng

chẳng hạn trong bài thi VMO 2012 (bài số 6, ngày 2). Ngoài lời giải trên, ta cũng

có thể giải này bằng phương pháp song ánh hoặc hàm Sinh.

Ví dụ 1.3 (Mở rộng Pro.3-Canada MO 1996). Tìm số hoán vị (a1; a2; ...; an)

của tập 1; 2; 3; ...;n thỏa mãna1 = 1

|ai+1 − ai| ≤ 2, ∀i = 1; 2; ...;n− 1

Lời giải. Gọi xn là số các hoán vị của 1; 2; ...;n thỏa mãn bài toán và An là

tập hợp các hoán vị đó.

Ta có với n ≥ 5 xét hoán vị (a1; a2; ...; an) thỏa mãn tính chất đã cho.

Vì |a2 − a1| = |a2 − 1| < 2 nên a2 = 2 ∨ 3 do đó chỉ xảy ra các trường hợp sau:

36

www.VNMATH.com

Page 38: [Vnmath.com] Ky Yeu Trai He Hv 2012

1. Nếu a2 = 2 thì xét (b1; b2; ...; bn−1) ≡ (a2 − 1; ...; an − 1) và (b1; b2; ...; bn−1) là một

hoán vị của 1; 2; ...;n− 1 mà

b1 = 1; |bi−1 − bi| = |ai − ai+1| ≤ 2, i = 1; 2; ...;n− 1.

Suy ra (b1; b2; ...; bn−1) ∈ An−1. Hơn nữa, (a1; 2; a3; ...; an) ∈ An và phép tương

ứng

(a1; 2; a3; ...; an)→ (a1 − 1; a2 − 1; ...; an − 1)

là một song ánh nên có xn−1 hoán vị mà a2 = 2.

2. Nếu a2 = 3 ta có các khả năng sau:

• Nếu ai = 2 với mọi 3 < i < n thì ai−1 = ai+1 = 4, vô lí. Ta xét a3 = 2∨an =

2.

• Nếu an = 2 thì an−1 = 4; a3 = 5, có một bộ số thỏa mãn là(1; 3; ...; 2

[n+ 1

2

]− 1; 2

[n

2

]; 2[n

2

]− 2; ....; 2

).

• Nếu a3 = 2 thì a4 = 4, mỗi bộ số (a1; a2; ...; an) tương ứng với một bộ số

(a4 − 3; a5 − 3; ....; an − 3) ∈ An−3. Vậy nên có xn−3 hoán vị như thế.

Ta có hệ thức xn = xn−1 + xn−3 + 1, n ≥ 5. Chú rằng x1 = x2 = 1; x3 = 2; x4 = 4, từ

đây có thể tìm được số bộ thỏa mãn theo n.

Ví dụ 1.4 (Swedish MO 2005-2006). Một hàng gồm n người đứng trước một

máy thu tiền. Sau đó máy thu tiền đóng lại vì một lí do kĩ thuật và n người đó được

bố trí lại trong một hàng khác. Hỏi có bao nhiêu cách xếp hàng mà nếu mỗi người

trong hàng hoặc đứng nguyên vị trí ban đầu hoặc đứng ở vị trí liền trước hoặc đứng

ở vị trí liền sau.

Lời giải. Gọi an là số hoán vị f của tập hợp 1; 2; 3; ...;n thỏa mãn tính chất

k − 1 ≤ f(k) ≤ k + 1 với 1 ≤ k ≤ n, khi đó ta có f(n) ∈ n− 1;n.Với n ≥ 3 nếu f có tính chất đó thì ta có:

1. Nếu f(n) = n − 1, thì f(n − 1) = n các số 1; 2; ...;n − 2 có an−2 hoán vị thỏa

mãn tính chất nêu trên.

2. Nếu f(n) = n, các số 1; 2; 3; ...;n− 1 có an−1 hoán vị thỏa mãn tính chất đó.

37

www.VNMATH.com

Page 39: [Vnmath.com] Ky Yeu Trai He Hv 2012

Ta có hệ thức an = an−1 + an−2. Chú ý rằng a1 = 1; a2 = 2 ta được

an = Fn+1 =1√5

((1 +√5

2

)n+1

+

(1−√5

2

)n+1)

,

trong đó Fn là số hạng thứ n trong dãy số Fibonacci.

Trường hợp n = 12 là bài toán gốc.

Ví dụ 1.5 (Romanian MO 1995). Cho A1;A2; ...;An là các điểm nằm trên đường

tròn. tìm số cách tô các điểm đó bằng p ≥ 2 màu sao cho hai điểm liên tiếp được

tô hai màu phân biệt.

Lời giải. Gọi an, n ≥ 2 là số cách tô màu cần tìm. Xét tập hợp điểm A1;A2; ...;An+1.

1. Nếu A1, An khác màu nhau thì có thể tô màu A1, A2, ..., An bởi an cách và

tô màu An+1 theo p−2 cách. Do đó chúng ta thu được (p− 2) an cách tô màu.

2. Nếu A1, An được tô màu giống nhau thì đồng nhất A1 ≡ An ta thu được an−2

cách tô màu các điểm A1, A2, ..., An, trong khi có (p− 1) cách tô màu An+1.

Ta thu được (p− 1) an−2 cách tô màu.

Do đó ta có hệ thức an+1 = (p− 2) an + (p− 1) an−1.

Chú ý rằng a2 = p(p−1); a3 = p(p−1)(p−2), ta được an = (p− 1)n+(−1)n (p− 1) .

Ví dụ 1.6 (Bulgarian MO 1998). Cho số nguyên dương n ≥ 2. Hãy tìm số các

hoán vị (a1; a2; ...; an) của tập 1; 2; 3; ...;n sao cho tồn tại duy nhất một chỉ số thỏa

mãn ai > ai+1.

Lời giải. Gọi xn là số các hoán vị thỏa mãn điều kiện bài toán. Mỗi hoán vị có

n+ 1 phần tử có thể tạo ra bằng một trong các cách sau đây:

1. Bổ sung thêm an+1 = n+1 vào mỗi hoán vị có n phần tử thỏa mãn điều kiện

bài toán, có tất cả xn hoán vị như vậy.

2. Bổ sung thêm phần tử aj = n+ 1vào giữa ai, ai+1 mà ai > ai+1 của một hoán

vị nào đó có n phần tử thỏa mãn bài toán.

3. Bổ sung thêm aj = n + 1, j = 0; 1; 2; ...;n − 1vào hoán vị mà n phần tử được

sắp thứ tự.

Do vậy thu được xn+1 = xn + xn + n = 2xn + n.

Chú ý rằng x2 = 1, từ đó ta có xn = 2n − n− 1.

38

www.VNMATH.com

Page 40: [Vnmath.com] Ky Yeu Trai He Hv 2012

Ví dụ 1.7. Có bao nhiếu số có n chữ số chia hết cho 5 và hai chữ số liên tiếp bất

kì khác nhau.

Lời giải. Gọi An là tập hợp các số có n chữ số thỏa mãn bài toán. Đặt an =

card (An), mỗi số thuộc An+1 được tạo ra bằng một trong hai cách sau đây:

1. Thêm vào trước mỗi số thuộc An một chữ số khác với chữ số đầu tiên của số

đó, có tất cả 8 cách thêm như vậy.

2. Bỏ chữ số đầu tiên của của một số thuộc An (sau khi bỏ như thế chỉ còn an−2

số phân biệt có n− 2 chữ số là bội của 5 và các chữ số liên tiếp là khác nhau)

và thêm vào trước đó hai chữ số 10; 20; 30; ...; 90; có tất cả 9 cách thêm như

thế.

Do đó ta có hệ thức an+2 = 8an+1 + 9an

Với chú ý rằng a1 = 2, a2 = 17 ta thu được an =19

90· 9n − (−1)n

10

Ví dụ 1.8. Tìm số tập con của tập 1; 2; ...;n sao cho trong mỗi tập con chứa

đúng hai phần tử là hai số nguyên liên tiếp.

Lời giải. Trước hết ta có bài toán phụ: Tìm số tập con của tập 1; 2; ...;nsao cho trong mỗi tập con không chứa hai phần tử là hai số nguyên

liên tiếp.

Bài toán này xem như bài tập (giải bằng truy hồi hoặc song ánh). Số tập con thỏa

mãn bài toán là

xn = Fn+2 =1√5.

((1 +√5

2

)n+2

+

(1−√5

2

)n+2)

,

trong đó Fn là số hạng thứ n trong dãy số Fibonacci.

Gọi An là tập các tập con của tập 1; 2; ...;n thỏa mãn điều kiện bài toán, đặt

an = card(An). Mỗi tập thuộc A ∈ An+2 được chia thành 3 loại: Loại 1 gồm các tập

hợp chứa cả 2 phần tử: n+1, n+2 ; loại 2 gồm các tập hợp không chứa n+2 ; loại

3 gồm các tập hợp chứa n+ 2 nhưng không chứa n+ 1.

1. Nếu A là tập con loại 1 thì A không chứa n. Bỏ đi khỏi A hai phần tử n+1, n+2

ta được một tập con của tập 1; 2; ...;n− 1 và không chứa hai số nguyên liên

tiếp. Như vậy có xn−1 = Fn+1 tập con loại 1.

2. Mỗi tập con loại 2 là một tập con của An+1 và ngược lại. Có an+1 tập loại 2.

39

www.VNMATH.com

Page 41: [Vnmath.com] Ky Yeu Trai He Hv 2012

3. Nếu A là tập con loại 3 thì A không chứa n + 1. Bỏ đi khỏi A phần tử n + 2

ta được một tập con của An. Vậy có an tập con loại 3.

Do đó ta có hệ thức

an+2 = an+1 + an + Fn+1.

Chú ý rằng a2 = 1; a3 = 2 ta thu được

an =2nFn+1 − (n+ 1)Fn

5. ()

Ví dụ 1.9 (Colombia MO 1997). Xét bảng ô vuông m× n được tô các viền bởi

3 màu. Chúng ta tô mỗi đoạn của đường viền bởi một trong số 3 màu sao cho mỗi

hình vuông đơn vị có hai cạnh tô 1 màu và hai cạnh kia tô bởi hai màu còn lại. Có

bao nhiêu cách tô màu có thể?

Lời giải. Gọi 3 màu là A,B,C; đặt an là số cách tô màu đường viền của một

bảng ngang 1× n khi đã biết màu của các đoạn viền phía trên bảng ngang đó. Với

n = 1, không mất tính tổng quát giả sử viền trên được tô màu A. Sau đó sẽ có 3

cách chọn tô màu 3 đoạn kia bằng màu A, có hai cách chọn tô màu hai đoạn còn

lại. Do vậy a1 = 6.

Xét bảng 1× n, không mất tính tổng quát giả sử đoạn cuối cùng bên phải tô màu

A. Bổ sung thêm một hình vuông đơn vị vào bên phải bảng vừa rồi ta được một

bảng 1× (n+ 1) với màu đoạn viền trên của hình vuông mới đã biết. Chúng ta có

hai cách tô màu cho hình vuông đã biết màu hai cạnh này (cạnh bên trái và cạnh

trên). Do đó an+1 = 2an, an = 3.2n

Trở lại bài toán, có 3n cách tô màu cho đường viền trên cùng và có 3 · 2n cách tô

cho mỗi hàng kế tiếp. Vậy có tất cả 3m+n · 2mn cách tô màu thỏa mãn bài toán.

Ví dụ 1.10 (Hungary-Irael Mathematical Competition 1997). Có tất cả

bao nhiêu dãy có độ dài bằng 1997 có thể lập được bằng cách sử dụng số lẻ lần các

chữ cái A,B,C.

Lời giải. Gọi xn là số dãy có độ dài bằng n = 2m + 1. Với mỗi dãy có độ dài

bằng n = 2m+ 1 ta có:

1. Nếu dãy đó thỏa mãn điều kiện thì ta có 3 cách thêm hai chữ cái giống nhau

vào cuối mỗi dãy để thu được các dãy có độ dài bằng n+ 2.

40

www.VNMATH.com

Page 42: [Vnmath.com] Ky Yeu Trai He Hv 2012

2. Nếu dãy đó không phải là dãy thỏa mãn điều kiện bài toán, tuy nhiên số lần

xuất hiện của A,B,C không thể đều là số chẵn nên sẽ có hai chữ cái xuất

hiện số chẵn lần và một chữ cái xuất hiện số lẻ lần trong mỗi dãy này. Thành

thử ta có hai cách thêm hai chữ cái vào cuối các dãy này để được dãy có độ

dài bằng n+ 2

Vì vậy, xn+2 = 3xn + 2 (3n − xn) = xn + 2 · 3n. Kết hợp với x3 = 6 ta có xn =3

4

(3n−1 − 1

)hay x2m+1 =

3

4

(32m − 1

), suy ra x1997 =

3

4

(31996 − 1

).

Ý tưởng truy hồi đối với nhiều bài toán là rất rõ ràng, song thực hiện nó đôi

khi không hề dễ dàng, chẳng hạn với bài IMO Shortlist 2008.

Ví dụ 1.11 (IMO Shortlist 2008). Với mỗi số nguyên dương n, hãy tìm số

hoán vị của tập hợp 1; 2; ...;n có tính chất 2 (a1 + a2 + · · ·+ an)...k với mọi số k =

1; 2; ...;n.

Lời giải. Với mỗi số nguyên dương n đặt Fn là số hoán vị của 1; 2; ...;n thỏamãn tính chất đã cho, ta gọi chúng là hoán vị đẹp.

Với n = 1, 2, 3 ta có số hoán vị đẹp là F1 = 1, F2 = 2, F3 = 6.

Với n > 3 xét hoán vị đẹp bất kì (a1; a2; ...; an) của 1; 2; ...;n. Khi đó ta có

(n− 1)|2 (a1 + a2 + · · ·+ an−1) = 2 ((1 + 2 + · · ·+ n)− an)

= n (n+ 1)− 2an = (n+ 2) (n− 1) + (2− 2an)

nên (2an − 2)...n − 1. Vì vậy (2an − 2) = 0;n − 1; 2n − 2. Điều đó đồng nghĩa với

a = 1 hoặc a =n+ 1

2hoặc a = n Giả sử a =

n+ 1

2, vì là một hoán vị đẹp nên khi

cho k = n− 2 ta có

(n−1)|2 (a1 + a2 + · · ·+ an−2) = (n+ 1)n−(n+ 1)−2an−1 = (n+ 2) (n− 2)+(3− 2an−1) .

Do đó 2an−1 − 3... (n− 2) nên 2an−1 − 3 = 0;n− 2; 2n− 4. Loại trường hợp 2an−1 − 3

bằng 0; 2n − 4 vì 2an−1 − 3 là số lẻ. Khả năng còn lại 2an−1 − 3 = n − 2 không thể

xảy ra vì khi đó an−1 =n+ 1

2= an. Điều đó có nghĩa là

n+ 1

2không là giá trị mà

an−1 có thể nhận. Xét hai trường hợp sau:

1. Nếu an = n thì (a1; a2; ...; an−1) là một hoán vị đẹp của 1; 2; ...;n− 1.Có Fn−1 hoán vị như thế. Bổ sung n vào cuối của mỗi hoán vị này ta được

một hoán vị đẹp của tập 1; 2; ...;n.

41

www.VNMATH.com

Page 43: [Vnmath.com] Ky Yeu Trai He Hv 2012

2. Nếu an = 1 thì là (a1 − 1; a2 − 1; ...; an−1 − 1)một hoán vị đẹp của 1; 2; ...;n− 1vì

2 ((a1 − 1) + · · ·+ (ak − 1)) = 2 (a1 + · · ·+ ak)− 2k... k,∀k ≤ n− 1.

Mỗi hoán vị (b1; b2; ...; bn−1) trong số hoán vị đẹp của 1; 2; ...;n− 1 cho ta một

hoán vị đẹp của tập 1; 2; ...;n có phần tử cuối là 1. Đó là (b1 + 1; b2 + 1; ...; bn−1 + 1; 1).

Hai phép tương ứng song ánh mà chúng ta đã thiết lập ở trên cho chúng ta hệ thức

Fn = 2Fn−1. Với chú ý rằng F1 = 1, F2 = 2, F3 = 6 ta thu được

Fn = 3 · 2n−2, n ≥ 3. ()

Ví dụ 1.12 (IMO 2011). Giả sử n là một số nguyên dương. Cho một cái cân

hai đĩa và n quả cân với trọng lượng là 20; 21; ...; 2n−1. Ta muốn đặt lên cái cân một

trong n quả cân, lần lượt từng quả một, theo cách để đảm bảo đĩa cân bên phải

không bao giờ nặng hơn đĩa cân bên trái. Ở mỗi bước ta chọn một trong các quả

cân chưa được đặt lên cân, rồi đặt nó hoặc vào đĩa bên trái, hoặc vào đĩa bên phải,

cho đến khi tất cả các quả cân đều đã được đặt lên cân. Xác định xem, có bao nhiêu

cách khác nhau để thực hiện được mục đích đề ra.

Lời giải. Xét n ≥ 2. Ta thấy trước tiên, đĩa cân bên trái luôn phải nặng hơn

đĩa cân bên phải ít nhất là 1. Vì vậy, nếu ở một bước nào đó ta đặt quả cân nặng

là 1 lên một trong hai đĩa thì thì đĩa cân bên phải không bao giờ nặng hơn đĩa cân

bên trái. Do đó ta chỉ xét các quả cân: 1; 22; ...; 2n−1.

Nếu ta chia trọng lượng mỗi quả cân cho 2 thì đáp án không đổi nên có f(n − 1)

cách đặt quả cân n− 1 đó. Giờ xét quả cân 1:

1. Nếu đặt quả cân này trong lượt đặt đầu tiên thì ta chỉ có thể đặt vào đĩa bên

trái.

2. Nếu đặt quả cân này trong các lượt đặt tiếp theo thì ta có thể đặt vào bên

trái hay phải tùy ý. Có 2n− 2 cách đặt trong trường hợp này.

Như vậy ta thu được hệ thức f(n) = (2n− 1)f(n− 1).

Với chú ý rằng f(1) = 1 ta có f(n) = (2n− 1)!! = 1 · 3 · 5 · · · · · (2n− 1).

Nhận xét 1.2. Nếu không xét riêng các khả năng của quả cân 1 như trên thì ta cũng

có thể xét riêng các khả năng của quả cân 2n−1 với chú ý rằng 20+21+ · · ·+2n−2 =

2n−1 − 1 < 2n.

42

www.VNMATH.com

Page 44: [Vnmath.com] Ky Yeu Trai He Hv 2012

2 Thiết lập các dãy truy hồi lồng nhau

Đếm bằng hệ thức truy hồi khá hữu dụng nhưng đối với đa số bài toán thì việc đếm bằng

một dãy truy hồi tỏ ra vô hiệu, ấy là khi ta phải nghĩ đến các dãy truy hồi lồng nhau

(cũng giống như ”DAC - Chia để trị” trong bất đẳng thức). Chúng ta sẽ bắt đầu ”công

cụ hữu hiệu ” này bằng một ví dụ đã từng xuất hiện trong kì thi IMO năm 1979, đó là

bài số 6 do Đức đề nghị.

Ví dụ 2.1 (IMO-1979). Cho A và E là hai đỉnh đối tâm của một hình bát giác

đều. Một con ếch bắt đầu nhảy từ A. Tại bất cứ đỉnh nào trừ E, ếch có thể tới một

trong hai đỉnh kề. Nếu ếch nhảy tới E thì nó dừng lại ở đó. Gọi anlà số đường đi

phân biệt của đúng n bước nhảy để ếch nhảy từ A đến E. Chứng minh rằng

a2n−1 = 0; a2n =1√2

[(2 +√2)n−1

−(2−√2)n−1

].

Lời giải. Rõ ràng a2n−1 = 0 là hiển nhiên. Gọi là số đường đi từ C tới E qua n

bước nhảy. Qua hai bước nhảy đầu tiên, ếch có thể về A hoặc đến C hoặc G. Do

đó

a2n = 2a2n−2 + 2b2n−2 với mọi n > 0.

Từ C (hoặc G), sau hai bước nhảy ếch có thể về lại C hoặc tới A, nếu n > 2. Do

đó ta có

b2n = 2b2n−2 + 2a2n−2 với mọi n > 1.

Suy ra a2n = 4a2n−2 − 2a2n−2

Kết hợp với a2 = 0; a4 = 2 ta có điều phải chứng minh.

Ví dụ 2.2 (Austrian-Polish MC 1998). Cho n điểm P1, P2, P3, ..., Pn được xếp

theo thứ thự đó trên một đường thẳng. Ta tô màu mỗi điểm bởi 1 trong năm màu:

trắng, đen, đỏ, xanh và tím. Một cách tô màu được coi là hợp lệ nếu hai điểm liên

tiếp Pi, Pi+1 được tô màu giống nhau hoặc ít nhất một trong hai điểm được tô màu

trắng. Có bao nhiêu cách tô màu hợp lệ như vậy?

Lời giải. Gọi an là số cách tô màu thỏa mãn bài toán mà điểm được tô màu

trắng, là bn số cách tô màu thỏa mãn bài toán mà điểm không được tô màu trắng

và cn là tổng số cách tô màu thỏa mãn bài toán. Chúng ta thu được các hệ thức

sauan = cn−1

bn = 4an−1 + bn−1

cn = an + bn

Do đó ta có cn = an + bn = cn−1 + bn = cn−1 + (an−1 + bn−1) + 3an−1 = 2cn−1 + 3cn−2.

43

www.VNMATH.com

Page 45: [Vnmath.com] Ky Yeu Trai He Hv 2012

Chú ý rằng c1 = 5, c2 = 13 ta thu được cn =3n+1 + (−1)n+1

2.

Ví dụ 2.3. Tìm số các bộ số nguyên (a1; a2; ...; an), n > 1 thỏa mãn |ai| ≤ 1, ∀i =1; 2; ...;n và |ai − ai+1| ≤ 1, ∀i = 1; 2; ...;n.

Lời giải. Gọi Sn là số bộ số thỏa mãn điều kiện bài toán. Gọi An, Bn, Cn là

tập hợp các bộ số bằng −1; 0; 1 tương ứng. Ta có |Sn| = |An|+ |Bn|+ |Cn|Mặt khác từ mỗi bộ số thuộc An hoặc Bn ta có thể bổ sung thêm phần tử an+1 = −1để được một bộ số thuộc An+1 nên |An+1| = |An|+ |Bn|. Tương tự ta có

|Cn+1| = |Cn|+ |Bn| ; |Bn+1| = |An|+ |Bn|+ |Cn| = |Sn| .

Do đó ta có

|Sn+1| = |An+1|+|Bn+1|+|Cn+1| = (|An|+ |Bn|+ |Cn|)+|Bn+1|+|Bn| = 2 |Sn|+|Sn−1| .

Chú ý rằng |S2| = 7; |S3| = 17 ta thu được |Sn| =(1 +√2)n+1

+(1−√2)n+1

2. ()

Một bài toán có cùng tư tưởng như bài toán trên là

Ví dụ 2.4 (Polish-Austrian Mathematical Competition). Xét tất cả các các

chuỗi kí tự gồm 3 chữ cái a, b, c. Có bao nhiêu chuỗi n kí tự đồng thời thỏa mãn hai

điều kiện sau:

1. Chuỗi kí tự phải bắt đầu và kết thúc bởi chữ a;

2. Hai vị trí liền kề phải được viết bởi hai chữ cái khác nhau.

Lời giải. Xét chuỗi kí tự có độ dài bằng n bắt đầu bởi chữ a và thỏa mãn điều

kiện 2.

Đặt an, bn, cn lần lượt là số chuỗi kí tự tận cùng bởi a, b, c tương ứng.

Mỗi chuỗi kí tự kết thúc bởi a và có độ dài bằng n+1 được tạo ra (và chỉ được tạo

ra) bằng cách thêm a vào cuối các chuỗi kí tự tận cùng bởi b hoặc c và có độ dài

bằng n, vì vậy ta thu được hệ thức an+1 = bn + cn. Tương tự ta có

bn+1 = an + cn ; cn+1 = an + bn.

Vì kí hiệu b, c chỉ mang tính hình thức, chúng có vai trò bình đẳng nên ta có cn = bn

Từ đó suy ra

an+1 = 2bn; bn+1 = an + bn ⇒ an+2 − an+1 = 2 (bn+1 − bn) = 2an.

Ta thu được hệ thức truy hồi an+2 = an+1 + 2an

Với chú ý rằng a1 = 1, a2 = 0 ta có an =2

3

(2n−2 + (−1)n−1) .

44

www.VNMATH.com

Page 46: [Vnmath.com] Ky Yeu Trai He Hv 2012

Ví dụ 2.5. Có bao nhiêu chuỗi gồm n kí tự được tạo ra từ tập hợp a; b; c; d trongđó a, b không bao giờ đứng cạnh nhau.

Lời giải. Gọi yn, yn, xn, xn lần lượt là số các chuỗi kí tự có tận cùng là a, b, c, d

(do tính đối xứng, số chuỗi kí tự tận cùng bằng a bằng số chuỗi kí tự tận cùng

bằng b và số chuỗi kí tự tận cùng bằng c bằng số chuỗi kí tự tận cùng bằng d) và

Sn là số cách thành lập chuỗi n kí tự thỏa mãn bài toán. Chúng ta thu được các

hệ thức

xn = 2xn−1 + 2yn−1; yn = 2xn−1 + yn−1.

Từ đó suy ra Sn = xn+1 và xn+1 = 3xn + 2xn−1.

Với chý ý rằng x1 = 1;x2 = 2, ta thu được

Sn = xn+1 =

(7√17

68− 1

4

)(3 +√17

2

)n+1

+

(−7√17

68− 1

4

)(3−√17

2

)n+1

. ()

Bài toán sau đây là một bài toán đã được đăng trên tạp chí THTT năm 2011 (bài

T12/410), tuy nhiên nó cũng đã xuất hiện trước đó rất lâu trong kì thi Romanian

MO 2000.

Ví dụ 2.6 (Romanian MO 2000-T12/410-THTT)). Với số nguyên dương n

không nhỏ hơn 2. tìm số các hàm số f : 1; 2; 3; ...;n → 1; 2; 3; 4; 5 và thỏa mãn

tính chất

|f(k + 1)− f(k)| ≥ 3 với mọi k ∈ 1; 2; ...;n− 1 .

Lời giải. Ta sử dụng nhận xét:

Nếu hàm số f thỏa mãn điều kiện bài ra thì với mọi n > 2 cho trước ta luôn có

f(n) 6= 3.Thật vậy, nếu f(n) = 3 thì suy ra f(n− 1) ≤ 0 hoặc f(n− 1) ≥ 6, điều này là vô lí.Kí hiệu an, bn, dn, en là số các hàm f : 1; 2; 3; ...;n → 1; 2; 3; 4; 5 thỏa mãn tính chất đã choứng với f(n) tương ứng bằng 1,2,4,5.Khi đó a2 = e2 = 2 và b2 = d2 = 1, nên với n ≥ 2 có

an+1 = dn + en ; bn+1 = en ; en+1 = an + bn ; dn+1 = an.

Ta cần tính tổng Sn = an + bn + dn + en.Ta có a2 = e2 = 2 và b2 = d2 = 1. Bằng phương pháp quy nạp, ta có an = en,bn = dn.Do vậy an+2 = an+1 + an, Sn = 2an+1. Suy ra

Sn =

(5− 3

√5

5

)(1−√

5

2

)n

+

(5 + 3

√5

5

)(1 +√

5

2

)n

. ()

Ví dụ 2.7 (Romanian TST 2006-Tạp chí AMM). Cho n là một số nguyên dương. Một tậpS ⊂ 0; 1; ...; 4n− 1 được gọi là ”rời rạc ” nếu với số k bất kì thỏa mãn điều kiện sau đây:

45

www.VNMATH.com

Page 47: [Vnmath.com] Ky Yeu Trai He Hv 2012

1. Tập hợp S⋂4k − 2; 4k − 1; 4k; 4k + 1; 4k + 2 có tối đa 2 phần tử.

2. Tập hợp S⋂4k + 1; 4k + 2; 4k + 3 có tối đa 1 phần tử.

Hỏi tập hợp 0; 1; ...; 4n− 1 có chính xác bao nhiêu tập con ”rời rạc”?

Lời giải. Đặt Tn là tổng số tập con ”rời rạc” cần đếm, An là số tập con ”rời rạc” bao gồmmột trong hai phần tử (4n− 1; 4n− 2) và Bn là số tập con không chứa phần tử nào trong haiphần tử đó (không có hai tập ”rời rạc” có thể chứa cả 2 phần tử vì khi đó điều kiện 2. khôngthỏa mãn).

Ta thu được các hệ thức

An+1 = Tn + Tn + Tn +Bn = 2Tn +Bn.

Và Bn+1 = Tn + Tn + Tn +Bn = 2Tn +Bn, suy ra Tn+1 = 6Tn + 2Bn = 7Tn.

Với chú ý rằng A1 = B1 = 4;T1 = 8, ta có Tn = 8 · 7n−1.

Phép đếm bằng truy hồi cũng được áp dụng trong việc đếm số các sự phân hoạch của hìnhvẽ. Ngay sau đây chúng ta sẽ tiếp tục với một bài toán đã từng được đăng trên tạp chí AmericanMathematical Monthly do I. Tomescu đề xuất.

Ví dụ 2.8 (Tạp chí AMM). Có bao nhiêu cách chia hình chữ nhật kích thước 2n × 3 thànhcác hình chữ nhật kích thước 2× 1.

Lời giải. Gọi an là số cách chia hình chữ nhật 2n× 3 thành các hình chữ nhật 2× 1, cònbn là số cách hình chữ nhật 2n × 3 bị khuyết một hình vuông 1 × 1 ở góc thành các hình chữnhật 2× 1.

Với mỗi cách chia hình chữ nhật 2n× 3 mà không có hình vuông 1× 1 nào bị khuyết ở góc tacó 3 cách thêm 3 hình chữ nhật để tạo thành một hình chữ nhật 2(n+ 1)× 3.

Và ta cũng có hai cách tạo hình chữ nhật 2(n+ 1)× 3 từ hai hình chữ nhật kích thước 2n× 3

bị khuyết hình vuông 1× 1 ở góc. Do đó ta có an+1 = 3an + 2bn

Tương tự ta cũng có bn+1 = an + bn.

Chú ý rằng a1 = 3, b1 = 1, a2 = 3 · 3 + 2 · 1 = 11 ta thu được

an =1

2√

3

[(√3 + 1

)(2 +√

3)n

+(√

3− 1)(

2−√

3)n]

. ()

Đối với bài toán đếm số cách phân hoạch hình vẽ bằng phương pháp truy hồi thì ta cũng cầnchuyển nó về ngôn ngữ đại số để dễ ”ăn nói” trong quá trình trình bày lời giải bài toán. Điềunày được minh chứng trong ví dụ 2.9.

Ví dụ 2.9 (Tukey TST 2006). Tìm số cách chia hình chữ nhật n×2 thành các hình chữ nhậtcó độ dài nguyên không vượt quá n. (Hai cách chia tạo bởi nhau bằng phép quay 180 được coilà hai cách chia phân biệt)

46

www.VNMATH.com

Page 48: [Vnmath.com] Ky Yeu Trai He Hv 2012

Lời giải. Xét hình chữ nhật với đỉnh (0; 0) , (0; 2) , (n; 0) , (n; 2) trong mặt phẳng Descaster.Một sự phân hoạch có thể biểu diễn bởi tập E các đoạn thẳng mà tạo thành những đường viền(khác với viền hình chữ nhật lớn) của hình chữ nhật nhỏ.Ta gọi 1 sự phân hoạch là loại A nếu [(n− 1; 1), (n; 1)] ∈ E và là loại B nếu [(n− 1; 1), (n; 1)] /∈ E.Với mỗi sự phân hoạch và với 0 ≤ k ≤ 2, 0 ≤ j ≤ 1, đặt tập

Ω = [(k;n− 1), (k;n)] , [(n; j), (n; j + 1)] ∪ [(n− 1; 0), (n− 1; 1)] , [(n− 1; 1) , (n− 1; 2)] .

Dễ thấy rằng E′ \ Ω tạo thành một sự phân hoạch hình chữ nhật 2× (n− 1) với đỉnh là

(0; 0) , (0; 2) , (n− 1; 0) , (n− 1; 2) .

Nếu E′ là tập loại A nghĩa là [(n− 1; 1), (n; 1)] ∈ E, khi đó có 5 tập E tương ứng có thể baogồm 4 tập loại A và 1 tập loại B.

Nếu E′ là tập loại B nghĩa là [(n− 1; 1), (n; 1)] /∈ E, khi đó có 3 tập E tương ứng có thể baogồm 1 tập loại A và 2 tập loại B.Đặt An = card(A), Bn = card(B), Cn = An +Bn ta có An = 4An−1 +Bn−1;Bn = An−1 +2Bn−1

Cn = An +Bn = 6(An−1 +Bn−1)− 7(An−2 +Bn−2) = 6Cn−1 − 7Cn−2.

Với chú ý rằng C1 = 2, C2 = 8 ta thu được Cn =2 +√

2

2

(2 +√

3)n−1

+2−√

2

2

(√3− 2

)n−1.

Ví dụ 2.10 (Romanian MO 2003). Cho n là một số nguyên dương. Có bao nhiêu số có nchữ số từ tập hợp 2; 3; 7; 9 và chia hết cho 3.

Lời giải. Gọi An là tập hợp các số có n chữ số lập từ tập 2; 3; 7; 9và chia hết cho 3, và Bn làtập hợp các số có n chữ số lập từ tập 2; 3; 7; 9 và không chia hết cho 3. Cần tìm an = card(An)

Đặt bn = card(Bn). Ta thấy mỗi số thuộc An+1 chỉ có thể thu được bằng hai cách sau:

• Lấy một số thuộc An rồi thêm 3 hoặc 9 vào phía sau (có 2 cách thêm).

• Lấy một số thuộc Bn rồi thêm 2 hoặc 7 vào phía sau nhưng chỉ có duy nhất một cáchthêm với mỗi số.

Suy ra an+1 = 2an + bn, tương tự ta có bn+1 = 2an + 3bn.

Từ đó ta có an+2 = 5an+1 − 4an, với chú ý a1 = 2, a2 = 6 ta được an =4n + 2

3.

Ví dụ 2.11 (IMO Shorlist 1987). Có bao nhiêu số có n chữ số từ tập các số 0; 1; 2; 3; 4thỏa mãn hai chữ số liên tiếp hơn kém nhau tối đa là 1 đơn vị.

Lời giải. Gọi xn là số các số có n chữ số thỏa mãn điều kiện bài toán, và yn, zn, un, zn, yn lầnlượt là số các số có n chữ số mà chữ số khởi đầu lần lượt là 0,1,2,3,4 (do tính đối xứng nên số cácsố bắt đầu bởi 0 bằng số các số bắt đầu bởi 4; số các số bắt đầu bởi 2 bằng số các số bắt đầu bởi 3).

47

www.VNMATH.com

Page 49: [Vnmath.com] Ky Yeu Trai He Hv 2012

Chúng ta thu được các hệ thức sau:

(1) yn = zn−1; (2) zn = yy−1 + un−1

(3) un = 2zn−1; (4) xn = 2yn + 2zn + un.

Từ (1),(2) và (3) ta có zn = zn−2 + 2zn−2 = 3zn−2 với z1 = 1, z2 = 2 ta thu được

z2n = 2 · 3n−1 và z2n+1 = 3n.

Vì vậy

y2n+1 = 2 · 3n−1, y2n = 3n, u2n+1 = 4 · 3n−1, u2n = 2 · 3n−1, x2n+1 = 14 · 3n−1, x2n = 8 · 3n−1

và x1 = 5.

Ví dụ 2.12. Có bao nhiêu số có n chữ số từ tập các số 0; 1; 2; 3; 4 thỏa mãn hai chữ số liêntiếp hơn kém nhau 1 đơn vị.

Lời giải. Gọi Anlà số các số có n chữ số mà tận cùng là 0 hoặc 4; Bn là số các số có n chữsố mà tận cùng là 1 hoặc 2; Cn là số các số có n chữ số mà tận cùng là 2. Với n là số nguyêndương ta có:

• Mỗi số thuộc có thể tạo ra từ một số thuộc bằng cách xóa một số ở cuối nên An+1 = Bn

• Mỗi số thuộc có thể tạo ra 1 số thuộc bằng cách thêm 1 (trong trường hợp tận cùng bằng0) hoặc thêm 3 (nếu tận cùng bằng 4) và mỗi số thuộc có thể tạo ra một số thuộc bằngcách thêm 1 hoặc 3 vào cuối. Do đó: Bn+1 = An + 2Cn

• Mỗi số thuộc có thể tạo ra một số thuộc bằng cách xóa số 2 ở cuối, vì vậy Cn+1 = Bn

Chúng ta cần tính Sn = An +Bn + Cn. Công việc này khá đơn giản !

Ví dụ 2.13 (Romanian MO 1998). Gọi An là tập hợp các chữ có độ dài bằng n được thànhlập từ các chữ cái a, b, c sao cho không có hai phần tử liên tiếp cùng bằng a hoặc cùng bằng b.Gọi Bn là tập hợp các chữ có độ dài bằng n, được thành lập từ các chữ a, b, c mà không có baphần tử liên tiếp nào đôi một phân biệt. Chứng minh rằng card (Bn) = 3 · card (An)

Lời giải. Gọi Xn, Yn, Zn lần lượt là các tập con của có tận cùng bởi chữ a, b, c tương úng.

Đặt un = card (Un). Với mỗi số nguyên n ta có các hệ thức sau đây:

|An| = xn + yn + zn

xn+1 = yn + zn;

yn+1 = xn + zn

zn+1 = xn + yn + zn = |An|

Do đó |An+2| = xn+2+yn+2+zn+2 = 2xn+1+2yn+1+3zn+1 = 2 |An+1|+ |An| và A1 = 3, A2 = 7.

48

www.VNMATH.com

Page 50: [Vnmath.com] Ky Yeu Trai He Hv 2012

Gọi Sn là tập con của có tận cùng bởi chữ aa, bb, cc. Đặt Tn = Bn − Sn thì Tn là tập hợpcác số trong có tận cùng bởi ab, ac, ba, bc, ca, cb. Ta thu được các hệ thức sau:

|Bn| = sn + tn

|Bn+1| = 3sn + 2tn

sn+1 = |Bn|

Vì vậy |Bn+2| = sn+2 + tn+2 = 2 |Bn+1|+ |Bn| và B2 = 9, B3 = 21.

Thành thử ta có điều phải chứng minh.

Việc giải bài toán trên thực chất là đi giải hai bài toán nhỏ về phép đếm và cả hai bài toánấy đều giải bằng cách thiết lập các dãy truy hồi lồng nhau.

Ví dụ 2.14 (Vietnam MO 2009). Cho số nguyên dương n. Kí hiệu T là tập hợp gồm 2n sốnguyên dương đầu tiên. Hỏi có tất cả bao nhiêu tập con S của T có tính chất: trong S khôngtồn tại các số a, b mà |a− b| ∈ 1, n? (Tập rỗng được coi là tập có tính chất nêu trên).

Lời giải. Trước hết ta có bài toánCho 2 hàng điểm A1;A2;A3; ...;An ở trên và B1;B2; ...;Bn ở dưới.

Các cặp điểm (Ai;Ai−1) ; (Bi;Bi−1) ; (Ai;Bi) được nối với nhau, ngoài ra An và cũng được nốivới nhau. Tính số cách chọn ra một số điểm mà không có hai điểm nào được nối với nhau.

Gọi Sn là số cách chọn thỏa mãn điều kiện trên, nhưng có thể chứa cả Ai và Bi. Gọi xn làsố cách chọn thỏa mãn nhưng không chứa điểm nào trong 4 điểm Ai, An, Bi, Bn. Gọi yn là sốcách chọn thỏa mãn nhưng chứa đúng 1 điểm trong 4 điểm trên. Gọi zn là số cách chia thỏamãn nhưng chứa đúng 2 điểm Ai, An hoặc Bi, Bn. Gọi tn là số cách chọn thỏa mãn nhưng chứađúng 2 điểm Ai, Bn, hoặc Bi, An.Khi đó ta có Sn = xn + yn + zn + tn và số cách chọn thỏa mãn bài toán là Sn −

tn2.

Dễ dàng lập công thức truy hồi cho Sn là S0 = 1, S1 = 3, Sn+1 = 2Sn + Sn−1

Mặt khác ta có

xn = Sn−2;

yn = 2 (Sn−1 − Sn−2) ;

zn = tn−1 +1

2yn−2;

tn = zn−1 +1

2yn−2.

Từ đó suy ra

zn + tn = Sn − Sn−2 − 2 (Sn−1 − Sn−2) = 2Sn−2;

zn − tn = − (zn−1 − tn−1) ;

tn = Sn−2 + (−1)n .

49

www.VNMATH.com

Page 51: [Vnmath.com] Ky Yeu Trai He Hv 2012

Do đó Sn −1

2tn =

2Sn − Sn−2 + (−1)n−1

Cuối cùng ta thu được kết quả là(5 + 4

√2) (

1 +√

2)n−1

+(5− 4

√2) (

1−√

2)n−1

+ 2 (−1)n−1

Trở lại bài toán nếu coi điểm Ai được gắn với số n + i và điểm Bi được gắn với số i thì ta cókết quả của bài toán gốc.

50

www.VNMATH.com

Page 52: [Vnmath.com] Ky Yeu Trai He Hv 2012

3 Bài tập áp dụng

Để kết thúc bài viết này chúng tôi xin nêu ra một số bài toán đếm áp dụng phương pháp thiếtlập hệ thức truy hồi

Bài toán 3.1. Xét tất cả các chuỗi gồm n kí tự từ tập hợp 0; 1; 2; 3. Có bao nhiêu số có sốchẵn các số:

1. 0

2. 0 và 1

Bài toán 3.2. Tìm số xâu nhị phân có độ dài bằng n và không có hai số 0 liên tiếp.

Bài toán 3.3 (ĐHQG Tp.HCM TST 2009). Cho số nguyên dương n. Có bao nhiêu số chiahết cho 3, có n chữ số và các chữ số đều thuộc 3; 4; 5; 6?

Bài toán 3.4. Chia một hình vuông làm n phần bằng nhau, ta được hình vuông nhỏ; trong mỗihình vuông nhỏ kẻ các đường chéo. Hỏi trong hình đó có tất cả bao nhiêu hình vuông.

Bài toán 3.5. Cho A = 1; 2; 3; ...;n , ∀n ≥ 4. Tìm số các tập con B của A có hiệu hai phầntử bất kì khác 1 và 3. Tập rỗng và tập có 1 phần tử được coi là các tập thỏa mãn điều kiện đó.

Bài toán 3.6 (St.Petersburg City MO 1995). Trên đường tròn cho 999 điểm. Có bao nhiêucách đánh cho mỗi điểm bởi các chữ cái A,B và C sao cho trên cung nằm giữa hai điểm đượcviết cùng chữ cái, có số chẵn các chữ cái khác nhau từ hai chữ cái đó.

Bài toán 3.7 (Poland MO 1995-Thụy Sĩ 2006). Cho số nguyên dương n. tìm số tập concủa tập A = 1; 2; 3; ...; 2n sao cho không tồn tại hai phần tử x, y ∈ A thỏa mãn x+y = 2n+ 1.

Bài toán 3.8 (IMO Longlist 1992). Cho an là ước số lớn nhất của n mà không chia hết cho3. Xét dãy số s0 = 0, sn = a1 +a2 +a3 + · · ·+an. Đặt An là số các tổng sk (0 ≤ k ≤ 3n, k ∈ N0)

chia hết cho 3. Chứng minh hệ thức

An = 3n−1 + 2 · 3n2−1 · cos

(nπ6

), ∀n ∈ N

Bài toán 3.9. Tìm số số có n chữ số a1a2...an mà a1a2 + a2a3 + · · ·+ an−1an... 2.

Bài toán 3.10. Cho số nguyên dương n ≥ 2. Hỏi có thể lập được tất cả bao nhiêu dãy số(a1; a2; a3; ...; a2n) thỏa mãn các điều kiện sau

1/ a1 ∈ −1; 1 , ∀n = 1; 2; ...; 2n

2/∣∣∣∑2l

i=2k−1 ai

∣∣∣ ≥ 2, ∀l; k thỏa mãn 1 ≤ k ≤ l ≤ n.

Bài toán 3.11. Tìm số tập con của tập hợp n số nguyên dương đầu tiên sao cho mỗi tập cóđúng 3 phần tử và tổng các phần tử chia hết cho 3.

51

www.VNMATH.com

Page 53: [Vnmath.com] Ky Yeu Trai He Hv 2012

Bài toán 3.12 (USA MO 1990). Cho n là một số nguyên dương. Tìm số các số nguyên dươngmà trong biểu diễn hệ cơ số n của nó có các chữ số phân biệt và thỏa mãn tính chất: trừ chữ sốđầu tiên bên trái, mọi chữ số đều khác các chữ số nào đó bên trái là ±1.

Bài toán 3.13. Cho số nguyên dương n. Đặt S = x1y1 + x2y2 + · · ·+ xnyn, với xi, yi ∈ 0; 1.Gọi an, bn lần lượt là bộ 2n-số (x1; ...;xn; y1; ..; yn) có S lẻ và chẵn tương ứng. Chứng minh hệthức

anbn

=2n − 1

2n + 1.

Bài toán 3.14. Cho lục giác đều ABCDEF có tâm O được nối với các đỉnh của lục giác bởicác bán kính. Đếm số đường đi độ dài n xuất phát từ O và kết thúc ở O.

Bài toán 3.15 (IMO Shortlist-1996). Cho bảng ô vuông n×n, n > 1. Hỏi có bao nhiêu cáchđánh dấu các ô vuông trong mỗi hình vuông 2× 2 có đúng 3 ô vuông được đánh dấu? (hai cáchđánh dấu được coi là khác nhau nếu có một ô vuông nào đó mà trong cách này thì được đánhdấu còn trong cách kia thì không).

Bài toán 3.16. Tìm số hoán vị không có điểm cố định của tập hợp 1; 2; 3; ...;n.

Bài toán 3.17 (ĐHKHTNHà Nội TST 2010). Tìm số hoán vị (a1; a2; ...; an) của 1; 2; 3; ...;nthỏa mãn đồng thời hai điều kiện sau:

1. ai 6= i với mọi i = 1; 2; ...;n.

2. ai+1 − ai ≤ 1 với mọi i = 1; 2; ...;n− 1.

Bài toán 3.18. Tìm số các số nguyên dương N thỏa mãn các điều kiện:

1. N có 2012 chữ số;

2. Tất cả các chữ số của N là số lẻ;

3. Hiệu của hai số liên tiếp bất kì của N luôn bằng 2.

Bài toán 3.19 (Vietnam MO 1977). Trong mặt phẳng cho n đường tròn. Hai đường tròn bấtkì cắt nhau tại hai điểm và không có điểm nào nằm trên 3 đường tròn. Hỏi có bao nhiêu miềnmặt phẳng bị chia bởi các đường tròn nói trên.

Bài toán 3.20. Tìm số tập con của tập 1; 2; ...;n sao cho trong mỗi tập con chứa ít nhất haiphần tử là hai số nguyên liên tiếp.

Bài toán 3.21 (Bà Rịa -Vũng Tàu 2010). Cho số nguyên dương n. Có bao nhiêu số tự nhiêncó n chữ số mà trong mỗi số, các chữ số đều lớn hơn 1 và không có hai chữ số khác nhau cùngnhỏ hơn 7 đứng liền kề nhau.

Bài toán 3.22 (Mở rộng Vietnam MO 2003A - Tạp chí THTT). Tìm số hoán vị(a1; a2; ...; an) của tập 1; 2; 3; ...;n thỏa mãn 1 ≤ |ai − i| ≤ 2, ∀i = 1; 2; ...;n.

52

www.VNMATH.com

Page 54: [Vnmath.com] Ky Yeu Trai He Hv 2012

Bài toán 3.23. Tìm số tập con của tập hợp 1; 2; ...;n mà hai phần tử bất kì hơn kém nhau ítnhất 3 đơn vị.

Bài toán 3.24 (Canadian MO 2009). Cho hình chữ nhật kích thước 3 × n ô. Một quân xexuất phát từ ô góc trên bên trái và đi đến ô góc dưới bên trái sao cho đường đi của nó tạo thànhmột đường gấp khúc không tự cắt. Hỏi có bao nhiêu cách đi như thế?

Bài toán 3.25 (Bulgarian MO 1999). Có bao nhiêu số tự nhiên có n chữ số mà trong biểudiễn nhị phân của chúng không có ba chữ số liên tiếp bằng nhau?

Bài toán 3.26. Có bao nhiêu cách xếp 8 quân xe lên bàn cờ vua sao cho không có quân xe nàonằm trên đường chéo chính (đường chéo nối góc trên bên trái và góc dưới bên phải) và không cóquân xe nào có thể ăn các quân xe khác?

53

www.VNMATH.com

Page 55: [Vnmath.com] Ky Yeu Trai He Hv 2012

Tài liệu

[1] Nguyễn Văn Mậu (chủ biên), Chuyên đề chọn lọc Tổ hợp và Toán rời rạc, NXB Giáo Dục,2008.

[2] Nguyễn Văn Mậu (chủ biên), Toán Rời rạc và một số vấn đề liên quan, Tài liệu bồi dưỡnggiáo viên hè 2007, Trường ĐHKHTN-ĐHQG Hà Nội.

[3] Titu Andreescu-Zuming Feng, A path to combinatorics for undergraduates, Birkhauser.

[4] Iiri Herman-Radan Kucera-Jarommir Simsa, Counting and Configurations, Canadian Math-ematical Society.

[5] Titu Andreescu-Zuming Feng, 102 Combinatorial problems from the training of the USAIMO team, Birkhauser 2002.

[6] Titu Andreescu-Kiran S.Kedlaya-Paul Zeitz, Mathematical Contests 1995-1996. OlympiadProblems from Around the World with Solutions, American Mathematics Competitions 1997.

[7] Titu Andreescu-Kiran S.Kedlaya, Mathematical Contests 1996-1997. Olympiad Problemsfrom Around the World with Solutions, American Mathematics Competitions 1998.

[8] Titu Andreescu-Kiran S.Kedlaya, Mathematical Contests 1997-1998. Olympiad Problemsfrom Around the World with Solutions, American Mathematics Competitions 1999.

[9] Titu Andreescu-Zuming Feng, Mathematical Olympiad. Problems from Around the World1998-1999, Mathematical Association of America 2000.

[10] Titu Andreescu-Zuming Feng, Mathematical Olympiad. Problems from Around the World1999-2000, Mathematical Association of America 2002.

[11] Titu Andreescu-Zuming Feng-George Lee, Mathematical Olympiad. Problems from Aroundthe World 2000-2001, Mathematical Association of America 2003.

[12] Titu Andreescu-Răzvan Gelca, Putnam and Beyond, Springer.

[13] Arthur Engel, Problem-Solving Strategies, Springer.

[14] ME Kuczma- E Windischbacher, Polish and Austrian Mathematical Olympiad 1981-1995,An Australian Mathematics trust publication.

[15] Trần Nam Dũng (chủ biên), Chuyên đề toán học số 8, Trường PTNK-ĐHQG TP. Hồ ChíMinh.

[16] Le Hai Chau-Le Hai Khoi, Selected Problems of the Vietnamese Mathematical Olympiad(1962-2009), World Scientific.

[17] Dusan Diukic- Vladimir Jankovic- Ivan Matic- Nikola Petrovic, The IMO Compendium ACollection of Problems Suggested for the International Mathematical Olympiads. 1959-2004,Springer.

[18] Tạp chí Toán học và tuổi trẻ - Việt Nam, Crux - Canada, AMM - USA

54

www.VNMATH.com

Page 56: [Vnmath.com] Ky Yeu Trai He Hv 2012

55Các chuyên đề Toán Trại hè Hùng Vương năm 2012, Cao Bằng 01-04/08/2012

Tính toán một số lớp tích phân xácđịnh dạng đặc biệt

Lê Hồ Quý, Trường THPT Duy Tân, Kon Tum

Tóm tắt nội dung

Trong nhiều bài toán việc tìm nguyên hàm rất phức tạp và khó khăn thậm chíkhông tìm được nguyên hàm dưới dạng hiện. Vì vậy, nhu cầu tính các tích phân xácđịnh khi chưa tường minh nguyên hàm tương ứng là một bài toán cần được khảosát chi tiết. Trong những trường hợp đó, nếu biết dựa vào những tính chất đặc biệtcủa hàm dưới dấu tích phân và những biến đổi thích hợp, ta có thể tính được mộtsố dạng tích phân xác định.

1 Tích phân đối với hàm chẵn và lẻ

Tính chất 1.1. Nếu hàm số y = f(x) lẻ, liên tục trên [−a; a], với a > 0 thì

I =

a∫−a

f(x)dx = 0.

Chứng minh. Do f(x) liên tục trên [−a; a] nên

I =

0∫−a

f(x)dx+

a∫0

f(x)dx

Đặt x = −t thì dx = −dt. Khi đó

0∫−a

f(x)dx = −0∫a

f(−t)dt =a∫

0

f(−t)dt =a∫

0

f(−x)dx = −a∫

0

f(x)dx

(Do f(x) là hàm số lẻ nên f(−x) = −f(x)

).

Vậy

I = −a∫

0

f(x)dx+

a∫0

f(x)dx = 0.

55

www.VNMATH.com

Page 57: [Vnmath.com] Ky Yeu Trai He Hv 2012

Ví dụ 1.1. Tính tích phân

I =

12∫

− 12

(cos 4x+ sin

x

2sinx

)ln

1 + x

1− xdx.

Lời giải. Xét hàm số

f(x) =(cos 4x+ sin

x

2sinx

)ln

1 + x

1− x

Ta thấy f(x) liên tục trên[− 1

2 ;12

]và

f(−x) =[cos(−4x) + sin

−x2

sin(−x)]ln

1− x1 + x

= (cos 4x+ sinx

2sinx) ln

(1 + x

1− x)−1

= −(cos 4x+ sinx

2sinx) ln

1 + x

1− x= −f(x).

Vậy f(x) liên tục trên[− 1

2 ;12

]và là hàm số lẻ nên theo tính chất 1.1, ta có I = 0.

Tính chất 1.2. Nếu f(x) là hàm số chẵn, liên tục trên [−a; a], với a > 0 thì

I =

a∫−a

f(x)dx = 2

a∫0

f(x)dx.

Chứng minh. Chứng minh tương tự như tính chất 1.1, chú ý f(−x) = f(x).

Ví dụ 1.2 (Đề thi tuyển sinh vào ĐH Lâm nghiệp - 1999). Tính tích phân

I =

1∫−1

x4 + sinx

x2 + 1dx.

Lời giải. Ta có

I =

1∫−1

x4

x2 + 1dx+

1∫−1

sinx

x2 + 1dx = I1 + I2.

Dễ thấysinx

x2 + 1là hàm số lẻ, liên tục trên [−1; 1] theo tính chất 1.1 ta có I2 = 0;

cònx4

x2 + 1là hàm số chẵn, liên tục trên [−1; 1] nên theo tính chất 1.2, ta có

I1 =2

1∫0

x4

x2 + 1dx = 2

1∫0

x4 − 1 + 1

x2 + 1dx = 2

[ 1∫0

(x2 − 1)dx+

1∫0

dx

x2 + 1

]=π

2− 4

3.

Vậy I =π

2− 4

3.

56

www.VNMATH.com

Page 58: [Vnmath.com] Ky Yeu Trai He Hv 2012

Tính chất 1.3. Nếu f(x) là hàm số chẵn, liên tục trên D ⊂ R thì với ∀a ∈ D taluôn có

I =a∫

−a

f(x)

bx + 1dx =

a∫0

f(x)dx, với a > 0 và 0 < b 6= 1.

Chứng minh. Ta có

I =

0∫−a

f(x)

bx + 1dx+

a∫0

f(x)

bx + 1dx

Đặt x = −t thì dx = −dt. Khi đó0∫

−a

f(x)

bx + 1dx = −

0∫a

f(−t)b−t + 1

dt =

a∫0

f(−t)bt

bt + 1dt =

a∫0

f(−x)bx

bx + 1dx =

a∫0

f(x)bx

bx + 1dx.

Vậy

I =

a∫0

f(x)bx

bx + 1dx+

a∫0

f(x)

bx + 1=

a∫0

(bx + 1)f(x)

bx + 1dx =

a∫0

f(x)dx.

Ví dụ 1.3. Tính tích phân

I =

2∫−2

x ln(x+√1 + x2

)(2011x + 1

)√1 + x2

dx

Lời giải. Xét hàm số

f(x) =x ln

(x+√1 + x2

)√1 + x2

.

Dễ thấy f(x) là hàm số chẵn và liên tục trên [−2; 2]. Từ tính chất 1.3 , ta được

I =

2∫0

x ln(x+√1 + x2

)√1 + x2

dx.

Để tính tích phân I, ta sử sụng phương pháp tích phân từng phần.

Đặt

u = ln(x+√1 + x2

)dv =

x√1 + x2

dx⇒

du =dx√1 + x2

v =√1 + x2

Vậy

I =√

1 + x2 ln(x+

√1 + x2

)∣∣20−

2∫0

dx =√5 ln

(2 +√5)− 2.

Nhận xét 1.1. Từ các tính chất riêng lẻ 1.1, 1.2 và 1.3, dẫn đến một tính chấtchung sau đây

Tính chất 1.4. Nếu f(x) là hàm liên tục trên [−a; a], với a > 0 thì

I =

a∫−a

f(x)dx =

a∫0

(f(x) + f(−x)

)dx

57

www.VNMATH.com

Page 59: [Vnmath.com] Ky Yeu Trai He Hv 2012

Chứng minh. Do f(x) liên tục trên [−a; a] nên

I =

0∫−a

f(x)dx+

a∫0

f(x)dx

Đặt x = −t thì dx = −dt. Khi đó0∫

−a

f(x)dx =

0∫a

f(−t)(−dt) =a∫

0

f(−t)dt =a∫

0

f(−x)dx

Vậy

I =

a∫0

f(x)dx+

a∫0

f(−x)dx =

a∫0

(f(x) + f(−x)

)dx.

Ví dụ 1.4 (Đề thi Olympic Sinh viên Toàn quốc - 2011). Tính tích phân1∫

−1

dx

1 + x+ x2 +√1 + 3x2 + x4

.

Lời giải. Xét f(x) =1

1 + x+ x2 +√1 + 3x2 + x4

, với x ∈ [−1, 1].

Ta nhận thấy f(x) liên tục trên đoạn [−1; 1] và f(x) + f(−x) = 1

1 + x2.

Từ đó, sử dụng tính chất 1.4, ta có

I =

1∫−1

f(x)dx =

1∫0

(f(x) + f(−x)

)dx =

1∫0

dx

1 + x2=π

4.

2 Tích phân đối với hàm tuần hoàn

Trong mục này ta chỉ xét các hàm tuần hoàn cộng tính. Đối với các hàm số tuầnhoàn nhân tính cần chuyển đổi qua hàm tuần hoàn cộng tính bằng phép lôgarit hóacác biểu thức tương ứng của biến số.

Ta nhắc lại định nghĩa hàm tuần hoàn.

Định nghĩa 2.1. Hàm số y = f(x) được gọi là hàm số tuần hoàn nếu có một sốT > 0 sao cho với mọi x thuộc miền xác định Df của hàm số, ta luôn có

1) x± T cũng thuộc miền xác định của hàm số,2) f(x+ T ) = f(x), ∀x ∈ Df .Số T (T > 0) được gọi là chu kì của hàm tuần hoàn. Chu kì nhỏ nhất (nếu tồn

tại) được gọi là chu kì cơ sở của hàm số đã cho.

Tính chất 2.1. Nếu hàm số f(x) tuần hoàn chu kì T , xác định và liên tục trên Rthì

a+T∫a

f(x)dx =

T∫0

f(x)dx =

T2∫

−T2

f(x)dx, ∀a ∈ R

58

www.VNMATH.com

Page 60: [Vnmath.com] Ky Yeu Trai He Hv 2012

Chứng minh. Đặt

I1 =

a+T∫a

f(x)dx, I2 =

T∫0

f(x)dx, I3 =

T2∫

−T2

f(x)dx

Ta có

I1 =

0∫a

f(x)dx+

T∫0

f(x)dx+

a+T∫T

f(x)dx.

Đổi biến x = t + T đối với tích phâna+T∫T

f(x)dx được I1 = I2. Chọn a = −T2 ta

được I1 = I3.

Ví dụ 2.1. Tính tích phân

I =

5π4∫

π

sin 2x

cos4 x+ sin4 xdx.

Lời giải. Dễ thấy f(x) =sin 2x

cos4 x+ sin4 x=

sin 2x

1− 12 sin

2 2xlà hàm tuần hoàn với chu

kì T = π, do đó theo tính chất 2.1, ta có

I =

π4∫

0

sin 2x

cos4 x+ sin4 xdx = 2

π4∫

0

sinx cosx

cos4 x+ sin4 xdx = 2

π4∫

0

tanxdx

cos2 x(1 + tan4 x)

Ta tính

I1 =

π4∫

0

tanxdx

cos2 x(1 + tan4 x)

Đặt t = tanx, ta có dt =dx

cos2 x.

Đổi cận: Khi x = 0⇒ t = 0; khi x = π4 ⇒ t = 1. Vậy nên

I1 =

1∫0

tdt

1 + t4=

1

2

1∫0

d(t2)

1 + t4=π

8.

Do đóI = 2I1 =

π

4.

Ví dụ 2.2 (Đề thi Olympic Sinh viên Toàn quốc - 2007). Tính tích phân

I =

2π∫0

ln(sinx+

√1 + sin2 x

)dx.

Lời giải. Nhận xét rằng f(x) = ln(sinx +

√1 + sin2 x

)là hàm liên tục, tuần

hoàn với chu kì T = 2π. Từ tính chất 2.1 ta được I =π∫

−πf(x)dx.

Mặt khác, do f(x) là hàm số lẻ nên theo tính chất 1.1 ta được I = 0.

59

www.VNMATH.com

Page 61: [Vnmath.com] Ky Yeu Trai He Hv 2012

3 Tích phân một số dạng với đặc trưng hàm

đặc biệt

Tính chất 3.1. Nếu hàm số f(x) liên tục trên [0; 1] thì

π2∫

0

f(sinx)dx =

π2∫

0

f(cosx)dx.

Chứng minh. Đặt t = π2 − x thì x = π

2 − t và dx = −dt. Khi đó

π2∫

0

f(sinx)dx = −0∫

π2

f[sin(π2− t)]dt =

π2∫

0

f(cos t)dt =

π2∫

0

f(cosx)dx.

Ví dụ 3.1. Tính tích phân

I =

π2∫

0

cosn x

cosn x+ sinn xdx (n ∈ N∗).

Lời giải. Theo tính chất 3.1, ta có

I =

π2∫

0

sinn x

sinn x+ cosn xdx

⇒ 2I =

π2∫

0

cosn x

cosn x+ sinn xdx+

π2∫

0

sinn x

sinn x+ cosn xdx =

π2∫

0

dx =π

2.

Vậy I =π

4.

Tính chất 3.2. Nếu hàm số f(x) liên tục trên đoạn [a; b] và thỏa mãn điều kiệnf(x) = f(a+ b− x) với mọi x ∈ [a; b], thì ta luôn có

1) I =b∫af(x)dx = 2

a+b2∫af(x)dx.

2) J =b∫axf(x)dx = a+b

2

b∫af(x)dx.

Đặc biệt: J1 =π∫0

xf(sinx)dx = π2

π∫0

f(sinx)dx.

Chứng minh. Ta có

b∫a

f(x)dx =

a+b2∫a

f(x)dx+

b∫a+b2

f(x)dx.

60

www.VNMATH.com

Page 62: [Vnmath.com] Ky Yeu Trai He Hv 2012

Đối với tích phân thứ hai, đặt x = a+ b− t, ta được

b∫a+b2

f(x)dx =−a∫

a+b2

f(a+ b− t)dt =

a+b2∫a

f(a+ b− t)dt =

a+b2∫a

f(t)dt =

a+b2∫a

f(x)dx

Do vậy, tính chất 3.2 1) được chứng minh.

Bằng cách đổi biến tương tự, đặt t = a+ b− x thì dt = −dx và

b∫a

xf(x)dx = −a∫b

(a+ b− t)f(a+ b− t)dt =b∫a

(a+ b− t)f(t)dt

= (a+ b)

b∫a

f(t)dt−b∫a

tf(t)dt = (a+ b)

b∫a

f(x)dx−b∫a

xf(x)dx

và tính chất 3.2 2) được chứng minh. Cho a = 0, b = π ta có J1.

Ví dụ 3.2. Tính tích phân

I =

2π∫0

dx

2 + cosx.

Lời giải. Ta có1

2 + cosx=

1

2 + cos(2π − x)

Từ tính chất 3.2, ta suy ra

I =

2π∫0

dx

2 + cosx= 2

π∫0

dx

2 + cosx= 2( π

2∫0

dx

2 + cosx+

π∫π2

dx

2 + cosx

)

Sử dụng công thức cosx =1− tan2 x21 + tan2 x2

, ta có

2 + cosx =1 + 3 cot2 x21 + cot2 x2

.

Ta tính

I1 =

π2∫

0

dx

2 + cosx.

Đặt t = tanx

2(0 ≤ t ≤ 1), ta có

I1 = 2

1∫0

dt

t2 + 3=π√3

9.

61

www.VNMATH.com

Page 63: [Vnmath.com] Ky Yeu Trai He Hv 2012

Ta tính

I2 =

π∫π

2

dx

2 + cosx.

Đặt u = cotx

2(0 ≤ u ≤ 1), ta được

I2 = 2

1∫0

du

1 + 3u2=

2

3

1∫0

du

u2 + 13

=2π√3

9.

Vậy

I =π√3

9+

2π√3

9=

2π√3

3.

Ví dụ 3.3. Tính tích phân

I =

π∫0

x sinxdx

1 + sin2 x.

Lời giải. Theo tính chất 3.2, ta có

I =π

2

π∫0

sinxdx

1 + sin2 x=π

2

π∫0

−d(cosx)2− cos2 x

= − π

4√2ln√2 + cosx√2− cosx

∣∣π0=

= − π√2ln (√2− 1).

Bài tập1. Tính các tích phân sau

a)π2∫

−π2

cosx. ln (x+√x2 + 1)dx(Học viện Kỹ thuật Mật mã - 1999);

b)2π∫0

sin(2002x+ sinx)dx(Đề thi Olympic Sinh viên toàn quốc-2002);

c)1∫

−1

x6+tanxx2+1

dx;

d)π4∫

−π4

sin6x+cos6x6x+1 dx(ĐH Mỏ - Địa chất - 1999);

e)4π∫0

sin73x.cos85x1+cos107x

dx;

f)4π∫0

(1

cos2(sinx)− tan2(cosx)

)dx;

g)π∫0

x sinxcos2xdx(Học viện Ngân hàng - 1998);

h)π∫0

x sinx1+cos2x

dx.

2. Cho f(x)là hàm số liên tục trên R thỏa mãnf(x) + f(−x) =√2− 2 cos 2x. Tính

3π2∫

− 3π2

f(x)dx(ĐHSP Hà Nội 2 - 1998).

62

www.VNMATH.com

Page 64: [Vnmath.com] Ky Yeu Trai He Hv 2012

Tài liệu

[1] Nguyễn Thủy Thanh (2010), Phương pháp giải các dạng toán cơ bản, Tập 2,Giải tích, NXB Giáo Dục.

[2] Nguyễn Văn Mậu (2004), Một số vấn đề chọn lọc về tích phân, NXB GiáoDục.

[3] Tô Văn Ban (2005), Giải tích, Những bài tập nâng cao, NXB Giáo Dục.

[4] Nguyễn Văn Mậu (2007), Nội suy và áp dụng, NXB Giáo Dục.

[5] Nguyễn Văn Mậu, Nguyễn Thuỷ Thanh (2002), Giới hạn dãy số và hàm số,NXB Giáo dục.

[6] Nguyễn Văn Mậu, Đặng Huy Ruận, Nguyễn Thuỷ Thanh (2000), Phép tính viphân và tích phân hàm một biến, NXB Đại học Quốc gia Hà Nội.

[7] Nguyễn Văn Mậu (2005), Bất đẳng thức, Định lý và áp dụng,NXB Giáo Dục.

[8] Trần Phương (2006), Tuyển tập các chuyên đề và kỹ thuật tính Tích phân,NXB Tri Thức.

[9] Trịnh Tuân (2008), "Sử dụng một số kết đẹp để tính tích phân", Tạp chí Toánhọc và Tuổi trẻ (số 367), tr. 12-13,30.

[10] Đặng Thanh Hải, Trần Tuyết Thanh (2010), "Đổi biến trong tích phân hàmlượng giác", Tạp chí Toán học và Tuổi trẻ (số 391), tr. 8-9,12.

[11] Lê Hồ Quý (2011), "Cách tính tích phân một số hàm số vô tỉ", Tạp chí Toánhọc và Tuổi trẻ (số 403), tr. 5-7.

[12] James Stewart (2008), Calculus Early Transcendentals, 6e, Thomson.

63

www.VNMATH.com

Page 65: [Vnmath.com] Ky Yeu Trai He Hv 2012

Một số phương pháp giải Hệ phương trình cho học sinh giỏi Nguyễn Anh Tuấn

64

M Ộ T SỐ PH ƯƠN G P H ÁP GIẢ I H Ệ P H ƯƠN G TR Ì NH

Nguy ễn Anh Tuấn, Trường THPT Chuyên Bắc Giang

Danh mục các từ viết tắt 1. Vt: Vế trái của phương trình.

2Vt : Bình phương của vế trái phương trình. 2. Vp: Vế phải của phương trình.

2Vp : Bình phương của vế phải phương trình.

3. Vt (1) : Vế trái của phương trình (1) .

4. Vp (1) : Vế phải của phương trình (1) .

5. Đk, đk: Điều kiện.

6. BĐT: Bất đẳng thức. HSG, HSG: Học sinh giỏi.

7. VMO, VMO: Thi chọn học sinh giỏi Toán lớp 12 THPT của Việt Nam.

IMO: Thi Olympic Toán học quốc tế. 8. HD: Hướng dẫn.

9. Quy ước: Nghiệm của tất cả các Ví dụ và Bài toán trong Chuyên đề được xét trong tập số thực R.

www.VNMATH.com

Page 66: [Vnmath.com] Ky Yeu Trai He Hv 2012

Một số phương pháp giải Hệ phương trình cho học sinh giỏi Nguyễn Anh Tuấn

65

Lời mở đầu

Toán học có một vẻ đẹp lôi cuốn và quyến rũ, ai đã đam mê thì mãi mãi đam mê… Trong vẻ đẹp đầy huyền bí đó thì các bài toán liên quan đến Hệ phương trình – có nét đẹp thật sự xao xuyến và quyến rũ! Có lẽ vì lý do đó mà trong các kì thi chọn HSG Quốc gia lớp 12 THPT (VMO)

của chúng ta, bài toán liên quan đến Hệ phương trình thường có mặt để thách thức các nhà Toán học tương lai với dung nhan "muôn hình, muôn vẻ". Bài toán giải Hệ phương trình cũng thường xuyên xuất hiện trong các kì thi của học sinh THCS và THPT như thi chọn HSG cấp tỉnh, thi chọn HSG cấp thành phố, thi chọn HSG cấp

huyện, thi chọn HSG cấp trường, thi Đại học, thi tuyển sinh vào lớp 10 THPT chuyên, hay bài toán đó còn có trong đề thi chọn giáo viên dạy giỏi cấp tỉnh ở vòng thi lý thuyết, … Chuyên đề: “Một số phương pháp giải Hệ phương trình cho học sinh giỏi”, tôi

viết với mong muốn phần nào giúp các Thầy cô giáo dạy Toán, các em học sinh phổ thông cũng như các em học sinh trong các đội tuyển thi chọn học sinh giỏi Toán có thể tìm thấy nhiều điều bổ ích và nhiều điều thú vị đối với dạng toán này. Trong Chuyên đề có cả những bài với cấp độ giải trí cho học sinh giỏi (rèn luyện phản xạ nhanh).

Đối với việc giải Hệ phương trình thì hầu hết các phương pháp giải, các phương pháp biến đổi hay đều có trong cuốn Chuyên đề này. Cách phân tích để nhận dạng một Hệ phương trình và chọn lựa phương pháp giải thích hợp là khó và đa dạng. Để có khả năng này chúng ta phải tìm cách giải và tự sáng tác nhiều Hệ phương trình để tự rút ra

những nhận xét, kinh nghiệm và hay hơn nữa là một vài thuật giải toán, cũng như lưu ý rằng một bài toán có thể có nhiều cách giải khác nhau. Tôi viết Chuyên đề này chỉ với những Ví dụ và Bài tập về giải Hệ phương trình cho học sinh giỏi (không chứa tham số). Tôi hy vọng rằng Chuyên đề sẽ để lại trong

lòng Thầy cô và các em học sinh một ấn tượng tốt đẹp. Với mỗi Ví dụ trong từng Phương pháp giải, người đọc có thể tự sáng tác cho mình những bài toán với những con số mà mình yêu thích. Tuy nhiên, Chuyên đề chắc sẽ không thể tránh khỏi những điều không mong

muốn. Tôi rất mong nhận được sự động viên và những ý kiến đóng góp chân thành của Quý Thầy cô và các em học sinh để Chuyên đề tiếp tục được hoàn thiện hơn.

Tôi xin chân thành cảm ơn!

Bắc Giang, ngày 25 tháng 4 năm 2012

www.VNMATH.com

Page 67: [Vnmath.com] Ky Yeu Trai He Hv 2012

Một số phương pháp giải Hệ phương trình cho học sinh giỏi Nguyễn Anh Tuấn

66

Chương 1

MỘT SỐ PHƯƠNG PHÁP GIẢI HỆ PHƯƠNG TRÌNH

I.1. Ví dụ mở đầu

1. Ví dụ 1 (2010 – VMO)

Giải hệ phương trình 4 4

3 3 2 2

x y 240

x 2y 3(x 4y ) 4(x 8y)

(1)

(2)

HD: Cách 1 Lấy vế với vế của (1) trừ vế với vế của 4 x (2) và lấy vế với vế của (1) trừ vế với vế

của 8 x (2)), ta được hệ phương trình đã cho tương đương với 4 3 2 4 3 2

4 3 2 4 3 2

x 4x 12x 16x 16 y 8y 48y 128y 256

x 8x 24x 32x y 16y 96y 256y 240

2 2 2 2

2 2 2 2

(x 2x 4) (y 4y 16)

(x 4x 4) (y 8y 16)

2 2

2 2

x 2x 4 y 4y 16

x 4x 4 y 8y 16

(do các biểu thức ở các vế đều dương)

2

x 2y

3y 12

x 4

y 2

x 4

y 2

(thỏa mãn).

Vậy hệ phương trình đã cho có hai nghiệm là (x; y) (4;2), ( 4; 2) .

Cách 2

Lấy vế với vế của (1) trừ vế với vế của 8 x (2), ta được 4 3 2 4 3 2x 8x 24x 32x 16 y 16y 96y 256y 256

4 4(x 2) (y 4)

x 2 y 4

x 2 4 y

x y 2

x 6 y

.

Thay vào phương trình (1) rồi biến đổi tiếp, ta được hệ phương trình đã cho có hai

nghiệm là (x; y) (4;2), ( 4; 2) .

Cách 3

Đặt y = 2t, thay vào hệ phương trình và viết lại hệ dưới dạng

www.VNMATH.com

Page 68: [Vnmath.com] Ky Yeu Trai He Hv 2012

Một số phương pháp giải Hệ phương trình cho học sinh giỏi Nguyễn Anh Tuấn

67

4 4

3 2 3 2

x 16 16(t 16)

x 3x 4x 16(t 3t 4t)

4 3 2 4 3 2(x 16)(t 3t 4t) (t 16)(x 3x 4x)

Dễ thấy, nếu (x;t) là nghiệm của hệ thì xt 0 nên ta chia hai vế của phương trình trên

cho 2 2x t thì được

2 22 2

16 4 16 4(x )(t 3 ) (t )(x 3 )

x t t x (1')

Đặt 4

u xx

và 4

v tt

thì ta có phương trình 2 2(u 8)(v 3) (v 8)(u 3)

2 2 2 2u v v u 3(u v ) 8(u v) 0

(u v)(uv 3(u v) 8) 0 (2').

Từ (1') suy ra x và t cùng dấu.

Do đó áp dụng bất đẳng thức Cauchy, ta dễ dàng suy ra u 4, v 4

u 4, v 4

u 3 1, v 3 1

u 3 7, v 3 7

.

Suy ra uv 3(u v) 8 (u 3)(v 3) 1 0 . Dấu đẳng thức xảy ra khi u = v = 4.

Từ lý luận trên và từ (2') ta suy ra u = v, từ đó suy ra x t

4x

t

.

Nếu x = t thì (1') trở thành 4 4t 16 16(t 16) (vô nghiệm).

Nếu 4

xt

thì (1') trở thành 4 8 44

25616 16(t 16) t 15t 16 0

t

4 4(t 1)(t 16) 0

t 1 .

Từ đó ta được hệ phương trình đã cho có hai nghiệm là (x; y) (4;2), ( 4; 2) .

2. Ví dụ 2 (Hội thi Khu vực Duyên Hải & Đồng Bằng Bắc Bộ năm 2012)

2.1 Đề Toán dành cho học sinh Lớp 10

Giải hệ phương trình 3 3 2

2 2

x y 3y 9

x y x 4y

(1)

(2)

HD: Cách 1 (Theo đáp án) Hệ phương trình tương đương

3 3 2

2 2

x y 3y 9

3(x y ) 3(x 4y)

3 3 2 2 2x y 3y 3(x y ) 9 3(x 4y)

3 2 3 2x 3x 3x 1 y 6y 12y 8

www.VNMATH.com

Page 69: [Vnmath.com] Ky Yeu Trai He Hv 2012

Một số phương pháp giải Hệ phương trình cho học sinh giỏi Nguyễn Anh Tuấn

68

3 3(x 1) (y 2) x 1 y 2 x y 3

Thế vào phương trình (2), ta được 22y 9y 6 0

9 33y

4

hoặc

9 33y

4

.

Với 9 33

y4

thì

9 33 3 33x 3

4 4

.

Với 9 33

y4

thì

9 33 3 33x 3

4 4

.

Vậy phương trình có hai nghiệm

3 33 9 33x;y ;

4 4

, 3 33 9 33

;4 4

.

Cách 2

Hệ tương đương với 2 2 2

2 2

(x y)(x xy y ) 3y 9

x y xy x 4y xy

2(x y)(x 4y xy) 3y 9 2(x y) 3xy xy(x y) 9 0 .

Đặt x y z

xy t

, coi phương trình trên là phương trình bậc hai ẩn z, có 2(t 6)

Từ đó suy ra z = 3 hoặc z = t - 3.

Cách 3

Hệ tương đương với 2 2 2

2 2

(x y)(x xy y ) 3y 9

x y xy x 4y xy

2(x y)(x 4y xy) 3y 9

(x y 3)(x y xy 3) 0

x y 3

x(y 1) y 3

.

Lý luận để được y 3

xy 1

dẫn đến hệ không có nghiệm trong trường hợp này.

Cách 4

Ta thấy (2) tương đương với 2 2(x y )(x y) (x 4y)(x y) (do x = y không là nghiệm của hệ)

3 3 2 2 2 2x y x 4y 5xy xy x y , thay vào (1) ta được

www.VNMATH.com

Page 70: [Vnmath.com] Ky Yeu Trai He Hv 2012

Một số phương pháp giải Hệ phương trình cho học sinh giỏi Nguyễn Anh Tuấn

69

2 2 2 2x y 5xy xy x y 9 2 2 2(y 1)x (y 5y)x y 9 0 .

Nếu y = -1 thì x = 2, không thỏa mãn.

Xét y 1 , phương trình được coi là ẩn x có 2 2(y 3y 6)

Từ đó suy ra x y 3

x(y 1) y 3

.

Nhận xét: Tất cả những cách giải đưa về giải phương trình bậc hai một ẩn, khi

tính ra nháp là bình phương của một biểu thức thì trong lời giải ta biến đổi phương trình về phương trình tích ngay.

2.2 Đề Toán dành cho học sinh Lớp 11

Giải hệ phương trình 2 2

2 2

(x 1)(y 6) y(x 1)

(y 1)(x 6) x(y 1)

.

HD: Cách 1 (Theo đáp án)

Cộng vế với vế của hai phương trình trong hệ và sau khi rút gọn, ta được 2 2

5 5 1x y

2 2 2

(1).

Trừ vế với vế của phương trình thứ hai cho phương trình thứ nhất và sau khi phân tích,

ta được

x y x y 2xy 7 0

Nếu 0x y , thay x y vào một trong hai phương trình trong hệ ta giải được

x = y = 2 và x = y= 3.

Nếu 1 1 15

x y 2xy 7 0 x y2 2 4

(2).

Đặt 5

a x2

,5

b y2

từ (1) và (2) ta có hệ phương trình

2 2 2 21 1a b a b

2 215 1

a 2 b 2 2ab 4a 4b4 2

Trừ vế với vế của hai phương trình trong hệ ta có

2a b 4 a b 1 (3).

Cộng vế với vế của hai phương trình trong hệ ta có

www.VNMATH.com

Page 71: [Vnmath.com] Ky Yeu Trai He Hv 2012

Một số phương pháp giải Hệ phương trình cho học sinh giỏi Nguyễn Anh Tuấn

70

2 a b 0a b 4 a b 0

a b 4

.

Dễ thấy nếu a b 4 thì mâu thuẫn với (3) do đó a b 0 ,

thay a b 0 vào (3) ta có a – b = ±1.

Giải hệ a b 0

a b 1

, tìm được (a;b) = 1 1 1 1

; ; ;2 2 2 2

.

Vậy hệ phương trình ban đầu có các nghiệm (x;y) = (2;2), (3;3), (2;3), (3;2).

Cách 2

Từ hệ suy ra 6(x y) xy(x y) (x y)(x y) xy(x y) (x y)

x y

2xy x y 7

.

Nếu x = y thì xét đơn giản như trên. Nếu 2xy x y 7 .

Xét 1

x2

thì 1

y y 72

(loại).

Do đó 1

x2

, suy ra x 7

y2x 1

Khi đó 2

2 2 2x 7 x 7(y 1)(x 6) x(y 1) 1 (x 6) x 1

2x 1 2x 1

2 2

2

(8 x )(x 6) 5x(x 2x 10)

2x 1 (2x 1)

4 3 2... x 6x 15x 26x 24 0 2(x 2)(x 3)(x x 4) 0

Cách 3

Hệ tương đương với 2

(x y)(x y 2xy 7) 0

(x y) 5(x y) 2xy 12 0

(1)

(2)

Nếu x y 2xy 7 0 thì thay vào (2), ta được

2 x y 5(x y) 6(x y) 5 0

x y 1

.

Cách 4

Nếu x y 2xy 7 thì cộng vế với vế của (1) và (2), thay vào ta suy ra

2 2 xy 6x y 10xy 24 0

xy 4

www.VNMATH.com

Page 72: [Vnmath.com] Ky Yeu Trai He Hv 2012

Một số phương pháp giải Hệ phương trình cho học sinh giỏi Nguyễn Anh Tuấn

71

x y 5

x y 1

.

Cách 5

Nếu x y 7 2xy thì cộng vế với vế của (1) và (2), thay vào ta suy ra 2(x y 3) 4 .

Nhận xét Qua các Ví dụ trên ta thấy rằng có thể có nhiều cách để giải một Hệ phương

trình nào đó. Mọi phương pháp đều chung một mục đích, đó là tìm cách biến đổi Hệ phương trình đã cho về Hệ phương trình hoặc Phương trình mà ta đã biết cách giải.

Sau đây ta sẽ xét một số Phương pháp giải cụ thể.

www.VNMATH.com

Page 73: [Vnmath.com] Ky Yeu Trai He Hv 2012

Một số phương pháp giải Hệ phương trình cho học sinh giỏi Nguyễn Anh Tuấn

72

I.2. Phương pháp biến đổi tương đương I.2.1. Một số lưu ý Phương pháp này chủ yếu sử dụng các kỹ năng biến đổi đồng nhất, đặc biệt là

kỹ năng phân tích nhằm đưa một phương trình trong hệ về dạng đơn giản (có thể tính x theo y hoặc ngược lại) rồi thế vào phương trình còn lại. Cũng có thể Hệ phương trình

có nhiều ẩn, khi phân tích sẽ tính được ẩn nọ theo ẩn kia để thế lại. 1. Trong hệ có một phương trình bậc nhất với ẩn x hoặc y, khi đó ta tính y theo

x hoặc x theo y. 2. Một phương trình trong hệ có thể đưa về dạng tích của các phương trình bậc

nhất hai ẩn. Đôi khi ta đặt ẩn phụ để cách trình bày gọn và tường minh hơn chứ đó không

phải là bản chất của lời giải. Phương pháp cộng, thế coi như là Phương pháp biến đổi tương đương.

I.2.2. Một số ví dụ

1. Ví dụ 1

Giải hệ phương trình 2 2

2

x (y 1)(x y 1) 3x 4x 1

xy x 1 x

(1)

(2)

HD: Ta thấy x = 0 không thỏa mãn (2).

Nếu x 0 thì từ (2) suy ra 2x 1

y 1x

, thay vào (1) ta được

2 22 2x 1 x 1

x x 3x 4x 1x x

2 2(x 1)(2x 1) (x 1)(3x 1)

3(x 1)(2x 2x 4x) 0

x 1

x 2

(do x 0 ).

Từ đó hệ có hai nghiệm là (x;y) = (1;-1), 5

2;2

.

2. Ví dụ 2

Giải hệ phương trình 2 2xy x y x 2y

x 2y y x 1 2x 2y

(1)

(2)

HD: Đk x 1; y 0 .

Biến đổi (1) (x y)(x 2y 1) 0

x 2y 1 0 (do từ điều kiện ta được x + y > 0).

x 2y 1 , thay vào (2) và biến đổi ta được

www.VNMATH.com

Page 74: [Vnmath.com] Ky Yeu Trai He Hv 2012

Một số phương pháp giải Hệ phương trình cho học sinh giỏi Nguyễn Anh Tuấn

73

(y 1)( 2y 2) 0 y 2 (do y 0 ).

Vậy nghiệm của hệ là (x;y) = (5;2).

3. Ví dụ 3

Giải hệ phương trình 2

2 2

y (5x 4)(4 x)

y 5x 4xy 16x 8y 16 0

(1)

(2)

HD: Biến đổi (2) thành (y x 4)(y 5x 4) 0 , thế y = 4 – x và y = 5x + 4

vào (1) ta sẽ có nghiệm của hệ.

Nghiệm của hệ là (x;y) = (0;4), (4;0), 4

;05

.

Nhận xét: Hai Ví dụ 2 và Ví dụ 3, ta có thể làm ra nháp coi phương trình đó là phương trình bậc hai ẩn x (Ví dụ 2), hoặc ẩn y (Ví dụ 3) để tính ẩn nọ theo ẩn kia, làm

vào bài thì phân tích ra nhân tử.

4. Ví dụ 4

Giải hệ phương trình 2 2

2

2xyx y 1

x y

x y x y

(1)

(2)

HD: Đk x + y > 0. Phương trình (1) tương đương với

2 2 2 2 2

2 2 (x y) (x y ) x yx y 1 0 (x y 1) 1 0

x y x y

x y 1 0 (do biểu thức còn lại dương).

Thế y = 1 - x vào (2), ta được hệ có nghiệm là (x;y) = (1;0), (-2;3).

5. Ví dụ 5

Giải hệ phương trình 3 3

2 2

x y 91

4x 3y 16x 9y

(1)

(2)

HD: Ta có 2 2(2) 12x 9y 48x 27y (3).

Từ (1) và (3), ta được 3 2 3 2(x 12x 48x 64) (y 9y 27y 27) 0

3 3(x 4) (y 3) 0

x 4 y 3

x 7 y .

Thay vào (2), ta được 2y 7y 12 0 .

Vậy hệ có hai nghiệm là (x;y) = (4;3), (3;4).

6. Ví dụ 6

Giải hệ phương trình 3 2

3 2

x xy 2000y 0

y yx 500x=0

.

www.VNMATH.com

Page 75: [Vnmath.com] Ky Yeu Trai He Hv 2012

Một số phương pháp giải Hệ phương trình cho học sinh giỏi Nguyễn Anh Tuấn

74

HD: Ta có hệ trên tương đương với 2 2

2 2

x(x y ) 2000y

y(x y )=500x

(1)

(2)

Nếu x = 0 thì y = 0 và (x; y) (0;0) là nghiệm của hệ.

Nếu x 0 thì từ (1) và (2) suy ra 2 2

y 02000y( y) 500x

x x 4y

(3)

Ta có 2 2 2x y 3y 0 và (1) suy ra x, y trái dấu, kết hợp với (3) ta được x 2y 0 .

Thay vào (1), ta được 23y 1000 .

Từ đó ta sẽ được hệ có ba nghiệm 20 30 10 30 20 30 10 30

(x; y) (0;0),( ; ), ( ; )3 3 3 3

.

Nhận xét: Có thể dễ mắc các sai lầm như nhân hai vế phương trình với x hoặc y

mà chưa biện luận trường hợp x = 0, y = 0, từ 2 1000y

3 chỉ lấy nghiệm

1000y

3 ,

viết nghiệm kiểu 20 30 10 30

x ; y3 3

.

7. Ví dụ 7

Giải hệ phương trình 3 3

2 2

13x y

x y

x y 1

.

HD: Đk x y 0 .

Với đk đó, ta có hệ 3 3

2 2

(3x y )(x y) 1

x y 1

3 3 2 2 2(3x y )(x y) (x y )

4 3 3 4 4 2 2 43x 3x y xy y x 2x y y

4 3 2 2 3 42x 3x y 2x y xy 2y 0

2 2(x y)(x 2y)(2x xy y ) 0

2 2

x y 0

x 2 0

2x xy y 0

(1)

(2)

(3)

Xét (1) và thay vào hệ, ta được 2 2 2 2

(x; y) ( ; ), ( ; )2 2 2 2

.

Xét (2) và thay vào hệ, ta được 2 5 5 2 5 5

(x; y) ( ; ), ( ; )5 5 5 5

.

Xét (3) 2 2

2y 3yx x 0 x y 0

2 2

(loại).

Vậy hệ đã cho có bốn nghiệm trên.

www.VNMATH.com

Page 76: [Vnmath.com] Ky Yeu Trai He Hv 2012

Một số phương pháp giải Hệ phương trình cho học sinh giỏi Nguyễn Anh Tuấn

75

8. Ví dụ 8

Giải hệ phương trình 3

4 2x 2y

x y

2x y 3

(1)

(2)

HD: Đk x 0, y 0 .

Với đk đó (1) x 2y2

(x 2y) 1 0xy 2xy

.

Nếu x = 2y thì từ (2), ta được 3 2y 4y 3 0 (y 1)(y y 3) 0

2

y 1 0

y y 3 0

y 1

x 21 13

x 1 13y2

.

Nếu xy = -2 thì 2

xy

, thay vào (2) ta được

3 44y 3 y 3y 4 0

y

2 22 1 3 3

y y 02 2 2

(vô nghiệm).

Vậy hệ đã cho có ba nghiệm là 1 13 1 13

(x; y) (2;1), 1 13; , 1 13;2 2

.

9. Ví dụ 9

Giải hệ phương trình

2011 20122013 zx

x z2012 2013

2011 xyy x

2013 20112012 yz

z y

.

HD: Đk x, y, z 0 . Ta thay 2011 = a, 2012 = b và 2013 = c.

Với đk đó, hệ

2

2

2

ca bcc czx

x zab ca

a axyy x

bc abb byz

z y

.

Cộng vế với vế của ba phương trình trên, ta được 2 2 2a b c axy+byz+czx (1).

www.VNMATH.com

Page 77: [Vnmath.com] Ky Yeu Trai He Hv 2012

Một số phương pháp giải Hệ phương trình cho học sinh giỏi Nguyễn Anh Tuấn

76

Mặt khác hệ

2 2

2 2

2 2

az bx czx z x

bx cy axy x y

cy ax=byz y z

Cộng vế với vế của ba phương trình này, ta được 2 2 2 2 2 2x y +y z z x axy+byz+czx (2).

Từ (1) và (2), ta được 2 2 2(a xy) (b yz) (c zx) 0

xy a

yz b

zx c

.

Mà a, b, c dương nên x, y, z cùng dấu, từ đó

ca ab bc ca ab bc(x; y;z) ; ; , ; ;

b c a b c a

.

Trở lại bài toán, hệ có hai nghiệm là

2013.2011 2011.2012 2012.2013 2013.2011 2011.2012 2012.2013(x; y;z) ; ; , ; ;

2012 2013 2011 2012 2013 2011

.

10. Ví dụ 10

Giải hệ phương trình 2 2 2

2 2 2

x y z 1

xyz 1

x y z y z x

y z x x y z

(1)

(2)

(3)

HD: Đặt 2 2z x

a , by z

và 2y

cx

. Từ (2) có abc = xyz = 1,

từ (3) có 1 1 1

a b c a b c ab bc ca abc 1 0a b c

(a 1)(b 1)(c 1) 0

a 1

b 1

c 1

.

Nếu a = 1 thì 2z y (4), thay vào (1) và (2) ta được hệ 2

3

x z z 1

xz 1

(5).

Từ đó 2 5 4 33

1z z 1 z z z 1 0

z

4 3z (z 1) (z 1) 0

4 2z (z 1) (z 1)(z z 1) 0

4 2(z 1)(z z z 1) 0

www.VNMATH.com

Page 78: [Vnmath.com] Ky Yeu Trai He Hv 2012

Một số phương pháp giải Hệ phương trình cho học sinh giỏi Nguyễn Anh Tuấn

77

z 1 (do 2

4 2 1 3z 0,z z 1 z 0

2 4

).

Thay vào (4) và (5), ta được (x;y;z) = (- 1;1;- 1). Tương tự nếu b = 1, c = 1, ta được (x;y;z) = (- 1;- 1;1), (1;- 1;- 1).

11. Ví dụ 11

Giải hệ phương trình

1 1 1

x y z 2

1 1 1

y z x 3

1 1 1

z x y 4

.

HD: Đk xyz 0, x y, y z,z x .

Khi đó hệ tương đương với

xy zxx y z

2yz xy

x y z3

zx yzx y z

4

.

xy zx yz xy zx yz xy yz zx xy yz zx

9 1 5 32 3 42 2 2 2

.

Mà y 0, z 0 nên suy ra 5

y x,z 5x3

.

Thay vào phương trình đầu của hệ đã cho, ta được 1 1 1 23 23 23

x , y , z5x 2 10 6 2x 5x3

(thỏa mãn).

Vậy hệ đã cho có nghiệm là 23 23 23

(x; y;z) ; ;10 6 2

.

12. Ví dụ 12

Giải hệ phương trình 1 1 1

20 x 11 y 2007 zx y z

xy yz zx 1

.

HD: Đk x 0, y 0, z 0 .

Giả sử (x; y; z) là nghiệm của hệ, suy ra x, y, z cùng dấu và vì vậy

x y 0, y z 0, z x 0 .

Ta có 21 x 1

20 x 20x x

2x xy yz zy20

x

www.VNMATH.com

Page 79: [Vnmath.com] Ky Yeu Trai He Hv 2012

Một số phương pháp giải Hệ phương trình cho học sinh giỏi Nguyễn Anh Tuấn

78

20(x y)(z x)

x

.

Tương tự, ta cũng có 1 11(x y)(y z)11 y

y y

1 2007(y z)(z x)2007 z

z z

.

Suy ra

20(z x)y 11(y z)x20(x y)(z x) 11(x y)(y z) 2007(y z)(z x)

20(x y)z 2007(y z)xx y z

9xy 20yz 11zx 0

2007xy 20yz 1987zx 0

.

Từ hệ này và điều kiện đã cho xy yz zx 1 , ta được 988xy

1019 , nghĩa là x và y trái

dấu (mâu thuẫn). Vậy hệ đã cho vô nghiệm.

Nhận xét: Ta có thể giải bằng Phương pháp lượng giác.

13. Ví dụ 13

Giải hệ phương trình

2 2 2 2 2

2 2 2 2 2

2 2 2 2 2

x (y z) (3x x 1)y z

y (z x) (4y y 1)z x

z (x y) (5z z 1)x y

(1)

(2)

(3)

HD: Nếu xyz = 0 thì dễ thấy hệ phương trình có nghiệm là (x;y;z) = (x;0;0), (0;y;0), (0;0;z).

Nếu xyz 0 thì đặt 1 1 1

a, b, cx y z , khi đó hệ

2 2

2 2

2 2

(b c) 3 a a

(c a) 4 b b

(a b) 5 c c

(1')

(2 ')

(3')

Cộng vế với vế các phương trình (1’), (2’) và (3’) rồi rút gọn, ta được 2(a b c) (a b c) 12 0

a b c 3

a b c 4

.

Nếu a b c 3 thì thay vào (1’), (2’) và (3’), ta được 5 5

x , y 1, z6 4

.

Nếu a b c 4 thì ta được 9 3 9

x , y ,z13 4 11

.

Vậy hệ phương trình đã cho có các nhgiệm là

5 5 9 3 9(x; y;z) (x;0;0), (0; y;0), (0;0; z), ; 1; , ; ;

6 4 13 4 11

.

www.VNMATH.com

Page 80: [Vnmath.com] Ky Yeu Trai He Hv 2012

Một số phương pháp giải Hệ phương trình cho học sinh giỏi Nguyễn Anh Tuấn

79

14. Ví dụ 14

Giải hệ phương trình 2 2 2 2

3 3 3 3

2 2 2 2

x y z t 12

x y z t 50

x y z t 252

x t y z 2xyzt

.

HD: Ta có hệ tương đương với

2 2

2 2

2

x y z t 12

(x t) (y z) 2xt 2yz 50

(x t) (x t) 3xt (y z) (y z) 3yz 252

(xt yz) 0

(*)

Từ (*) có xt = yz. Đặt x + t = a, y + z = b, xt = yz = c, khi đó ta có hệ

2 2

2 2 2

a b 12 a b 12

a b 4c 50 ab 2c 47

a(a 3c) b(b 3c) 252 (a b) (a b) 3ab 36c 252

a 5 a 7

b 7 b 5

c 6 c 6

.

Hay ta có các hệ x t 5

y z 7

xt yz 6

và x t 7

y z 5

xt yz 6

.

Dễ dàng giải hai hệ này ta được hệ phương trình đã cho có 8 nghiệm là (x; y;z) (2;1;6;3), (2;6;1;3), (3;1;6;2), (3;6;1;2), (1;2;3;6), (1;3;2;6), (6;2;3;1), (6;3;1; 2) .

15. Ví dụ 15 (Tuyển sinh vào trường THPT Chuyên Bắc Giang, năm học

2011 - 2012)

Giải hệ phương trình 2 2 4 2

2

x x 4y 8y y 1

5x+6 2y 7 7

(1)

(2)

HD: Điều kiện 6

x5

.

Biến đổi 22 2 2 2(1) x 2y x y y 3y 4 0

2

22 2 2

x 2y 0

x y y 3y 4 0

.

Xét x = 2y2. Thay vào (2) được 5x+6 x 7 7 (3).

www.VNMATH.com

Page 81: [Vnmath.com] Ky Yeu Trai He Hv 2012

Một số phương pháp giải Hệ phương trình cho học sinh giỏi Nguyễn Anh Tuấn

80

Giải (3) được x = 2 (thỏa mãn điều kiện), suy ra y = 1.

Xét 22 2 2x y y 3y 4 0 , ta được x = y = 0 không thỏa mãn (2).

Vậy hệ phương trình đã cho có hai nghiệm là (x;y) = (2;1), (2;-1).

16. Ví dụ 16

Giải hệ phương trình 3 3 3

3 2 4

x y 3(x y)

4x 6x 4x 1 15y

(1)

(2)

HD: Ta thấy 33 3(1) x y 3 xy( x y) 3(x y)

3 3 33 3 3( x y)(2 x y)( x 2 y) 0

3 3

3 3

3 3

x y 0

2 x y 0

x 2 y 0

y x

y 8x

8y x

.

Xét 4 4 4(2) (x 1) 15y x (3).

Nếu y = - x, thay vào (3) ta được 4 4 x 1 2x(x 1) 16x

x 1 2x

x 1

1x

3

.

Nếu y = 8x, thay vào (3) ta được 4

4 4

4

x 1 61441x(x 1) 61441x

x 1 61441x

4

4

1x

61441 11

x61441 1

.

Nếu 1

y x8

, thay vào (3) ta được

4

4 4

4

4111x 1 x

4111 4096(x 1) x

4096 4111x 1 x

4096

www.VNMATH.com

Page 82: [Vnmath.com] Ky Yeu Trai He Hv 2012

Một số phương pháp giải Hệ phương trình cho học sinh giỏi Nguyễn Anh Tuấn

81

4

4

1x

41111

40961

x4111

14096

.

Vậy hệ phương trình đã cho có 6 nghiệm là

1 1 1 8 1 8 1 1 1 1(x; y) (1; 1), ; , ; , ; , ; , ;

3 3 a 1 a 1 a 1 a 1 b 1 8(b 1) b 1 8(b 1)

,

trong đó 4a 61441 , 44111

b4096

.

17. Ví dụ 17

Giải hệ phương trình 2 2

2 2

3x yx 3

x y

x 3yy 0

x y

.

HD: Dễ thấy x = 0 hoặc y = 0 đều không thỏa mãn hệ.

Xét xy 0 .

Khi đó hệ tương đương với 2 2

1 3 x y

x y 3x y x 3y

2 2

311 1xy xx y 3 3x y

2 2 2 2

321 (2x 3)yxx yx y x yy x

(2x 3)y 1 .

Do đó, (1) suy ra (3 x)(x 3y) y(3x y)

(3 x)(2x 3)(x(2x 3) 3y(2x 3)) y(2x 3)(3x(2x 3) y(2x 3)) (do 2x 3 0 ) 4 3 24x 24x 57x 63x 26 0

2(x 1)(x 2)(4x 12x 13) 0

x 1

x 2

(do 24x 12x 13 0 với mọi x 0 ).

Suy ra y 1

y 1

(thỏa mãn).

Vậy hệ phương trình đã cho có hai nghiệm là (x;y) = (1;-1), (2;1).

www.VNMATH.com

Page 83: [Vnmath.com] Ky Yeu Trai He Hv 2012

Một số phương pháp giải Hệ phương trình cho học sinh giỏi Nguyễn Anh Tuấn

82

18. Ví dụ 18

Giải hệ phương trình 2 2

2 2

15x 1 12

x y

15y 1 4

x y

.

HD: Dễ thấy xy 0 .

Khi đó hệ tương đương với 2 2

2 2 2 2

1 12 6 21 1

x y 5x 5x 5y

1 4 1 6 21

x y 5y x y 5x 5y

2 2 2 2

6 21

5x 5y

25 36 4

x y x y

(1)

(2)

Xét (2), ta được 4 2 2 4 2 2 2 236y 7x y 4x 0 (4x 9y )(x 4y ) 0

2 2x 4y 0 (do 2 24x 9y 0 với mọi xy 0 ).

x 2y

x 2y

.

Thay vào (1), từ đó hệ phương trình đã cho có hai nghiệm là 2 1

(x; y) (2;1), ;5 5

.

I.2.3. Một số bài tập tương tự

1. Bài 1 (Đề tuyển sinh vào trường THPT Chuyên Bắc Giang, năm học

2008 - 2009)

Giải hệ phương trình 2

2 2

y 2x 2xy 1

x y x y 8

.

2. Bài 2

Giải hệ phương trình 2 2 2

3 3 3

x y z 7

x y z 37

x y z 1

.

3. Bài 3

Giải hệ phương trình 4 3 2 2

3 2

x x y x y 1

x y x xy 1

.

www.VNMATH.com

Page 84: [Vnmath.com] Ky Yeu Trai He Hv 2012

Một số phương pháp giải Hệ phương trình cho học sinh giỏi Nguyễn Anh Tuấn

83

4. Bài 4

Giải hệ phương trình

1 1 1

x y z 2

1 1 1

y z x 3

1 1 1

z x y 4

.

5. Bài 5

Giải hệ phương trình

2

2

2

5x (y z)

3

y (z x) 3

1z (x y)

3

.

www.VNMATH.com

Page 85: [Vnmath.com] Ky Yeu Trai He Hv 2012

Một số phương pháp giải Hệ phương trình cho học sinh giỏi Nguyễn Anh Tuấn

84

I.3. Phương pháp đặt ẩn phụ I.3.1. Một số lưu ý Phát hiện ẩn phụ u = f(x;y), v = g(x;y) hay a = f(x), b = f(y), c = f(z) có ngay

trong từng phương trình hoặc xuất hiện sau một số phép biến đổi hằng đẳng thức cơ bản hoặc phép chia cho một biểu thức khác không để đưa hệ về dạng đơn giản hơn.

I.3.2. Một số ví dụ

1. Ví dụ 1

Giải hệ phương trình 2

2

x 1 y(y x) 4y

(x 1)(y x 2) y

(1)

(2)

HD: Ta thấy y = 0 không thỏa mãn (1), do đó hệ

2

2

x 1y x 4

y

x 1(y x 2) 1

y

.

Đặt 2x 1

u ;v y x 2y

suy ra u = v = 1.

Ta được hệ có nghiệm là (x;y) = (1;2), (-2;5).

2. Ví dụ 2

Giải hệ phương trình

2 22

34xy 4(x y ) 7

(x y)

12x 3

x y

(1)

(2)

HD: Đk x y 0 .

Khi đó ta được

2 22

33(x y) (x y) 7

(x y)

1x y x y 3

x y

.

Đặt 1

u x y ; v x yx y

(với u 2 ), ta được hệ 2 23u v 13

u v 3

.

Từ đó hệ có nghiệm là (x;y) = (1;0).

3. Ví dụ 3

Giải hệ phương trình 33

2 2

y 1 x 3

x y 82

.

HD: Đk x 0 .

www.VNMATH.com

Page 86: [Vnmath.com] Ky Yeu Trai He Hv 2012

Một số phương pháp giải Hệ phương trình cho học sinh giỏi Nguyễn Anh Tuấn

85

Đặt u x , 33v y 1 với u 0 , suy ra 2

3 3

x u

y 1 v

.

2 4

3 3

x u

y v 1

, thay vào hệ ta được

4 3 4 3

u v 3 v 3 u

u v 81 u 3 u 81( )

3 2u 3 u 2u 15u 18 0( )( )

u 3 (do 3 2u 2u 15u 18 0 với u 0 ). Từ đó, hệ phương trình có nghiệm là (x;y) = (9;1).

4. Ví dụ 4

Giải hệ phương trình

1 1 1 8x y z

3x y z

1 1 1 118x y z

x y z 9

1 1 1 728x x y y z z

27x x y y z z

.

HD: Đk x 0, y 0, z 0 .

Đặt 1

x ax

, 1

y by

, 1

z cz

.

Khi đó 21x a 2

x , 31

x x a 3ax x

, 21y b 2

y , 31

y y b 3by y

,

21z c 2

z , 31

z z c 3cz z

.

Hệ đã cho trở thành 2 2 2

3 3 3

8a b c

364

a b c9

512a b c

27

(1)

(2)

(3)

Ta được 3

3 3 3 3 8 5123(a b)(b c)(c a) (a b c) (a b c ) 0

3 27

(do (1) và (3)).

Nếu a + b = 0 thì từ (1) suy ra 8

c3

, thay vào (2) ta được 2 2a b 0 a b 0 .

Do đó 1

x 0x

, 1

y 0y

, 1 8

z3z

, suy ra (x; y;z) (1;1;9) .

Tương tự, nếu b + c = 0 thì (x; y;z) (9;1;1) , nếu c + a = 0 thì (x; y;z) (1;9;1) .

Vậy hệ phương trình có ba nghiệm là (x; y;z) (9;1;1), (1;9;1), (1;1;9) .

www.VNMATH.com

Page 87: [Vnmath.com] Ky Yeu Trai He Hv 2012

Một số phương pháp giải Hệ phương trình cho học sinh giỏi Nguyễn Anh Tuấn

86

I.3.3. Một số bài tập tương tự

1. Bài 1

Giải hệ phương trình 3 3 3

2 2

8x y 27 18y

4x y 6x y

.

2. Bài 2

Giải hệ phương trình 3 3 2

4 4

x 8y 4xy 1

2x 8y 2x y 0

.

3. Bài 3

Giải hệ phương trình 2x y x 4y 1

2x y x 3

.

4. Bài 4

Giải hệ phương trình 4 3 2 2

3 2

x x y x y 1

x y x xy 1

.

5. Bài 5

Giải hệ phương trình

2 2

3 3

2xy y x y x y x y

14 2 2

x y x y9

2 2

.

www.VNMATH.com

Page 88: [Vnmath.com] Ky Yeu Trai He Hv 2012

Một số phương pháp giải Hệ phương trình cho học sinh giỏi Nguyễn Anh Tuấn

87

I.4. Phương pháp lượng giác I.4.1. Một số lưu ý

Khi giải phương trình lượng giác ta có thể đặt ẩn phụ cho các hàm số lượng

giác để chuyển phương trình lượng giác về phương trình đại số cơ bản mà ta đã biết cách giải. Tuy nhiên trong nhiều bài toán giải Hệ phương trình (Phương trình) ta cũng

có thể làm ngược lại rất hiệu quả, bằng những tính chất của hàm số lượng giác ta sẽ chuyển bài toán đại số về bài toán lượng giác và giải giải quyết bài toán lượng giác

này. Thực chất Phương pháp lượng giác cũng có thể gọi là Phương pháp đặt ẩn phụ. Khi giải Hệ phương trình bằng Phương pháp lượng giác ta có thể đặt

f (x) k sin nếu f (x) k;k với điều kiện ;2 2

hoặc f (x) k cos với điều

kiện 0; . Cũng có khi đặt f (x) tan ;f (x) cot , … để đưa Hệ phương trình đã

cho về Hệ phương trình hoặc phương trình lượng giác với một số điều cần lưu ý như

hàm số sinx và cosx có tập giá trị là 1;1 , 2 2 2 2sin x cos x 1 1 sin x cos x , còn

hai hàm số tanx và cotx có tập giá trị là R và 2 22 2

1 11 tan x ,1 cot x

cos x sin x , ...

Thường ta dựa vào điều kiện của ẩn để quyết định cách đặt ẩn phụ lượng giác.

Giải Hệ phương trình hoặc Phương trình lượng giác rồi từ đó tìm nghiệm của Hệ phương trình đã cho.

Đôi khi trong lời giải ta cũng kết hợp với việc sử dụng Phương pháp đánh giá.

I.4.2. Một số ví dụ

1. Ví dụ 1

Giải hệ phương trình 1 1 1

20 x 11 y 2007 zx y z

xy yz zx 1

.

HD: Đk x 0, y 0, z 0 .

Đặt x = tan A, y = tanB, z = tanC.

Khi đó xy yz zx 1 A B C k2

, k Z .

Mặt khác 1 1 120 x 11 y 2007 z

x y z

, suy ra

20sin 2B 11sin 2A20 11 2007

20sin 2C 2007sin 2Asin 2A sin 2B sin 2C

.

Do đó

20(sin2C – sin2B) = 1998sinAcosA 40cos(B + C)sin(B – C) = 1998sinAcosA

Mà A B C k2

, k Z nên 40sin(B – C) = 1998sin(B + C)

www.VNMATH.com

Page 89: [Vnmath.com] Ky Yeu Trai He Hv 2012

Một số phương pháp giải Hệ phương trình cho học sinh giỏi Nguyễn Anh Tuấn

88

- 1958sinBcosC = 2038sinCcosB, hay – 1958y = 2038z. Suy ra y và z trái dấu.

Vậy hệ đã cho vô nghiệm.

2. Ví dụ 2

Giải hệ phương trình 2 2 2 2 2 2

x y z xyz

x(y 1)(z 1) y(z 1)(x 1) z(x 1)(y 1) 0

.

HD: Nếu xyz = 0 thì dễ dàng thấy hệ có các họ nghiệm là (x;y;z) = (0; y; - y), (z; 0; - z), (x; -x; 0).

Nếu xyz 0 thì chia cả hai vế của phương trình thứ hai cho 4xyz, ta được 2 2 2 2 2 2y 1 z 1 z 1 x 1 x 1 y 1

. . . 02y 2z 2z 2x 2x 2y

(1)

Đặt x y ztan , tan , tan thì tan tan tan tan .tan .tan

k , với k Z .

Do đó 2 2 2 k2 , với k Z .

Suy ra 2 2 2 2 2 2 1cot .cot cot .cot cot .cot

2 2 2 2 2 2y 1 z 1 z 1 x 1 x 1 y 1

. . . 12y 2z 2z 2x 2x 2y

(vô lý).

Vậy hệ đã cho có các họ nghiệm là (x;y;z) = (0; y; - y), (z; 0; - z), (x; -x; 0).

3. Ví dụ 3

Giải hệ phương trình 2 2x 1 y y 1 x 1

(1 x)(1 y) 2

.

HD: Đk x 1 y 1, .

Đặt x cos ;y=cos với 0, ; , khi đó hệ tương đương với

c c 12

1 c 1 c 2c c 1 0

os .sin + os .sin( os )( os ) sin os sin . os

Đặt t csin os , t 2 suy ra 21 t

c2

sin . os .

Khi đó 2

21 tt 1 0 t 2t 3 0

2

t 1 (do t 2 ).

Với t =1, ta được 2 14 2

sin

0 .

Từ đó hệ phương trình đã cho có nghiệm là (x;y) = (0;1).

www.VNMATH.com

Page 90: [Vnmath.com] Ky Yeu Trai He Hv 2012

Một số phương pháp giải Hệ phương trình cho học sinh giỏi Nguyễn Anh Tuấn

89

I.4.3. Một số bài tập tương tự

1. Bài 1

Giải hệ phương trình 2 2

2(x y)(1 4xy) 3

x y 1

.

2. Bài 2

Giải hệ phương trình

2 2

2 2

x 1 y y 1 x 1

1x 1 y y 1 x

2

.

3. Bài 3

Giải hệ phương trình

2

2

2

2x x y y

2y y z z

2z z x x

.

4. Bài 4

Giải hệ phương trình

2

2

2

x(4 y ) 8y

y(4 z ) 8z

z(4 x ) 8x

.

5. Bài 5

Giải hệ phương trình

2 3

2 3

2 3

x 3z 3z x z 0

y 3x 3x y x 0

z 3y 3y z y 0

.

6. Bài 6

Giải hệ phương trình 1 1 1

3 x 4 y 5 zx y z

xy yz zx 1

.

www.VNMATH.com

Page 91: [Vnmath.com] Ky Yeu Trai He Hv 2012

Một số phương pháp giải Hệ phương trình cho học sinh giỏi Nguyễn Anh Tuấn

90

I.5. Phương pháp đánh giá I.5.1. Một số lưu ý 1. Phương trình f(x) = m có nghiệm khi và chỉ khi m thuộc tập giá trị của hàm

số y = f(x) và số nghiệm của phương trình là số giao điểm của đồ thị hàm số y = f(x) với đường thẳng y = m.

2. Khi gặp hệ phương trình dạng f (x) f (y)

g(x; y) 0

(1)

(2), ta có thể tìm lời giải theo

một trong hai cách.

Cách 1

Xét (1) f (x) f (y) 0 , rồi tìm cách đưa về phương trình tích có vế phải bằng 0.

Cách 1 Xét hàm số y = f(t), ta thường gặp trường hợp hàm số liên tục trong tập xác định của nó.

Nếu y = f(t) đơn điệu thì từ (1) suy ra x = y. Khi đó bài toán đưa về giải (hoặc biện luận) phương trình (2) theo ẩn x.

Nếu hàm số y = f(t) có một cực trị tại t = a thì nó thay đổi chiều biến thiên một lần khi qua a. Từ (1) suy ra x = y hoặc x, y nằm về hai phía đối với a.

3. Nếu hệ phương trình ba ẩn x, y, z không thay đổi khi hoán vị vòng quanh đối

với x, y, z thì không giảm tính tổng quát có thể giả sử x max x;y;z , nghĩa là

x y, x z .

4. Sử dụng đạo hàm hoặc các bất đẳng thức cổ điển Cauchy, Bunhiacopxky… thường đánh giá dẫn đến x = y hoặc x = y = z.

I.5.2. Một số ví dụ 1. Ví dụ 1

Giải hệ phương trình

x y

32 2

e e x y

xlog log 4y 10

2

(1)

(2)

HD: Đk x > 0, y > 0.

Với đk đó x y(1) e x e y (3).

Xét hàm số tf (t) e t , suy ra tf '(t) e 1 0 với mọi t > 0.

Do đó hàm số f(t) đồng biến trong khoảng (0; ) .

Từ (3) suy ra f (x) f (y)

x yx 0, y 0

Thay vào (2), ta được 32 2 22

xlog log 4y 10 log x 1 2(2 3log x) 10

2

www.VNMATH.com

Page 92: [Vnmath.com] Ky Yeu Trai He Hv 2012

Một số phương pháp giải Hệ phương trình cho học sinh giỏi Nguyễn Anh Tuấn

91

2log x 1 .

Tứ đó, hệ có nghiệm duy nhất là (x;y) = (2;2).

2. Ví dụ 2

Giải hệ phương trình 2 2

ln(1 x) ln(1 y) x y

2x 5xy y 0

(1)

(2)

HD: Đk x > -1, y > -1.

Với điều kiện đó (1) ln(1 x) x ln(1 y) y (3).

Xét hàm số f(t) = ln(1+t) – t, với t ( 1; ) .

1 tf '(t) 1

1 t 1 t

, do đó f '(t) 0 t 0 .

Hàm số f(t) đồng biến trong khoảng (-1;0) và nghịch biến trong khoảng (0; ) .

Ta thấy (3) f (x) f (y) , từ đó suy ra x = y (nếu x, y cùng một khoảng đơn điệu)

hoặc xy < 0 (nếu x, y khác khoảng đơn điệu). Nếu xy < 0 thì Vt(2) < 0 (loại). Nếu x = y thì thay vào (2), ta được nghiệm của hệ là (x;y) = (0;0).

3. Ví dụ 3

Giải hệ phương trình 2 2 2x 2y x 2y 2y x 2

x

4 9.3 (4 9 ).7

4 4 4x 4 2y 2x 4

(1)

(2)

HD: Đk y x 2 0 .

Đặt 2t x 2y , khi đó t 2 t 2 t(1) 4 3 (4 9 ).7

t 2 2t

t 2 2t

4 3 4 3

7 7

f (t 2) f (2t) ,

trong đó t tt

t

4 3 1 3f (t) 4

7 7 7

, với t R .

Dễ thấy f(t) làm hàm số nghịch biến do 1 3

0 ; 17 7

.

Do đó f (t 2) f (2t) t 2 2t

t 2 , hay 2x 2y 2 .

Khi đó 2(1) 2y x 2 , thay vào (2) ta được x 24 4 4x 4 x 2 2x 4

x 1 24 x 1 (x 1) 1 (3)

z 24 z z 1 , trong đó z = x - 1.

z 24 z z 1 (do z z4 .4 1 )

Suy ra z z4 4 2z .

www.VNMATH.com

Page 93: [Vnmath.com] Ky Yeu Trai He Hv 2012

Một số phương pháp giải Hệ phương trình cho học sinh giỏi Nguyễn Anh Tuấn

92

Xét hàm số z zg(z) 4 4 2z , z R .

Có z zg '(z) ln 4(4 4 ) 2 0 , với mọi z R .

Suy ra hàm số g(z) đồng biến trên R.

Dễ thấy z = 0 là nghiệm duy nhất của g(z) 0 , dẫn đến (3) có nghiệm duy nhất x = 1.

Từ đó hệ đã cho có nghiệm duy nhất là 1

(x; y) 1;2

.

4. Ví dụ 4

Giải hệ phương trình 3 3

8 4

x 5x y 5y

x y 1

(1)

(2)

HD: Từ (2), ta được x 1, y 1 .

Xét 3f (t) t 5t , dễ thấy hàm số nghịch biến trên đoạn 1;1 .

Do đó (1) x y , thay vào (2) ta được 8 4x x 1 0 .

Tóm lại hệ đã cho có nghiệm là

1 5 1 5 1 5 1 5(x; y) ; , ;

2 2 2 2

.

5. Ví dụ 5

Giải hệ phương trình

2

3 2

2

23

2xyx x y

x 2x 92xy

y y xy 2y 9

.

HD: Cộng vế với vế hai phương trình của hệ ta được

2 2

3 2 23

2xy 2xyx y

x 2x 9 y 2y 9

(1).

Ta thấy 3 2 23

3 2 3 2

2 xy 2 xy2xyx 2x 9 (x 1) 8 2 xy

2x 2x 9 x 2x 9

.

Tương tự 23 y 2y 9 xy .

Mặt khác, theo Bất đẳng thức Cauchy 2 2x y 2 xy , nên Vt(1) Vp(1) .

Dấu đẳng thức xảy ra khi x y 0

x y 1

, thử lại ta được hệ có hai nghiệm là (x;y) =

(0;0), (1;1).

6. Ví dụ 6

Giải hệ phương trình 2 y 1

2 x 1

x x 2x 2 3 1

y y 2y 2 3 1

.

www.VNMATH.com

Page 94: [Vnmath.com] Ky Yeu Trai He Hv 2012

Một số phương pháp giải Hệ phương trình cho học sinh giỏi Nguyễn Anh Tuấn

93

HD: Đặt u = x – 1, v = y – 1, ta được 2 v

2 u

u u 1 3

v v 1 3

(1)

(2).

Trừ vế với vế hai phương trình trên, ta được 2 u 2 vu u 1 3 v v 1 3 (3).

Xét hàm số 2 tf (t) t t 1 3 , có 2

t

2

t 1 tf '(t) 3 ln 3

t 1

.

Dễ thấy f'(t) > 0 với mọi t, do đó hàm số f'(t) đồng biến trên R.

Từ đó (3) u v , thay vào (1) ta được 2 uu u 1 3 (4).

Do hai vế của (4) đều dương nên 2(4) ln(u u 1) u ln 3 0 .

Xét hàm số 2g(u) ln(u u 1) u ln 3 , có 2

1g '(u) ln 3 1 ln 3 0

u 1

,

với mọi u R .

Do đó hàm số g(u) nghịch biến trên R, ta được (4) có nghiệm duy nhất là u = 0. Từ đó hệ đã cho có nghiệm là (x;y) = (1;1).

7. Ví dụ 7

Giải hệ phương trình 3

3

y x 3x 4

x 2y 6y 2

.

HD: Hệ đã cho tương đương với 2

2

y 2 (x 1) (x 2)

x 2 2(y 1) (y 2)

(1)

(2)

Nếu x > 2 thì từ (1) suy ra y - 2 < 0, mẫu thuẫn với (2) có x - 2 và y - 2 cùng dấu. Tương tự, nếu x < 2 cũng vô lý.

Vậy nghiệm của hệ là (x;y) = (2;2).

8. Ví dụ 8

Giải hệ phương trình 3 2

2 2 2

x 3y 6y 4 0

x x y 2y 0

.

HD: Xét phương trình thứ nhất của hệ, ta được 3 2x 3(y 1) 1 1 x 1 (1).

Mặt khác, từ phương trình còn lại của hệ, ta thấy 2 2 22

2yx (y 1) 2y x 1

y 1

1 x 1 (2). Từ (1) và (2) suy ra x = -1, thay vào phương trình đầu của hệ suy ra y = 1. Thử lại đúng, vậy hệ có nghiệm là (x;y) = (-1;1).

9. Ví dụ 9 Giải hệ phương trình (Tuyển sinh Đại học Khối A năm 2010)

www.VNMATH.com

Page 95: [Vnmath.com] Ky Yeu Trai He Hv 2012

Một số phương pháp giải Hệ phương trình cho học sinh giỏi Nguyễn Anh Tuấn

94

2

2 2

(4x 1)x (y 3) 5 2y 0

4x y 2 3 4x 7

(1)

(2)

HD: Đk 3 5x , y

4 2 .

Xét 2(1) (4x 1)2x (5 2y 1) 5 2y .

Đặt 2 22x u

(u 1)u (v 1)v5 2y v

.

Dễ thấy hàm số 2f (t) (t 1)t đồng biến trên R, suy ra u = v hay

2

x 02x 5 2y 5 4x

y2

.

Thế y vào (2), ta được 2

2 254x 2x 2 3 4x 7 0

2

(3).

Dễ thấy x = 0 và 3

x4

không là nghiệm của (3).

Xét hàm số 2

2 25g(x) 4x 2x 2 3 4x 7

2

trên khoảng 3

0;4

.

Ta có 2 25 4 4g '(x) 8x 8x 2x 4x(4x 3) 0

2 3 4x 3 4x

trên khoảng

30;

4

.

Do đó g(x) nghịch biến trên khoảng 3

0;4

.

Mà 1

g( ) 02 , nên (3) có nghiệm duy nhất là

1x

2 , suy ra y = 2.

Vậy hệ có nghiệm duy nhất là (x;y) = (1

2;2).

10. Ví dụ 10

Giải hệ phương trình 3

4

x 1 y 8 x

(x 1) y

(1)

(2)

HD: Đk x 1, y 0 .

Thế y từ (2) vào (1), ta được 2 3x 1 (x 1) 8 x (3).

Từ (3) có 3 2x 1 x x 2x 9 (4).

Xét hàm số 3 2f (x) x x 2x 9 , với x 1 .

Ta có 2f '(x) 3x 2x 2 0 , với mọi x 1 .

Do đó hàm số f(x) luôn nghịch biến trong đoạn 1; .

Mặt khác hàm số g(x) x 1 luôn đồng biến trong đoạn 1; .

Từ đó suy ra (4) có nghiệm duy nhất là x = 2.

www.VNMATH.com

Page 96: [Vnmath.com] Ky Yeu Trai He Hv 2012

Một số phương pháp giải Hệ phương trình cho học sinh giỏi Nguyễn Anh Tuấn

95

Vậy hệ có nghiệm duy nhất là (x;y) = (2;1). Nhận xét: Với bài toán này, dùng đạo hàm để giải quyết là khá hay.

Tuy nhiên, ta cũng có thể không sử dụng đạo hàm mà có thể biến đổi khéo léo như sau:

Ta thấy 2 3(3) ( x 1 1) ((x 1) 1) x 8 0

2x 2x(x 2) (x 2)(x 2x 4) 0

x 1 1

21(x 2) x x 4 0

x 1 1

x 2 (do biểu tức còn lại luôn dương với mọi x 1 ).

11. Ví dụ 11

Giải hệ phương trình 2

x y z 2

4x 4y 2xy z 4

(1)

(2)

HD: Coi z là tham số, ta được 2

x y 2 z

z 4z 4xy

2

.

Hệ có nghiệm 2

2 2(z 2)(z 2) 4 0 (z 2) 0

2

z 2 . Khi đó x = y = 0 Vậy hệ phương trình có nghiệm duy nhất là (x;y;z) = (0;0;2).

12. Ví dụ 12

Giải hệ phương trình 5 4 10 6

2

x xy y y

4x 5 y 8

(1)

(2)

HD: Đk 5

x4

.

Dễ thấy y 0 , chia cả hai vế của (1) cho 5y 0 , ta được 5

5x xy y

y y

.

Hàm số 5f (t) t t luôn đồng biến trên khoảng xác định nên 2xy x y

y .

Thế vào (2), ta được 4x 5 x 8 6 , đánh giá hoặc biến đổi tương đương dẫn đến

x = 1. Vậy hệ phương trình có hai nghiệm là (x;y) = (1;-1), (1;1).

13. Ví dụ 13

Giải hệ phương trình 2 3

2 3

x 2x y 2

x y 2x y 0

(1)

(2)

www.VNMATH.com

Page 97: [Vnmath.com] Ky Yeu Trai He Hv 2012

Một số phương pháp giải Hệ phương trình cho học sinh giỏi Nguyễn Anh Tuấn

96

HD: Coi (1) là phương trình bậc hai ẩn x, (1) có nghiệm 3' 1 y 0

y 1 (*1).

Coi (2) là phương trình bậc hai ẩn x, (2) có nghiệm 4' 1 y 0

1 y 1 (*2).

Từ (*1) và (*2) suy ra y = -1, thay vào (1) ta được x = 1 (thỏa mãn).

Vậy hệ phương trình có nghiệm là (x;y) = (1;-1).

14. Ví dụ 14

Giải hệ phương trình 24

4

2x 2 6 x y 2 2

2x 2 6 x 2 2y 8 2

.

HD: Đk 0 x 6 . Cộng vế với vế hai phương trình của hệ rồi biến đổi, ta được

24 4( 2x 2 6 x ) ( 2x 2 6 x) (y 2) 6 3 2 6 3 2 (1).

Áp dụng BĐT Bunhiacopxky, ta được 2( 2x 2 6 x ) (1 2)(2x (12 2x)) 36

2x 2 6 x 6 (2).

Đẳng thức xảy ra khi và chỉ khi x = 2.

Tương tự ta có 24 4( 2x 2 6 x ) (1 2)( 2x 2 6 x ) 18

4 42x 2 6 x 3 2 (3).

Đẳng thức xảy ra khi và chỉ khi x = 2.

Từ (2) và (3), ta được 4 4( 2x 2 6 x ) ( 2x 2 6 x ) 6 3 2 (4).

Từ (1) và (4) dẫn đến các đẳng thức ở (1), (2) và (3) xảy ra đồng thời.

Khi đó x = 2 và y 2 , thử lại ta đượ hệ có nghiệm là (x; y) (2; 2) .

Nhận xét: Có thể xét hàm số 4 4f (x) ( 2x 2 6 x ) ( 2x 2 6 x) , với

0 x 6 .

Tính đạo hàm và xét sự biến thiên của f(x) trong đoạn 0;6 .

15. Ví dụ 15

Giải hệ phương trình

2

2

3

4 2

4

6 4 2

2xy

x 1

3yz

y y 1

4zx

z z z 1

.

HD: Ta thấy 2

2

2xy 0

x 1

, với mọi x.

Nếu y = 0 thì x = z = 0, do đó (x;y;z) =(0;0;0) là nghiệm của hệ.

www.VNMATH.com

Page 98: [Vnmath.com] Ky Yeu Trai He Hv 2012

Một số phương pháp giải Hệ phương trình cho học sinh giỏi Nguyễn Anh Tuấn

97

Nếu y > 0 thì z > 0, x > 0. Dễ thấy 2x 1 2x 0 nên 2

2

2xx

x 1

hay y x .

Theo BĐT Cauchy, ta có 4 2 4 2 23y y 1 3 y .y .1 3y 0 nên 3

4 2

3yz y

y y 1

.

Từ phương trình thứ ba của hệ suy ra x z , do đó x y z x , suy ra x = y = z.

Thay vào phương trình đầu của hệ, ta được x = y = z = 1.

Vậy hệ phương trình có hai nghiệm là (x;y;z) = (0;0;0), (1;1;1).

16. Ví dụ 16

Giải hệ phương trình x y z 9

x 1 y 1 z 1 6

(1)

(2)

HD: Đk x, y, z 1 .

Áp dụng BĐT Bunhiacopxky, ta được 2 2( x 1 y 1 z 1) (1. x 1 1. y 1 1. z 1)

(1 1 1)(x 1 y 1 z 1) 36 .

Suy ra x 1 y 1 z 1 6 , dấu đẳng thức xảy ra khi x = y = z = 3 (thỏa mãn

(2)). Vậy hệ phương trình có nghiệm duy nhất là (x;y;z) = (3;3;3).

17. Ví dụ 17

Giải hệ phương trình

5 4 2

5 4 2

5 4 2

x x 2x y 2

y y 2y z 2

z z 2z x 2

.

HD: Dễ thấy x, y, z khác 0.

Nếu x > 1 thì 5 4 4 22

1x x x (x 1) 0 2x y 2 y 1

x ,

và 5 4 4 2 5 4 42

1y y y (y 1) 0 2y z 2 z 1 z z z (z 1) 0

y

22

12z x 2 x 1

z , mâu thuẫn.

Nếu x < 1 thì tất cả các bất đẳng thức ở trên cùng đổi chiều nên cũng mâu thuẫn. Nếu x = 1 thì dễ thấy y = 1 và z = 1.

Vậy hệ đã cho có nghiệm duy nhất là (x;y;z) = (1;1;1).

18. Ví dụ 18

Giải hệ phương trình

2 3

2 3

2 3

12x 48x 64 y

12y 48y 64 z

12z 48z 64 x

(1)

(2)

(3)

www.VNMATH.com

Page 99: [Vnmath.com] Ky Yeu Trai He Hv 2012

Một số phương pháp giải Hệ phương trình cho học sinh giỏi Nguyễn Anh Tuấn

98

HD: Giả sử 0 0 0(x; y;z) (x ; y ;z ) là nghiệm của hệ thì dễ thấy

0 0 0 0 0 0(x; y;z) (y ; z ; x );(z ; x ; y ) cũng là nghiệm của hệ, do đó có thể giả sử

x max x; y;z .

Ta thấy 2 212x 48x 64 12(x 2) 16 16 suy ra y > 2. Tương tự z > 2 và x > 2.

Trừ vế với vế của (1) và (3) ta được 3 3 2 2x y 12(z x ) 48(z x) 12(z x)(z x 4) (4)

Theo trên, có 3 3x y 0 , z x 0 , z x 4 0 , nên từ (4) suy ra x = y = z.

Thay vào (1), ta được 3 2 3x 12x 48x 64 0 (x 4) 0

x 4 . Vậy hệ có nghiệm là (x;y;z) = (4;4;4).

19. Ví dụ 19

Giải hệ phương trình

19 5 2013

19 5 2013

19 5 2013

x y 1890z z

y z 1890x x

z x 1890y y

(1)

(2)

(3)

HD: Dễ thấy 0 0 0(x; y;z) (x ; y ;z ) là nghiệm của hệ thì 0 0 0(x; y;z) ( x ; y ; z )

cũng là nghiệm của hệ. Không giảm tính tổng quát, giả sử có ít nhất hai trong ba số x, y, z không âm, chẳng

hạn x 0, y 0

Từ (1) suy ra z 0 .

Mặt khác, nếu 0 t 1 thì 2012 18 41890 t 2 t t .

Nếu t > 1 thì 2012 2012 2012 1006 18 41890 t 1 t 2 t 2t t t .

Do đó 2013 19 51890t t t t với mọi t > 0. Cộng vế với vế các phương trình trên, ta được x = y = z = 0. Vậy hệ có nghiệm là (x;y;z) = (0;0;0).

20. Ví dụ 20

Giải hệ phương trình 3

3

x y z 3

(1 x)(1 y)(1 z) 1 xyz

.

HD: Đk x, y, z 0 .

Theo BĐT Cauchy, ta có (1 x)(1 y)(1 z) 1 (x y z) (xy yz zx) xyz

2 33 331 3 xyz 3 (xyz) (xyz)

331 xyz .

Đẳng thức xảy ra khi và chỉ khi x = y = z, do đó hệ đã cho tương đương với

www.VNMATH.com

Page 100: [Vnmath.com] Ky Yeu Trai He Hv 2012

Một số phương pháp giải Hệ phương trình cho học sinh giỏi Nguyễn Anh Tuấn

99

x y z 3x y z 1

x y z

.

Vậy hệ có nghiệm là (x;y;z) = (1;1;1).

21. Ví dụ 21

Giải hệ phương trình 9 3 4 2

3x 4y 2z1

x 1 y 1 z 1

8 .x .y .z 1

(1)

(2)

HD: Từ (1) có 2x 4y 2z 1

x 1 y 1 z 1 x 1

, áp dụng BĐT Cauchy cho 8 số dương,

ta được 2 4 2

82 4 2

1 x x y y y y z z x y z8

x 1 x 1 x 1 y 1 y 1 y 1 y 1 z 1 z 1 (x 1) (y 1) (z 1)

Tương tự 3 3 2

83 3 2

1 x y z8

y 1 (x 1) (y 1) (z 1)

,

3 4

83 4

1 x y z8

z 1 (x 1) (y 1) (z 1)

.

Từ ba BĐT trên có 43 2 24 32 16

98

24 32 16

1 1 1 x y z. . 8

x 1 y 1 z 1 (x 1) (y 1) (z 1)

3 4 2

93 4 2

x y z8

(x 1) (y 1) (z 1)

.

Suy ra 3 4 29

1x y z

8 (3).

Kết hợp với (2) thì (3) có đẳng thức xảy ra khi và chỉ khi x y z 1 1

x y zx 1 y 1 z 1 9 8

.

Thử lại thấy thỏa mãn.

Vậy hệ có nghiệm là 1 1 1

(x; y;z) ; ;8 8 8

.

22. Ví dụ 22

Giải hệ phương trình

2

2

2

y20. 11y 2013

xz

20. 11z 2013y

x20. 11x 2013

z

(1)

(2)

(3)

HD: Từ (1), ta có 2

20y 11 2013 y 0

x

.

Tương tự, từ (2) và (3) z 0, x 0 .

www.VNMATH.com

Page 101: [Vnmath.com] Ky Yeu Trai He Hv 2012

Một số phương pháp giải Hệ phương trình cho học sinh giỏi Nguyễn Anh Tuấn

100

Do hệ hoán vị vòng quanh nên ta giả sử x max x; y;z , nghĩa là x y, x z .

Trừ vế với vế của (3) cho (1), ta được

3 2 2 22 2

x y20 11(x y) 0 20(x yz ) 11x z (x y) 0

z x

(4).

Vì x y 0, x z 0 nên dễ thấy x y 0 và 3 2x yz 0 .

Do đó (4) 3 2x yz

x y zx y

.

Thay vào (1), ta được 22011x 2013 11x 2013x 20 0

x .

Từ đó suy ra nghiệm của hệ.

23. Ví dụ 23

Giải hệ phương trình

3 2

3 2

3 2

x 3x 5x 1 4y

y 3y 5y 1 4z

z 3z 5z 1 4x

.

HD: Xét hàm số 3 2f (t) t 3t 5t 1 , có 2f '(t) 3t 6t 5 0 , với mọi t R .

Do đó f(t) luôn đồng biến trên R.

Ta có hệ phương trình f (x) 4y

f (y) 4z

f (z) 4x

. Không giảm tính tổng quát, giả sử x max y;z

(hay x y, x z ).

Nếu x > y thì f(x) > f(y) y z f (y) f (z) z x (mâu thuẫn).

Tương tự, nếu x > z thì cũng mâu thuẫn, suy ra x = y = z. Từ đó ta được hệ có hai nghiệm là

(x; y;z) (1;1;1;), (1 2;1 2;1 2), (1 2;1 2;1 2) .

Nhận xét: Ta có hệ f (x) g(y)

f (y) g(z)

f (z) g(x)

, nếu các hàm số f(t) và g(t) cùng tính đơn điệu thì lý

luận như trên, ta suy ra x = y = z.

24. Ví dụ 24

Giải hệ phương trình x y z 1

x y z x y y z1

y z x y z x y

, với x, y, z là các số thực

dương.

HD: Ta chứng minh x y z x y y z

1y z x y z x y

(1).

www.VNMATH.com

Page 102: [Vnmath.com] Ky Yeu Trai He Hv 2012

Một số phương pháp giải Hệ phương trình cho học sinh giỏi Nguyễn Anh Tuấn

101

Thật vậy

(1) 2 2x(y z)(x y) y(y z)(x y) z(y z)(x y)(x y) (y z) (x y)(y z)

y z x

2 22 2 2 2 2 2x z y (x y) yz(y z)

x xy zx y xy z yz x zx z 3y 3xy 3yzy z x

2 22x z y (x y) yz(y z)

xy 2yz 2yy z x

(2).

Mặt khác, áp dụng BĐT Cauchy thì Vt(2) 2 3 2 2 3 2 4

2 3 21 x z y 1 x z z y 1 y z y z x y zy xy xz 2y

2 y z 2 y x 2 z x x z x

2xy 2yz 2y .

Do đó (2) đúng hay (1) đúng. Đẳng thức ở (1) xảy ra khi và chỉ khi x = y = z.

Thay vào hệ ta được hệ có nghiệm duy nhất 1 1 1

(x; y;z) ; ;3 3 3

.

25. Ví dụ 25

Giải hệ phương trình

2 3

2 3

2 3

x (x 1) 2(y x) 1

y (y 1) 2(z y) 1

z (z 1) 2(x z) 1

.

HD: Ta có hệ

3 2 3

3 2 3

3 2 3

x x 2x 2y 1

y y 2y 2z 1

z z 2z 2z 1

hay f (x) g(y)

f (y) g(z)

f (z) g(x)

(1)

(2)

(3)

Trong đó 3 2f (t) t t 2t và 3g(t) 2t 1 .

Ta thấy 2f '(t) 3t 2t 2 0 và 2g '(t) 6t 0 với mọi t R , do đó f(t) và g(t) là hai

hàm số đồng biến trên R.

Gọi (x;y;z) là nghiệm của hệ, không giảm tính tổng quát giải sử x max x; y;z .

Nếu x y z hay x z y thì ta dễ dàng chứng minh được x = y = z.

Do đó hệ đã cho 3 2

x y z

h(x) x x 2x 1

.

Mặt khác, hàm số h(x) liên tục trên R và h(-2) < 0, h(0) > 0, h(1) < 0 và h(2) > 0 nên

h(x) = 0 có cả ba nghiệm trong khoảng (-2;2).

Đặt x = 2cosu với u (0; ) , khi đó sin u 0 và ta được

3 2

x y z 2cosu

8cos u 4cos u 4cos u 1 0

(*).

Xét (*) 3 2sin u(8cos u 4cos u 4cos u 1) 0

sin 4u s in 3u .

www.VNMATH.com

Page 103: [Vnmath.com] Ky Yeu Trai He Hv 2012

Một số phương pháp giải Hệ phương trình cho học sinh giỏi Nguyễn Anh Tuấn

102

Từ đó ta được x y z 2cosu

3 5u ; ;

7 7 7

.

26. Ví dụ 26

Giải hệ phương trình

2 2

2 2

2 2

36x y 60x 25y 0

36y z 60y 25z 0

36z x 60z 25x 0

.

HD: Hệ đã cho tương đương với

2

2

2

2

2

2

60xy

36x 25

60yz

36y 25

60zx

36z 25

.

Dễ thấy hệ có nghiệm (x;y;z) = (0;0;0).

Nếu x.y.z 0 , từ hệ trên ta thấy x, y, z > 0.

Áp dụng BĐT Cauchy, ta được 2 2 2

2 2

60x 60x 60xy x

36x 25 60x2 36x .25

.

Tương tự, ta được y x z y , suy ra x = y = z và ta tìm được x = y = z = 5

6.

Vậy hệ phương trình có hai nghiệm là 5 5 5

(x; y;z) (0;0;0), ; ;6 6 6

.

27. Ví dụ 27

Giải hệ phương trình

3 2

3 2

3 2

x 3z 2z

y 3x 2x

z 3y 2y

.

HD: Do hệ hoán vị vòng quanh, giả sử x max x;y;z .

Hệ đã cho tương đương với

3 2

3 2

3 2

x z 3z 2z z

y x 3x 2x x

z y 3y 2y y

.

Xét hàm số 3 2f (t) 3t 2t t , khi đó hệ trở thành x z f (z)

y x f (x)

z y f (y)

.

Mặt khác 2f '(t) 9t 4t 1 0 , với mọi t R .

Do đó f(t) đồng biến trên R, dẫn đến x = y = z. Thay vào một trong các phương trình của hệ, ta được

3 23x 2x x 0 x(x 1)(3x 1) 0

www.VNMATH.com

Page 104: [Vnmath.com] Ky Yeu Trai He Hv 2012

Một số phương pháp giải Hệ phương trình cho học sinh giỏi Nguyễn Anh Tuấn

103

x 0

x 1

1x

3

.

Vậy hệ phương trình đã cho có ba nghiệm là 1 1 1

(x; y;z) (0;0;0), ( 1; 1; 1), ; ;3 3 3

.

28. Ví dụ 28

Giải hệ phương trình 1 2 2012

1 2 2012

x x ... x 2012

x 8 x 8 ... x 8 6036

.

HD: Đk ix 0,i 1,2012 .

Ta thấy với mọi x R, x 0 thì 2 2 29(x 8) ( x 8) 8( x 1) 0 9(x 8) ( x 8)

3 x 8 x 8

Đẳng thức xảy ra khi x = 1.

Từ đó ta có 2012 2012

i ii 1 i 1

3 x 8 x 8

, đẳng thức xảy ra khi ix 1,i 1,2012 .

Vậy hệ có nghiệm là 1 2 2012(x ; x ;...; x ) (1;1;...;1) .

Nhận xét: - Có thể thay 2012 = n, 6036 = 3n.

- Cách khác, hệ đã cho

nn

i ii ii 1i 1

nn

i ii 1i 1 i i

x 8 x 4nx 8 x 4n

82nx 8 x 2n

x 8 x

.

Mặt khác, theo BĐT Bunhiacopxki

n n 2

2 2i i

i 1 i 1 i i

88n x 8 x . 2 2n 8n

x 8 x

.

Do đó 1 1 2 2 n nx 8 x x 8 x ... x 8 x , mà dễ thấy

y 8 y x 8 x nếu y x 0 .

Bởi vậy 1 2 nx x ... x 1 .

I.5.3. Một số bài tập tương tự

1. Bài 1

Giải hệ phương trình

19 5 2012

19 5 2012

19 5 2012

x y 1890z z

y z 1890x x

z x 1890y y

.

www.VNMATH.com

Page 105: [Vnmath.com] Ky Yeu Trai He Hv 2012

Một số phương pháp giải Hệ phương trình cho học sinh giỏi Nguyễn Anh Tuấn

104

2. Bài 2(Đề tuyển sinh vào trường THPT Chuyên Bắc Giang, năm học

2009 - 2010)

Giải hệ phương trình

2

2

2

y24 7y 2009

xz

24 7z 2009y

x24 7x 2009

z

.

3. Bài 3 Giải các hệ phương trình

a. x x

y y

2 2 3y 3

3 2 3x 2

b. 2 2

2

x y 4xy 6

2x 8 3y 7x

.

4. Bài 4

Giải hệ phương trình 7 3

2 3

2log (2x 3y) log (2x 3y 2)

ln(4x x 1) x 21 9y

.

5. Bài 5 (Đề dự bị tuyển sinh vào trường THPT Chuyên Bắc Giang, năm

học 2011 - 2012)

Giải hệ phương trình

3 2

3 2

3 2

3x 1 y y 2y

3y 1 z z 2z

3z 1 x x 2x

.

6. Bài 6

Giải hệ phương trình

c1 xe

1 y

1

osx-cosy

2y= 2x-x

.

7. Bài 7

Giải các hệ phương trình

a.

y 1x 1

x

z 1y 1

y

x 1z 1

z

b.

201220131 2 3

201220132 3 4

201220132012 1 2

1 2 2012

30 x 4 x x

30 x 4 x x

.....................................

30 x 4 x x

x 0;x 0;...;x 0

.

www.VNMATH.com

Page 106: [Vnmath.com] Ky Yeu Trai He Hv 2012

Một số phương pháp giải Hệ phương trình cho học sinh giỏi Nguyễn Anh Tuấn

105

I.6. Một số phương pháp khác I.6.1. Một số lưu ý Ngoài những Phương pháp thường gặp ở trên, đôi khi ta cũng có những lời giải

khác lạ đối với một số Hệ phương trình. Cũng có thể ta sử dụng kết hợp các Phương pháp ở trên để giải một Hệ phương trình.

I.6.2. Một số ví dụ 1. Ví dụ 1

Giải hệ phương trình 2 2

2 2

3x yx 3

x y

x 3yy 0

x y

.

.

HD: Từ hệ suy ra 2 2

(3x y) (x 3y)ix yi 3

x y

, với i là đơn vị ảo.

2 2 2 2

3(x yi) i(x yi)x yi 3

x y x y

.

Đặt z = x + yi, ta được phương trình ẩn z C là 2

(3 i)z 3 iz 3 z 3

zz

2z 3z 3 i 0 .

Giải phương trình trên, ta được z = 2 + i và z = 1 - i. Từ đó, hệ phương trình đã cho có hai nghiệm là (x;y) = (1;-1), (2;1).

Nhận xét: Ví dụ này là Ví dụ 17 Mục I.2. Phương pháp biến đổi tương đương.

2. Ví dụ 2

Giải hệ phương trình 2 2

2 2

15x 1 12

x y

15y 1 4

x y

(1)

(2)

HD: Từ (2) suy ra 2 2

5yi5yi 4i

x y

(3), với i là đơn vị ảo.

Từ (1) và (3) suy ra 2 2

5(x yi)5(x yi) 12 4i

x y

.

Đặt z = x + yi 2 2

x yi 1

x y z

, ta được 25

5z 12 4i 5z (12 4i)z 5 0z

z 2 i

2 1z i

5 5

.

www.VNMATH.com

Page 107: [Vnmath.com] Ky Yeu Trai He Hv 2012

Một số phương pháp giải Hệ phương trình cho học sinh giỏi Nguyễn Anh Tuấn

106

Từ đó ta được hệ có hai nghiệm là 2 1

(x; y) (2;1), ;5 5

.

Nhận xét: Ví dụ này là Ví dụ 18 Mục I.2. Phương pháp biến đổi tương đương.

3. Ví dụ 3

Giải hệ phương trình 2 2

2 2

x 91 y 2 y

y 91 x 2 x

.

HD: Đk x 2, y 2 .

Dễ thấy (x;y) = (x;2) hoặc (x;y) = (2;y) đều không là nghiệm của hệ đã cho.

Nếu x > 2, y > 2 thì trừ vế với vế của hai phương trình đã cho, ta được 2 2 2 2x 91 y 91 y 2 x 2 y x

2 2

2 2

x y y x(y x)(x y)

y 2 x 2x 91 y 91

2 2

x y 1(x y) x y 0

y 2 x 2x 91 y 91

x y (do 2 2

x y 1x y 0

y 2 x 2x 91 y 91

, với mọi x > 2, y > 2).

Thay x = y vào một trong hai phương trình của hệ, ta được 2 2 2 2x 91 x 2 x x 91 10 x 2 1 x 9

2

2

x 9 x 3(x 3)(x 3)

x 2 1x 91 10

2

x 3 1(x 3) x 3 0

x 2 1x 91 10

2

1 1(x 3) (x 3) 1 0

x 2 1x 91 10

x 3 (do 2

1 1(x 3) 1 0

x 2 1x 91 10

, với mọi x > 2).

Vậy hệ phương trình đã cho có nghiệm là (x;y) = (3;3).

Nhận xét: Do vai trò x, y như nhau nên có thể giả sử x y 2 .

Xét x > y dẫn đến 2 2 2 2x 91 y 91 y 2 y x 2 x , suy ra y > x.

Do đó ta được x = y.

www.VNMATH.com

Page 108: [Vnmath.com] Ky Yeu Trai He Hv 2012

Một số phương pháp giải Hệ phương trình cho học sinh giỏi Nguyễn Anh Tuấn

107

Chương II

MỘT SỐ BÀI TOÁN THI CHỌN HỌC SINH GIỎI II.1. Một số bài toán thi chọn đội tuyển học sinh giỏi quốc gia lớp

12 THPT của tỉnh Bắc Giang và một số tỉnh, thành phố

Trong những kì thi chọn đội tuyển hoặc kiểm tra đội tuyển Toán của tỉnh Bắc

Giang hay của một số tỉnh và thành phố để chuẩn bị cho đội tuyển dự thi chọn học sinh giỏi quốc gia lớp 12 THPT (VMO) cũng có những bài toán giải Hệ phương trình.

Sau đây là một số bài toán.

1. Bài 1 (Chọn đội tuyển HSG quốc gia tỉnh Bắc Giang, năm học 2010 – 2011)

Giải hệ phương trình

22 2

2

5

y y 9x y x xy y 2 6 ln

x x 9

x y 3xy 1 0

.

HD: Biến đổi 2

2 2

2

y y 9x y x xy y 2 6 ln

x x 9

3 2 3 22 6 ln 9 2 6 ln 9 x x x x y y y y (1)

Xét 3 2f t t 2 t 6 ln t t 9 , với t R .

suy ra 2 2

2 2

6 2 2f ' t 3 t 2 3 t

3t 9 t 9

.

Chứng minh f ' t 0 với mọi t R .

Suy ra hàm số đồng biến và liên tục trên R

Do đó: (1) f x f y x y .

Thay vào phương trình còn lại của hệ ta được 6 2x 3x 1 0 (2)

Đặt 2x u u 0 suy ra 3u 3u 1 (3)

Xét 3g u u 3u 1 với u 0 , 2g ' u 3u 3 có g ' u 0 u 1 .

Lập bảng biến thiên, căn cứ vào bảng biến thiên, dễ thấy phương trình (3) có nghiệm duy nhất thuộc khoảng (0; 2).

Đặt u 2cos với 0;2

,

(3) trở thành 1cos3 = = x 2 cos

2 9 9

Vậy hệ có nghiệm (x ; y) 2 cos ; 2 cos ; 2 cos ; 2 cos9 9 9 9

.

www.VNMATH.com

Page 109: [Vnmath.com] Ky Yeu Trai He Hv 2012

Một số phương pháp giải Hệ phương trình cho học sinh giỏi Nguyễn Anh Tuấn

108

2. Bài 2 (Kiểm tra đội tuyển HSG quốc gia tỉnh Bắc Giang, năm học 2010 – 2011)

Giải hệ phương trình

2 2 2 3 1x x y y z z

3

2 3(x y)(y z)(z x)(x y z)

9

.

HD: Đk 0 x, y,z 1 .

Giả sử x = Max (x;y;z) x y;x z .

Nếu x y z thì phương trình sau không thỏa mãn.

Nếu x z y thì T = (x y)(y z)(z x)(x y z) 2 3

9 .

Thật vậy

4T = 4(x y)(y z)(z x)(x y z) 4 (z y)(x z)(x y z)

= 2(z y) ( 3 1)(x z) ( 3 1)(x y z)

31

2(z y) ( 3 1)(x z) ( 3 1)(x y z)27

= 31

2 3x (3 3)y27

31

2 3x27

31(2 3)

27 =

8 3

9, hay T

2 3

9 .

Dấu “=” xảy ra khi và chỉ khi (x;y;z) = 1

(0;0; )3

, thay vào phtrình đầu thấy thỏa mãn.

Tương tự với y = Max (x;y;z), z = Max (x;y;z) ta được hệ có 3 nghiệm là

(x;y;z) = 1

(1;0; )3

, 1

( ;1;0)3

, 1

(0; ;1)3

.

3. Bài 3 (Chọn đội tuyển HSG quốc gia Tp Đà Nẵng, năm học 2010 – 2011)

Giải hệ phương trình

3

3

3

x 3x 12y 50

y 12y 3z 2

z 27x 27z

.

4. Bài 4 (Chọn đội Dự tuyển HSG quốc gia trường THPT Chuyên ĐHSP

Hà Nội, năm học 2010 – 2011)

Giải hệ phương trình 2 2

2 2

2x 3x 4 2y 3y 4 18

x y xy 7x 6y 14 0

( )( ).

5. Bài 5 (Kiểm tra đội Dự tuyển HSG quốc gia trường THPT Chuyên ĐHSP

Hà Nội, năm học 2010 – 2011)

Giải hệ phương trình 4 4

2 2 3

x 2x y y

x y 3

( ).

www.VNMATH.com

Page 110: [Vnmath.com] Ky Yeu Trai He Hv 2012

Một số phương pháp giải Hệ phương trình cho học sinh giỏi Nguyễn Anh Tuấn

109

6. Bài 6 (Chọn đội tuyển HSG quốc gia trường THPT Chuyên ĐHSP Hà Nội,

năm học 2010 – 2011)

Giải hệ phương trình

2 2

2

2 2

x 2 y 3 y 3 x z 2

x 5x 9z 7y 15 3yz

8x 18y 18xy 18yz 84x 72y 24z 176

( ) ( ) ( )( ).

7. Bài 7 (Chọn đội tuyển HSG quốc gia tỉnh Đồng Tháp, năm học 2010 – 2011)

Giải hệ phương trình 2 2

2y x

2

2 2

x 1e

y 1

3 x 2y 6 2 x y 2 1

log ( ) log ( ).

8. Bài 8 (Chọn đội tuyển HSG quốc gia tỉnh Đồng Nai, năm học 2010 – 2011)

Giải hệ phương trình 32 2x 1 2x 1 2y 3 y 2

4x 2 2y 4 6

( ) ( ).

9. Bài 9 (Chọn đội tuyển HSG quốc gia tỉnh Bình Định, năm học 2010 – 2011)

Giải hệ phương trình 2 2

1 y 2 x2

x yx

y x 1 1 3x 3

( )

.

10. Bài 10 (Chọn đội tuyển HSG quốc gia tỉnh Hưng Yên, năm học 2010 – 2011)

Giải hệ phương trình 4 3 3 2 2

3 3

x x y 9y y x x y 9x

x y x 7

( ).

11. Bài 11 (Kiểm tra đội Dự tuyển HSG quốc gia tỉnh Bắc Giang, năm học

2011 – 2012)

Giải hệ phương trình 2 2

1 y 2 x2

x yx

y( x 1 1) 3x 3

.

www.VNMATH.com

Page 111: [Vnmath.com] Ky Yeu Trai He Hv 2012

Một số phương pháp giải Hệ phương trình cho học sinh giỏi Nguyễn Anh Tuấn

110

II.2. Một số bài toán thi chọn học sinh giỏi Quốc gia (VMO)

Thực tế, bài toán giải Hệ phương trình trong kỳ thi chọn học sinh giỏi quốc gia lớp 12 THPT (VMO) được cho là bài toán không khó. Tuy nhiên để làm được việc lớn thì trước hết phải làm tốt việc nhỏ, do đó học sinh muốn đoạt giải từ khuyến khích trở lên thì phải làm tốt bài toán này với mục đích “Bài được cho là dễ thì đạt điểm tối đa,

bài được cho là khó thì ít nhất là không bị mất điểm”. Dù biết vậy nhưng không phải học sinh xuất sắc nào cũng vượt qua được.

1. Bài 1 (1994 – VMO, Bảng A)

Giải hệ phương trình

3 2

3 2

3 2

x 3x 3 ln(x x 1) y

y 3y 3 ln(y y 1) z

z 3z 3 ln(z z 1) x

.

HD: Xét hàm số 3 2f (t) t 3t 3 ln(t t 1) trên R.

Viết lại hệ, ta được f (x) y f (x) y

f (y) z f (y) z

f (z) x f (f (f (x))) x

(1)

(*)

Ta có 2

22

3t t 2f '(t) 3t 0

t t 1

với mọi t R nên f(t) đồng biến trên R.

Từ đó dễ dàng chứng minh được 3 2(1) f (x) x x 2x 3 ln(x x 1) 0 .

Xét hàm 3 2g(x) x 2x 3 ln(x x 1) trên R.

Ta có 2

22

2x 1g '(x) 3x 0

x x 1

với mọi x R nên g(x) đồng biến trên R.

Mà g(1) = 0, nên suy ra (1) x 1 . Từ đó ta được x = y = z = 1.

2. Bài 2 (1994 – VMO, Bảng B)

Giải hệ phương trình 2

2

x 3x ln(2x 1) y

y 3y ln(2y 1) x

.

HD: Đk 1 1

x , y2 2

.

Với đk đó, từ hệ ta có 2 2x 4x ln(2x 1) y 4y ln(2y 1) (1)

Xét hàm số 2f (x) x 4x ln(2x 1) trên khoảng 1

;2

.

Ta có 2

f '(x) 2x 4 02x 1

với mọi 1

x2

nên f(x) là hàm tăng thật sự trên

khoảng 1

;2

.

www.VNMATH.com

Page 112: [Vnmath.com] Ky Yeu Trai He Hv 2012

Một số phương pháp giải Hệ phương trình cho học sinh giỏi Nguyễn Anh Tuấn

111

Do đó (1) x y , thay vào hệ phương trình đã cho, ta được 2x 2x ln(2x 1) 0 .

Xét hàm số 2g(x) x 2x ln(2x 1) trên khoảng 1

;2

.

Suy ra 2

g '(x) 2x 2 02x 1

với mọi 1

x2

nên g(x) là hàm tăng thật sự trên

khoảng 1

;2

.

Từ đó ta thấy x = 0 là nghiệm duy nhất của phương trình g(x) = 0. Thử lại ta được (x;y) = (0;0) là nghiệm duy nhất của hệ đã cho.

3. Bài 3 (1996 – VMO, Bảng A)

Giải hệ phương trình

13x 1 2

x y

17y 1 4 2

x y

.

HD: Dễ thấy, nếu (x;y) là nghiệm của hệ thì x > 0 và y > 0.

Do đó, hệ đã cho

1 2 1 1 2 21

x y x y3x 3x 7y

1 4 2 1 2 21 1x y 7y 3x 7y

(1)

(2)

.

Nhân theo từng vế của (1) và (2), ta được 1 1 8

21xy (x y)(7y 24x)x y 3x 7y

2 27y 38xy 24x 0

(y 6x)(7y 4x) 0

y 6x (do 7y + 4x > 0 với mọi x > 0, y > 0),

thay vào (2) ta được 1 2 11 4 7

1 x213x 21x

22 8 7

y 6x7

.

Vậy hệ phương trình đã cho có nghiệm là 11 4 7 22 8 7

(x; y) ;21 7

.

4. Bài 4 (1999 – VMO, Bảng A)

Giải hệ phương trình 2x y 1 2x y 2x y 1

3 2

(1 4 ).5 1 2

y 4x 1 ln(y 2x) 0

.

HD: Đk 2y 2x 0 .

www.VNMATH.com

Page 113: [Vnmath.com] Ky Yeu Trai He Hv 2012

Một số phương pháp giải Hệ phương trình cho học sinh giỏi Nguyễn Anh Tuấn

112

Đặt 2x y t thì phương trình thứ nhất của hệ trở thành t 1 t t 1(1 4 ).5 1 2

t t 1

t

1 4 1 2

5 5

(1)

Ta thấy, Vt(1) là hàm số nghịch biến, Vp(1) là hàm số đồng biến nên t 1 là nghiệm duy nhất của (1).

Do đó y 1

2x y 1 x2

, thế vào phương trình thứ hai của hệ ta được

3 2y 2y 3 ln(y y 1) 0 (2)

Vt(2) là hàm số đồng biến nên y 1 là nghiệm duy nhất của (2).

Vậy hệ phương trình đã cho có nghiệm là (x; y) (0; 1) .

5. Bài 5 (2001 – VMO, Bảng B)

Giải hệ phương trình 7x y 2x y 5

2x y x y 2

.

HD: Đk min 2x;7x y .

Đặt 7x y t và 2x y z . Khi đó hệ đã cho trở thành t z 5

z x y 2

(1)

(2)

Nhận thấy 2 2t z 5x , kết hợp với (1) suy ra 5 x

z2

.

Thế vào (2), ta được 5 x

x y 2 x 2y 12

(3).

Thế (3) vào (2), ta được 5y 2 y 1 2 5y 2 3 y

2

y 3

y 11y 11 0

11 77y

2

.

Thế vào (3), ta được x 10 77 .

Thử lại ta thấy 15 77

7x y2

;

77 52x y

2

;

9 77x y

2

thỏa mãn hệ

phương trình đã cho.

Vậy hệ phương trình đã cho có nghiệm duy nhất là 11 77

(x; y) (10 77; )2

.

6. Bài 6 (2004 – VMO, Bảng A)

Giải hệ phương trình

3 2

3 2

3 2

x x(y z) 2

y y(z x) 30

z z(x y) 16

.

www.VNMATH.com

Page 114: [Vnmath.com] Ky Yeu Trai He Hv 2012

Một số phương pháp giải Hệ phương trình cho học sinh giỏi Nguyễn Anh Tuấn

113

HD: Biến đổi hệ đã cho tương đương với 2 2 2 2 2 2

2 2 2 2 2 2

2 2 2 2 2 2

x(x y z ) 2xyz 2 x(x y z ) 2xyz 2

y(x y z ) 2xyz 30 (y z)(x y z ) 14

z(x y z ) 2xyz 16 (z x)(x y z ) 14

.

Dễ thấy (x;y;z) = (0;0;0) không là nghiệm của hệ nên ta được 2 2 2 3 2 2

2 2 2 3 2 2 3

x(x y z ) 2xyz 2 2x 2x z xz 2

(y z)(x y z ) 14 2x 6x z 9xz 5z 14

y 2z x y 2z x

3 2 2

3 2 2 3

2x 2x z xz 2

16x 20x z 16xz 5z 0

y 2z x

(1)

Do x,z 0 nên đặt zt

x ,

từ (1) ta được 3 2 25t 16t 20t 16 0 (t 2)(5t 6t 8) 0

t 2 (do 25t 6t 8 0 ).

Từ đó z = 2x, vì vậy ta được hệ

3 2 22x 2x z xz 2 x 1

z 2x y 3

y 2z x z 2

.

Vậy hệ phương trình đã cho có nghiệm là (x; y; z) (1;3;2) .

7. Bài 7 (2004 – VMO, Bảng B)

Giải hệ phương trình 3 2

2 2

x 3xy 49

x 8xy y 8y 17x

(1)

(2)

HD: Cách 1

Nhân hai vế của (2) với 3 rồi cộng vế với vế với (1), ta được 3 2 2 2x 3x 3xy 24xy 3y 24y 51x 49

3 2 2x 3x 3x 1 3y (x 1) 24y(x 1) 48(x 1) 0

2 2(x 1) (x 1) 3y 24y 48 0

2 2(x 1) (x 1) 3(y 4) 0

x 1

x 1

y 4

.

Nếu x 1 thì thay vào (1), ta được y 4

y 4

.

Vậy hệ phương trình đã cho có hai nghiệm là (x; y) ( 1;4);( 1; 4) .

www.VNMATH.com

Page 115: [Vnmath.com] Ky Yeu Trai He Hv 2012

Một số phương pháp giải Hệ phương trình cho học sinh giỏi Nguyễn Anh Tuấn

114

Cách 2

Đặt x + y = z và x – y = t, ta được z t z t

x ; y2 2

.

Từ hệ phương trình đã cho ta có hệ 3 3 3 3

2 2 2 2

z t 98 z 27 t 125

3z 5t 9z 25t 3z 9z 5t 25t

(1')

(2 ')

Nhân hai vế của (2’) với 3 rồi cộng vế với vế với (1’), ta được 3 3(z 3) (t 5) z (t 2) , thế lại vào (1’), suy ra

2t 2t 15 0 (t 3)(t 5) 0

t 5 z 3

t 3 z 5

.

Từ đó, ta được hệ phương trình đã cho có hai nghiệm là (x; y) ( 1;4);( 1; 4) .

8. Bài 8 (2006 – VMO, Bảng A)

Giải hệ phương trình

23

23

23

x 2x 6.log (6 y) x

y 2y 6.log (6 z) y

z 2z 6.log (6 x) z

.

HD: Đk x, y, z < 6.

Hệ đã cho tương đương với

3 2

3 2

3 2

xlog (6 y)

x 2x 6y

log (6 z)y 2y 6

zlog (6 x)

z 2z 6

(1)

(2)

(3)

Xét hàm số 2

xf (x)

x 2x 6

với x < 6, có

2 2

6 xf '(x) 0

(x 2x 6) x 2x 6

với

mọi x < 6, do đó f(x) đồng biến trên khoảng ( ;6) .

Mặt khác, hàm số 3g(x) log (6 x) có 1

g '(x) 0(6 x) ln 3

với mọi x < 6, do đó

g(x) nghịch biến trên khoảng ( ;6) .

Giả sử (x; y;z) là một nghiệm của hệ phương trình.

Không giảm tính tổng quát, giả sử x max x; y;z thì xảy ra hai trường hợp sau

Nếu x y z thì do f(x) là hàm đồng biến nên từ (1), (2) và (3) suy ra

3 3 3log (6 y) log (6 z) log (6 x) .

Từ đó do g(x) là hàm nghịch biến nên x z y , do dó y = z.

www.VNMATH.com

Page 116: [Vnmath.com] Ky Yeu Trai He Hv 2012

Một số phương pháp giải Hệ phương trình cho học sinh giỏi Nguyễn Anh Tuấn

115

Thay y = z vào (1) và (2), ta được x = y = z.

Nếu x z y thì tương tự, ta được x = y = z.

Thay x = y = z vào (1), (2) và (3) ta được f(x) = g(x) (*).

Mà f(x) đồng biến, g(x) nghịch biến trên khoảng ( ;6) nên (*) có nhiều nhất một

nghiệm trên khoảng ( ;6) .

Ta thấy x = 3 là nghiệm của (*).

Vậy hệ phương trình đã cho có nghiệm duy nhất là (x; y;z) (3;3;3) .

9. Bài 9 (2006 – VMO, Bảng B)

Giải hệ phương trình

3 2

3 2

3 2

x 3x 2x 5 y

y 3y 2y 5 z

z 3z 2z 5 x

.

HD: Giả sử x max x; y;z .

Nếu x y z thì ta được

23 2

3 2 2

(x 1) (x 2) 1 0x 3x 2x 5 x

z 3z 2z 5 z (z 1) (z 2) 1 0

x 1

z 1

.

Nếu x z y thì tương tự, ta được x 1

y 1

.

Cả hai trường hợp ta đều có x = y = z = 1.

Thử lại đúng, vậy hệ phương trình đã cho có nghiệm duy nhất là

(x; y;z) (1;1;1) .

10. Bài 10 (2007 – VMO)

Giải hệ phương trình

121- x 2

y+3x

121+ y 6

y+3x

.

HD: Đk x > 0, y > 0, y 3x 0 .

Với đk đó, hệ

1 312 2+ =11-

x yy+3x x

12 6 1 3 121+ +

y+3x y y+3xx y

(1)

(2)

Nhân (1) và (2) theo vế, ta được 2 29 1 12y 6xy 27x 0

y x y+3x

www.VNMATH.com

Page 117: [Vnmath.com] Ky Yeu Trai He Hv 2012

Một số phương pháp giải Hệ phương trình cho học sinh giỏi Nguyễn Anh Tuấn

116

y 3x

y 9x

.

Có y = 3x thỏa mãn.

Thay vào (1), ta được 1 3

+ =1x 3x

và giải ra (x; y) (4 2 3;12 6 3) .

Vậy hệ có nghiệm là (x; y) (4 2 3;12 6 3) .

11. Bài 11 (2009 – VMO)

Giải hệ phương trình 2 2

1 1 2

1 2xy1 2x 1 2y

2x(1 2x) y(1 2y)

9

.

HD: Đk 1 1

0 x ,0 y2 2

(*)

Ta thấy, theo bất đẳng thức Bunhiacopxki 2

2 22 2

1 1 1 11. 1. 2

1 2x 1 2y1 2x 1 2y

(1).

Đẳng thức xảy ra x y .

Mặt khác 2

2 2 2 2

1 1 2 2(x y) (2xy 1)0

1 2x 1 2y 1 2xy (1 2x )(1 2y )(1 2xy)

(do (*))

2 2

1 1 2

1 2x 1 2y 1 2xy

(2).

Đẳng thức xảy ra x y .

Từ (1) và (2), ta được 2 2

1 1 2

1 2xy1 2x 1 2y

.

Đẳng thức xảy ra x y .

Từ đó ta được hệ

9 73x y x y

362

x(1 2x) y(1 2y) 9 739 x y

36

.

Vậy hệ đã cho có hai nghiệm là 9 73 9 73 9 73 9 73

(x; y) ; , ;36 36 36 36

.

Nhận xét: Có thể chứng minh rằng với 1

x, y 0;2

, phương trình thứ nhất của

hệ tương đương với x = y.

www.VNMATH.com

Page 118: [Vnmath.com] Ky Yeu Trai He Hv 2012

Một số phương pháp giải Hệ phương trình cho học sinh giỏi Nguyễn Anh Tuấn

117

Khảo sát hàm số 2 2

1 1 2f (x)

1 2xy1 2x 1 2y

trên đoạn

10;

2

, trong đó y

được coi là một điểm cố định thuộc đoạn 1

0;2

.

12. Bài 12 (2010 – VMO)

Giải hệ phương trình 4 4

3 3 2 2

x y 240

x 2y 3(x 4y ) 4(x 8y)

(1)

(2)

Chú ý: Đó chính là Ví dụ mở đầu.

www.VNMATH.com

Page 119: [Vnmath.com] Ky Yeu Trai He Hv 2012

Một số phương pháp giải Hệ phương trình cho học sinh giỏi Nguyễn Anh Tuấn

118

II.3. Một số bài tập tự làm Sau đây là một số Bài tập tự làm mà chúng ta có thể sử dụng các Phương pháp

đã nói ở trên.

1. Bài 1 Giải các hệ phương trình

1. 2 2

2 2

2x xy y 5x y 2 0

x y x y 4 0

.

2. 3 2

3 2

2x 3x y 5

y 6xy 7

.

3. 2 2 2

x y z 6

xy yz zx 1

x y z 14

.

4. 2 2

2 2

(x y)(x y ) 15

(x y)(x y ) 3

.

5. 2 2 2

2 2 2

1 1 1 51x y z

x y z 4

1 1 1 771x y z

x y z 16

.

2. Bài 2 Giải các hệ phương trình

1. 2

2 2

6x 3xy x y 1

x y 1

.

2. 3 3 2

4 4

x y xy 1

4x y 4x y

.

3. 2 2x y 2xy 8 2

x y 4

.

4. 2

2 2 2

zx x 4

2y 7zx 3x 14

z x 35 y

.

5. 2 2 2

x y z 6

xy yz zx 7

x y z 14

.

3. Bài 3 Giải các hệ phương trình

1.

2

1 1 12

x y z

2 14

xy z

.

2.

2 2

2 2

(x 1)(y 1) 8xy 0

x y 1

x 1 y 1 4

.

3. 2 2

2 2

3 2y1

x y 1 x

4xx y 22

y

.

4. 3 3 2

3 3 2

(x 1) y 3xy

(y 1) x 3x y

.

5. 3 3 3

2 2

1 x y 19x 0

y xy 6x 0

.

4. Bài 4 Giải các hệ phương trình

1. 2 2

xy 3x 2y 16

x y 2x 4y 33

.

2. 3

3

x (2 3y) 8

x(y 2) 6

.

3. 2

4 2

x 3y 9

y 4(2x 3)y 48y 48x 155 0

.

4. 3 2

3 2

2(x 2x y 1) x (y 1)

y 4x 1 ln(y 2x) 0

.

5. 3 2

2 2

x y 2

x xy y y 0

.

6. x x 8 y x y y

x y 5

.

www.VNMATH.com

Page 120: [Vnmath.com] Ky Yeu Trai He Hv 2012

Một số phương pháp giải Hệ phương trình cho học sinh giỏi Nguyễn Anh Tuấn

119

5. Bài 5 Giải các hệ phương trình

1. 4 3 2 2

3 2

x x y x y 1

x y x xy 1

.

2. 4 3 2 2

2

x 2x y x y 2x 9

x 2xy 6(x 1)

.

3.

x2 6y x 2y

y

x x 2y x 3y 2

.

4. 11x y y x 1

7 y x 6y 26x 3

.

5. 2 2

x y x 1

x y y x

2 2 x y

.

6. Bài 6 Giải các hệ phương trình

1. 2 2x 12xy 20y 0

ln(1 x) ln(1 y) x y

.

2.

2

2

1 xyx

2 2 2

32 xy 2

2

(x y 2x) 2x y 4x 1 0

.

3. 2 3 4 6

2

2x y y 2x x

(x 2) y 1 (x 1)

.

4.

2 2

2

x 2 2y 1

(x y) 2

3 9 2( 2y x )

3 2 x y 29

.

5.

3 2 3

3y x

x 3x y 3y 2

x 2 y 1log log (x 3)

y 1 x 2

.

6. 2 2

x x 2 x 4 y 1 y 3 y 5

x y x y 44

.

www.VNMATH.com

Page 121: [Vnmath.com] Ky Yeu Trai He Hv 2012

Một số phương pháp giải Hệ phương trình cho học sinh giỏi Nguyễn Anh Tuấn

120

Phần kết luận

Sau một thời gian làm việc cẩn thận và nghiêm túc, tôi đã hoàn thành Chuyên đề với nhiều dạng Ví dụ và Bài tập về giải Hệ phương trình dành cho học sinh khá giỏi.

Thực tế, những Ví dụ và Bài tập này đã được chúng tôi dạy cho học sinh trong Đội tuyển học sinh giỏi Toán quốc gia của tỉnh Bắc Giang, dạy ôn thi Đại học đồng thời cũng đã dùng dạy ôn thi học sinh giỏi Toán lớp 9 cấp tỉnh và ôn thi vào lớp 10 Chuyên Toán – Tin của các trường THPT Chuyên rất hiệu quả.

Tôi thấy Chuyên đề rất thiết thực đối với các thầy cô dạy Chuyên Toán và các em học sinh của các lớp Chuyên Toán nói riêng đồng thời cũng rất thiết thực cho tất cả các thầy cô luyện thi và các em học sinh ôn thi môn Toán liên quan tới Hệ phương trình.

Tài liệu tham khảo

[1]. Tạp chí Toán học và tuổi trẻ.

[2]. Tuyển tập đề thi Olympic 30 tháng 4. [3]. Đề thi HSG quốc gia (VMO) lớp 12 THPT.

[4]. Kỷ yếu Toán học Trại hè Hùng Vương. [5]. Kỷ yếu Toán học Khu vực Duyên Hải và Đồng Bằng Bắc Bộ.

www.VNMATH.com

Page 122: [Vnmath.com] Ky Yeu Trai He Hv 2012

www.VNMATH.com

Page 123: [Vnmath.com] Ky Yeu Trai He Hv 2012

121

Chuyên đề dãy số nguyên

Trường THPT Chuyên Hạ Long

Các bài toán về dãy số luôn phong phú và đa dạng, đặc biệt là các bài toán về dãy số nguyên.

Trong chuyên đề này chúng tôi xin một số tính chất số học của dãy số nguyên: Tính chia hết, tính nguyên, tính chính phương,…

1. Tính chia hết trong dãy số nguyên

1 .1. Sử dụng số hạng tổng quát của dãy để chứng minh tính chia hết trong dãy số nguyên.

Bài 1: Cho dãy số un xác định như sau

1 2 3

3 1

0, 38, 90, 1,

19 30n n n

u u un n N

u u u

Chứng minh rằng u2011 chia hết cho 2011

Bài giải

Xét phương trình đặc trưng của dãy: x3-19x+30=0 (x-2)(x-3)(x+5)=0 (1)

Khi đó phương trình (1) có 3 nghiệm phân biệt x1=2, x2=3, x3=-5

Do đó un=c1.2n + c2.3

n +c3(-5)n

Từ điều kiện ban đầu u1=0; u2=38; u3=-90, khi đó ta tìm được c1=c2=c3=1.

Vậy un=2n + 3n +(-5)n

Với p là số nguyên tố bất kì, theo định lý Fermat ta có:

2 2(mod );3 3 mod ;( 5) ( 5)(mod )p p pp p p

Do đó ta có : 2 3 5 0(mod )pu p

Với p = 2011 ta có u2011 chia hết cho 2011, bởi vì 2011 là số nguyên tố.

Bài 2

Dãy số un được xác định như sau

u1= 1, u2 = 50

un+1= 4un + 5un-1 – 1975, n = 2,3,…

Chứng minh rằng u1996 chia hết cho 1997.

Bài giải:

Ta thấy công thức truy hồi của dãy là tuyến tính nhưng không thuần nhất, muốn tính số hạng tổng quát của dãy bằng phương pháp sai phân ta cần đưa về công thức truy hồi tuyến tính thuần nhất.

www.VNMATH.com

Page 124: [Vnmath.com] Ky Yeu Trai He Hv 2012

122

Muốn vậy ta đặt un = vn + c và ta cần phải xác định c để có công thức truy hồi vn+1 = 4vn + 5vn-1. Nhờ công thức này ta tính được số hạng tổng quát của dãy vn, từ đó suy ra số hạng tổng quát

của un.

Ta có

vn+1 + c = 4(vn+c) + 5(vn-1 + c) - 1975

vn+1 = 4vn + 5vn-1 + 8c – 1975.

Do đó c = 8

1975. Khi đó dãy vn được xác định như sau

v1=8

1919 ;v2=

8

1575

vn+1 = 4vn + 5vn-1, n = 2, 3, 4…

Phương trình đặc trưng : x2- 4x – 5 = 0 có 2 nghiệm x1 = 5, x2 = -1 nên

vn = c15n + c2(-1)n.

Cho n = 1 và n = 2 ta có:

5c1 – c2 = 8

1919

25c1 + c2 = 8

1575

Giải hệ pt này ta được c1 = 120

1747 ; c2 =

12

2005 , do đó

vn = 120

1747.5n +

12

2005.(-1)n.

Vậy

un =120

1747.5n +

12

2005.(-1)n +

8

1975

Cho nên

u1996 = 120

1747.51996 +

24

9935

= 120

496755.1747 1996 .

Do 1997 là số nguyên tố nên 51996 1 (mod 1997)

=> -1745.51996 + 49675 0 (mod 1997)

Lại có

51996 = 25998 1 (mod 3) => -1747.51996 + 49675 -1747 + 49675 = 0 (mod 3)

51996 = 25998 1 (mod 8)

=> -1747.51996 + 49675 -1747 + 49675 = 0 (mod 8)

Rõ ràng -1747.51996 + 49675 0 (mod 5) mà (3,5,8) = 1

www.VNMATH.com

Page 125: [Vnmath.com] Ky Yeu Trai He Hv 2012

123

=> (-1747.51996 + 49675) 120.

Mặt khác (120, 1997) = 1 nên 1997120

496755.1747 1996

hay u1996 1997

Bài 3 (60th Annual William Lowell Putnam Competition 1999)

Dãy số nguyên un được xác định như sau

u1 = 1, u2 = 2, u3 = 24

un = .4,.

.8.6

32

2213

21

nuu

uuuu

nn

nnnn

Chứng minh rằng un n, n = 1,2,3… Bài giải Công thức truy hồi của dãy không tuyến tính nhưng bằng cách biến đổi thích hợp ta sẽ dẫn đến một dãy số khác có công thức truy hồi tuyến tính. Ta làm như sau: Từ công thức truy hồi của dãy ta có:

.8.6.

86

3

2

2

1

32

223.1

1

n

n

n

n

nn

nnn

n

n

u

u

u

u

uu

uuu

u

u

Đặt nv = .1n

n

u

u Do vậy ta có dãy nv xác định như sau:

v2 = 2, v3 = 12

vn = 6vn-1 – 8vn-2, n4.

Phương trình đặc trưng x2 – 6x + 8 = 0 có 2 nghiệm x1 = 4, x2 = 2 nên

vn = c14n + c22

n.

Cho n = 2 và n = 3 ta có:

16c1 + 4c2 = 2

64c1 + 8c2 = 12.

Giải hệ pt này ta được c1 = 4

1,c2 =

2

1.

Do đó

vn = 4 n-1 – 2n-1.

Vậy

un = vn.vn-1… v2

= (4n-1 – 2n-1).(4n-2 - 2n-2)….(4 - 2)

= 2n-1.2n-2…2.(2n-1 - 1).(2n-2 - 1) …(2 – 1)

Theo định lý Fermat nhỏ với mọi số nguyên tố lẻ p ta luôn có (2p-1 – 1) p

và nếu p = 2 thì (2p-1 – 1) p.

Vậy (2p-1 – 1) p, với mọi số nguyên tố p và nếu s là bội của p – 1 thì

(2s – 1) p.

Với n nguyên dương tuỳ ý ta luôn có n = kiPpippp skk

ss ,1,....,. 22

11 .

www.VNMATH.com

Page 126: [Vnmath.com] Ky Yeu Trai He Hv 2012

124

=> n ,,...,, 11

211

11

ss pppnp

hay

n=p1.d1 = 11

1221 ..... s

s dpdp với d1 > d2 > … > ds1 1

Suy ra

n – d1 = d1(p1-1) (p1-1) => (2n-d1- 1)p1.

n – d2 = d2(21p -1) (p1-1) => (2n-d2- 1)p1.

n - .1)12()1()1( 11

1111 pppdd sdns

ss

Vậy (2n-d1 – 1)( 2n-d

2 – 1) . . .(2nds1 – 1) .1

1sp Do đó 1

1s

n pu .

Chứng minh tương tự ta cũng có , 2,sin iu p j i M . Do đó

un chia hết cho n, n = 1, 2, 3, . . .

Bài 4

Dãy anđược xác định theo công thức:

a1 = 2

an = 3an-1 + 2n3 – 9n2 + 9n, n 2

Chứng minh rằng với mọi số nguyên tố p thì dãy các tổng tương ứng

a1 + a2 + . . . + ap-1 chia hết cho p.

Bài giải

Tương tự bài 2 ta cũng biến đổi dãy đã cho để đưa về một sãy khác mà việc tính số hạng tổng quát của nó đơn giản hơn như sau:

Trước hết ta có:

an + n3 = 3an-1 + 3(n3 – 3n2 + 3n – 1)

= 3(an-1 + (n-1)3).

Đặt un = an + n3 thì ( un ) là cấp số nhân công bội 3 với u1 = a1 + 1 = 3.

Suy ra

un = 3n-1.u1 = 3n => an = 3n – n3.

Vậy

a1 + a2 + . . . + ap-1 = 31 + 32 + . . . + 3p-1 – (13+23+…+(p - 1)3)

= ).)1(...21(2

33 333

pp

- Nếu p = 2 hiển nhiên a1 2.

- Nếu p 3 thì p lẻ nên p – 1 chẵn.

Khi đó ta có (k3 + (p-k)3 )p, k = 1, 2 . . ., 2

1p.

=> (13 + 23 + . . . +(p-1)3)p.

www.VNMATH.com

Page 127: [Vnmath.com] Ky Yeu Trai He Hv 2012

125

Mặt khác theo định lý Fermat nhỏ (3p – 3) p nếu p > 3 còn nếu p = 3 hiển nhiên (33 – 3) 3. Do đó

(3p – 3)p, .3, pPp

Hơn nữa (3p – 3)2; (2,p) = 1 nên pp

2

33 . Do vậy ta có đpcm.

Bài 5

Cho λ là nghiệm dương của phương trình t2 – 1998t – 1 = 0. Dãy x0, x 1, x2…

được xác định như sau:

x0 = 1

xn+1 = [λxn], n 0

Tìm số dư của x1998 cho 1998.

Bài giải

Bài toán này lại có một cách tìm số hạng tổng quát hết sức đặc biệt, đó là dựa vào tính chất về phần nguyên của một số rồi biến đổi bất đẳng thức ta sẽ đạt được kết quả mong muốn.

Theo giả thiết ta có:

λ2 - 1998 λ – 1 = 0 => λ2 = 1998λ + 1

=> λ = 1998 + λ

1.

Suy ra λxn = 1998xn + λ

nx

Dễ thấy xn là dãy số nguyên dương, do đó ta có:

[λxn] = [1998xn +λ

nx ]

= 1998xn +

λ

nx.

Vì xn = [xn-1] nên xn < λ xn-1 <xn + 1 (do λ là số vô tỉ nên λxn-1 Z)

=>λ

nx<xn-1 <

λnx

+λ1

Hay

xn-1 - λ1

nx< xn-1.

Dễ thấy phương trình t2 – 1998t – 1 = 0 luôn có 2 nghiệm trái dấu mà tổng của chúng bằng 1998 nên nghiệm dương λ >1,

=> xn-1 – 1 <xn-1 - λ1

www.VNMATH.com

Page 128: [Vnmath.com] Ky Yeu Trai He Hv 2012

126

Vậy ta có xn-1 <λ

nx<xn-1 =>

λ

nx= xn-1 – 1.

Do đó xn+1 = [λxn] = 1998xn + xn-1 – 1. Suy ra

xn+1 xn-1 – 1 (mod 1998)

xn-3 – 2 (mod 1998)

xn-2k-1 – (k + 1) (mod 1998).

Vậy x1998x1 – 999 (mod 1998) trong đó x1 = [λ] = 1998 cho nên

x1998 999 (mod 1998).

Bài 6(Bulg. MO 1999)

Cho dãy số nguyên an thoả

(n-1)an+1 = (n + 1)an – 2(n – 1), n 1

Giả sử a1999 chia hết cho 2000, tìm số n nhỏ nhất (n 2) sao cho an chia hết cho 2000.

Bài giải

Trước hết ta xác định công thức tổng quát của an.

Ta thấy rằng dãy an = 2n – 2 thoả mãn công thức truy hồi cho trong bài và một suy nghĩ rất tự nhiên là đặt an = bn + 2n – 2. Công việc còn lại là xác định công thức tổng quát của bn. Khi đó ta có:

(n - 1)(bn+1 + 2n) = (n + 1)(bn + 2n – 2) – 2(n – 1)

=> (n - 1)bn+1 = (n + 1)bn.

Từ đó suy ra b3 = 3b2; b4 = 6b2, b5 = 10b2 và tổng quát lên ta được

bn = .2,2

)1(2

nb

nn

Công thức này dễ dàng chứng minh được bằng phương pháp quy nạp.

Vậy ta có:

an = 2 (n – 1) + 2,2

)1(

nc

nn (c = const).

Với n = 2 ta có c = a2 – 2 là số nguyên.

Theo giả thiết a1999 2000 nên ta có:

2(1999 – 1) + 02

1998.1999c (mod 2000)

=> c 4(mod 2000).

Do vậy

an 2000 2(n – 1) + 2n(n – 1) 0(mod 2000)

(n – 1)(n + 1) 0(mod 1000)

www.VNMATH.com

Page 129: [Vnmath.com] Ky Yeu Trai He Hv 2012

127

Ta thấy (n – 1)(n + 1) 8 n lẻ và (n – 1)(n + 1) 125 khi n – 1 hoặc n + 1

chia hết cho 125.

Do đó số tự nhiên n 2 nhỏ nhất thoả mãn là 249.

1.2 .Sử dụng phương pháp quy nạp toán học

Bài 1

Dãy số un] xác định như sau:

u0 = u1 = 1

un+1= un-1.un+1 + 1, n = 1, 2, . . .

Chứng minh rằng: u2008 3 (mod 4).

Bài giải

Trước hết ta tính một số số hạng đầu tiên của dãy

1; 1; 2; 3; 7;22; 155; 3411; 528706, . . .

Câu hỏi đặt ra là phải chăng ngoại trừ 2 số hạng đầu các số còn lại của dãy đều chia 4 dư 2 hoặc dư 3?

Ta thấy u2, u5, u8 chia 4 dư 2 còn u3, u4, u6, u7 chia 4 dư 3. Ta sẽ chứng minh:

un 2 (mod 4) nếu n = 3k + 2

un 3 (mod 4) nếu n 3k + 2.

bằng phương pháp quy nạp theo k.

Ta thấy mệnh đề đúng với k = 0,1,2.

Giả sử mệnh đề đúng đến k = p, ta chứng minh nó cũng đúng đến k = p + 1.

Thật vậy

Ta có u3(p+1)+2 = u3p+5 = u3p+4.u3p+3 + 1

u3(p+1)+1 = u3p+4 = u3p+3.u3p+2 + 1

u3(p+1)+3 = u3p+6 = u3p+5.u3p+4 + 1

Theo giả thiết quy nạp :u3p+4 3(mod4)

u3p+3 3(mod4)

u3p+2 2(mod4)

u3p+5 2(mod4),

nên

u3(p+1)+2 2(mod4)

u3(p+1)+1 3(mod4)

u3(p+1)+3 3(mod4).

Vậy un = 3(mod4) nếu n chia hết cho 3 hoặc chia 3 dư 1.

Do 2008 chia 3 dư 1 nên u2008 3 (mod 4) (đpcm).

Bài 2

www.VNMATH.com

Page 130: [Vnmath.com] Ky Yeu Trai He Hv 2012

128

Chứng minh rằng: Tồn tại vô số số nguyên dương n thoả mãn (2n +1) n.

Bài giải

Thoạt nhìn bài toán này không thấy xuất hiện bóng dáng của dãy số nhưng ta có thể tạo ra một dãy số có công thức truy hồi 1 2 1kx

kx , k 0 và chứng minh mỗi số hạng của nó đều chia hết

cho số liền trước nó.

Xét dãy xk xác định như sau:

x0 = 1

xk+1 = 2xk + 1, k 0.

Ta sẽ chứng minh xk+1, xk, kN bằng phương pháp quy nạp.

- Hiển nhiên mệnh đề đúng với k = 0, k = 1.

- Giả sử (2xk + 1xk, ta sẽ chứng minh (2xk+1 + 1)xk + 1.

Thật vậy:

Do xk + 1 =2xk + 1xk mà 2xk + 1 là số lẻ nên xk + 1 =mkxk (với mk lẻ).

Khi đó ta có (2xk+1 + 1) = kk xm2 + 1 = (( 12 xk km) + 1) ( kx2 + 1).

hay ( 12 xk + 1)xk+1

Mặt khác ta thấy x0 < x1 < x2 < . . . nên dãy xk gồm vô số nguyên dương phân biệt do đó tồn tại vô số số nguyên dương n thoả mãn (2n + 1)n.

Bài 3

Chứng minh rằng: Với một số nguyên dương a > 2 tồn tại vô số số nguyên dương n thoả mãn (an -1) n.

Bài giải

Ta sẽ chứng minh dãy xk xác định như sau:

x0 = 1

xk+1 = axk + 1, k 0.

thoả mãn điều kiện (axk – 1) xk, kN.

Thật vậy:

- Với k = 0, k = 1 mệnh đề đúng.

- Giả sử (axk – 1) xk tức là xk+1 = mkxk (mk N). Khi đó

(axk+1 – 1) = ( kk xma - 1)

= ((axk)mk – 1)(axk – 1)

hay (axk+1 – 1)xk+1

Vì dãy xk là dãy tăng vô hạn nên tồn tại vô số số nguyên dương n thoả mãn (an -1)n.

Bài 4 (Balkan MO 1990).

Dãy un xác định bởi

u0 = 1, u2 = 3

www.VNMATH.com

Page 131: [Vnmath.com] Ky Yeu Trai He Hv 2012

129

un = (n + 1)un-1 – nun-2n 3.

Tìm các số hạng của dãy chia hết cho 11.

Bài giải

Ta sẽ chứng minh un11,n 10 bằng quy nạp.

Thật vậy: Bằng tính toán trực tiếp ta có 11 số hạng đầu của dãy như sau

1, 2, 9, 33, 153, 873, 5913, 46233, 409113, 4037913, 43954713.

Vậy u10 và u11 đều chia hết cho 11.

Giả sử un-2 và un-1 chia hết cho 11, n 12.

Khi đó do un = (n + 1)un-1- nun-2 nên un cũng chia hết cho 11.

Vậy un11,n 10.

1.3. Sử dụng tính tuần hoàn của dãy số

Bài 1: Cho dãy số un xác định như sau

1 2

2 1

5, 11; 1,2,3,...

2 3n n n

u un

u u u

Chứng minh rằng: u2007 chia hết cho 11

Bài giải

Ta thấy 1 2 3 4 5 65; 11; 7; 19; 59; 61u u u u u u

Nên ta có: 1 2 3 4 55(mod11); 0(mod11); 7(mod11); 3(mod11); 7(mod11)u u u u u

Vì vậy nếu gọi rn là phần dư của un trong phép chia cho 11, n=1,2,3,…thì từ trên ta suy ra

1 2 3 4 55; 0; 7; 3; 7r r r r r . Ta sẽ chứng minh với mọi k=1,2,3…

5 1 5 2 5 3 5 4 5 55; 0; 7; 3; 7(1)k k k k kr r r r r . Để làm điều này ta sẽ chứng minh bằng qui nạp toán

học. Theo nhận xét ở trên thì k=1 đúng, giả sử (1) đúng với n=p, tức là:

5 1 5 2 5 3 5 4 5 55; 0; 7; 3; 7p p p p pr r r r r

Xét khi k=p+1. Ta có: 5 65 1 1 ppu u , theo cách xác định của dãy ta có

5 6 5 5 5 45 1 1 2 3p p ppu u u u (2)

Theo gt qui nạp ta có: 5 5 5 5 5 57 7(mod11) 7 11 ; ( )p p pr r u l l Z

5 4 5 4 5 43 3(mod11) 3 11 ; ( )p p pr r u s s Z , thay lại vào (2) ta có

5 6 2(7 11 ) 3(3 11 ) 5 11(2 3 ); ,pu l s l s k l Z , khi đó suy ra 5 6 5 65(mod11) 5p pu r

Bằng cách lập luận hoàn tương tự suy ra 5( 1) 2 5( 1) 3 5( 1) 4 5( 1) 50; 7; 3; 7p p p pr r r r

Nên nhận xét (1) đúng với k=p+1

Ta có 2007 = 401.5+2, do đó 2007 0r , điều đó có nghĩa là u2007 chia hết cho 11

Bài 2

www.VNMATH.com

Page 132: [Vnmath.com] Ky Yeu Trai He Hv 2012

130

Gọi a là nghiệm dương lớn nhất của phương trình x3 – 3x2 + 1 = 0. Xét dãy xxn xác định theo công thức sau:

xn = [an] + [an+1], n = 1,2, 3, …

Tìm số dư trong phép chia x2007 cho 17.

Bài giải:

Đặt f(x) = x3 – 3x2 + 1. Ta có

f(-1) = -3 < 0

f( )21 = 0

81

f( )2

1= 0

8

3

f(1) = -1 < 0

f(2 2 ) = 16 2 - 23 < 0

f(3) = 1 > 0.

Do hàm f(x) liên tục nên phương trình x3 – 3x2 + 1 = 0 có 3 nghiệm phân biệt. x1(-

1;2

1),x2( 1;

2

1), x3 = a(2 2 ; 3).

Đặt un = Nnaxx nnn ,21 . Khi đó un là nghiệm của phương trình sai phân tuyến tính thuần nhất có phương trình đặc trưng là x3 – 3x2 + 1 = 0.

Do đó ta có:

un+3 - 3un+2 + 0. un+1 + un = 0

un+3 = 3un+2 - un

Trong đó

u0 = 3

u1 = x1+ x2 + x3 = 3

u2 = 23

22

21 xxx = (x1+ x2 + x3)

2 – 2(x1x2 + x2x3+ x3x1) = 9

(sử dụng định lý Viet ta có x1+ x2 + x3 = 3, x1x2 + x2x3+ x3x1= 0)

Do đó un Z , .Nn

Vì x1+x2 = 3 - x3 > 0 nên x2 > -x1= 01 x

=> .0211

2

12 nnnn xxxxx

Ta lại có

x1(-1;2

1), x2(

2

1;1)

nên

1)8(19 23

23

22

2121 xxxxxx nn (do ).223 x

www.VNMATH.com

Page 133: [Vnmath.com] Ky Yeu Trai He Hv 2012

131

Vậy

)1()1()( 2121nn

nnn

nn xxuxxua

[an] = un – 1.

Cho nên xn = un – 1 + un+1 – 1 = un+1 + un – 2

x2007 = u2008+ u2007 - 2.

Gọi rn là số dư trong phép chia un cho 17. Khi đó rn0,1, 2, …, 16,,n 0 tức là dãy rn bị chặn.

Ngoài ra từ công thức truy hồi của dãy un ta cũng có:

rn+3 3 rn+2 - rn (mod 17) suy ra dãy rn tuần hoàn. Bằng tính toán trực tiếp ta có:

r0 = 3, r1 = 3, r2 = 9, r3 = 7, r4 = 1, r5 = 11, r6 = 9

r7 = 9, r8 = 16, r9 = 5, r10 = 6, r11 = 2, r12 = 1

r13 = 14, r14 = 6, r15 = 0, r16 = 3, r17 = 3, r18 = 9.

Dễ kiểm tra rn có chu kì 16, nghĩa là un un+16 (mod 17), nN . Vậy

u2007 = u125.16+7 u7 9 (mod 17)

u2008 = u125.16+8 u8 16 (mod 17)

x2007 9 + 16 – 2 6 (mod 17)

Vậy x2007 chia 17 dư 6.

Bài 3 (BMO 1998):Cho a1 = 19, a2 = 98. Với n 1, an +2 là số dư của phép chia an + an +1 cho 100. Tìm số dư trong phép chia 2

199822

21 ... aaa cho 8.

Bài giải

Gọi rn là số dư trong phép chia an cho 4.

Theo giả thiết ta có an + an +1 an +2 (mod 100) nên

an + an +1 an +2 (mod 4)

rn + rn +1 rn +2(mod 4).

Mặt khác rn0,1,2,3 tức là dãy rn bị chặn do đó dãy rn tuần hoàn.

Ta tính được

r1 = 3, r2 = 2, r3 = 1, r4 = 3, r5 = 0, r6 = 3, r7 = 3, r8 = 2, r9 = 1.

Dễ kiểm tra rn có chu kì 6, nghĩa là rn = rn+6, 1.

Lại có 4)().)((22 nnnnnnnn raDorarara nên an, rn đồng tính chẵn lẻ

=> (an+ rn) 2

=> ( 2na - 2

nr )8

hay 2na 2

nr (mod 8). Vậy 21998

22

21

21998

22

21 ...... rrraaa (mod 8).

Mà )...(333... 26

22

21

21998

22

21 rrrrrr = 333(9 + 4 + 1 + 9 + 0 + 9) 8

Do đó 21998

22

21 ... aaa chia hết cho 8.

www.VNMATH.com

Page 134: [Vnmath.com] Ky Yeu Trai He Hv 2012

132

Bài 4 (VMO 1995)

Cho dãy an,n = 0,1,2,… xác định bởi a0 = 1, a1 = 3 và

an+ 2 = an+ 2 + 9an nếu n chẵn

an+ 2 = an+ 2 + 5an nếu n lẻ.

Chứng minh rằng: 22000

1995i

i

a

chia hết cho 20.

Bài giải

Từ công thức truy hồi của dãy ta dễ thấy an+2 (an+1+ an )(mod 4).

Gọi rn là số dư của phép chia an cho 4, khi đó 0 rn 3.

Hơn nữa rn+2 (rn+1+ rn) (mod 4) nên tương tự bài 2 dãy rn tuần hoàn với chu kì 6.

Ta có an rn (mod 4) nên 22nn ra (mod 4). Vì vậy

2000

1995

222000

1995 iii

i

ra (1 + 9 + 0 + 9 + 9 + 4) 0 (mod 4)

tức là

2000

1995

2 .20i

ia Mặt khác với k 0 ta có: a2k+4 (a2k+3 - a2k+2 )(mod 5) ;

a2k+3 - a2k+2 (mod 5) => a2k+4 -2a2k+2 (mod 5), 232

222 kk aa (mod5)

=> 232

222

222

242 4 kkkk aaaa (mod 5) => ( 5)2

322

42 kk aa

Vậy

2000

1995

22000

21999

21998

21997

21996

21995

2 .5)()()(i

i aaaaaaa Vì (4,5) = 1 nên 22000

1995i

i

a

20 (đpcm).

4.Tính nguyên và tính chính phương

Bài 1: Cho dãy số un xác định như sau

1 2

1

1; 2; 1, 2,3,...

( 1) 2n n n

u un

u u u

Đặt: 2 2 2 21 2 3( 1)( 1)( 1)...( 1) 1; 1, 2,3,....n nS u u u u n

Chứng minh rằng: Sn là số chính phương với mọi n=1, 2, 3,.. Bài giải Ta đi chứng minh qui nạp: Sn=(un+1-1)2 với mọi n=1, 2, 3,… (1) Ta thấy n = 1 thỏa mãn (1) Giả sử (1) đúng với n=k 1 , tức là Sk=(uk+1-1)2 (2) Ta có

2 2 2 21 1 2 1 1( 1)( 1)...( 1) 1 ( 1)( 1) 1(3)k k k kS u u u S u

Thay (2) vào (3) ta có:

2 2 2 2 2

1 1 1 1 1 1

2 2 2 2 2 2 2 21 1 1 1 1 1 1 1

2 21 1

[( 1) 1]( 1) 1 [( 1) 2 1]( 1) 1

( 1) 2 1 1 1 ( 1) 2 1

( 1 )

k k k k k k

k k k k k k k k

k k

S u u u u u

u u u u u u u u

u u

Mặt khác:

www.VNMATH.com

Page 135: [Vnmath.com] Ky Yeu Trai He Hv 2012

133

2

1 1 1 1 1 1 2

21 2

1 ( 1) 1 ( 1) 2 1 2

( 1)

k k k k k k k

k k

u u u u u u u

S u

Nên ta có điều phải chứng minh.

Bài 2:Cho dãy số an xác định bởi công thức:

1,1 21 aa

1,212 naaa nnn .

Chứng minh rằng: n 2, 2n+1 - 7 21na là số chính phương.

Bài giải Đặt .72 22n

nn au Ta tính được 5,1,3,1 6543 aaaa . Do đó:

,)1)1(2(172 221

31 au

,)1)1(2(972 222

42 au

,)13.2(2572 223

53 au

,)3)1.(2(172 224

64 au

.)1)5.(2(12172 225

75 au

Ta sẽ chứng minh 21 )2( nnn aau bằng quy nạp.

Giả sử 21

22 )2(72 nnnn aaa . Ta sẽ chứng minh:

.)2(72 212

21

31

nnn

nn aaau

Thật vậy: Theo giả thiết quy nạp ta có:

21

21

22 4472 nnnnnn aaaaa

nnnnn aaaa 1

221

2 4842

nnnnn aaaa 1

221 22 .

Do đó 211

221

321

3 7)2(272 nnnnnn

n aaaaaa

nnnnn aaaau 122

11 816 211 )4( nnn aau .

Thay 212 nnn aaa ta được: 212

22111 )2())(2( nnnnnn aaaaau

Vậy un = (2an+1 +an)2 mà an là dãy số nguyên nên un là số chính phương.

Bài 3 Cho dãy an xác định như sau:

4,1 21 aa

.2,4 12 naaa nnn

Chứng minh rằng: 12 1 nnaa là số chính phương.

Bài giải

Xét phương trình đặc trưng 0142 xx có 2 nghiệm

32,32 21 xx

www.VNMATH.com

Page 136: [Vnmath.com] Ky Yeu Trai He Hv 2012

134

nên số hạng tổng quát của dãy được xác định như sau:

.)32()32( 21nn

n cca

Lần lượt cho n = 1, n = 2 ta có:

1)32()32( 21 cc

.4)32()32( 22

22 cc

Giải hệ phương trình này ta được : 32

1,

32

121 cc

Do đó ta có .32

)32()32( nn

na

Áp dụng công thức khai triển nhị thức Niutơn ta có:

3)32( nnn BA

,3)32( nnn BA

ở đây An và Bn là các số nguyên và hiển nhiên ta có an = Bn

Ta lại có

,3)2()32()3)(32()32( 1nnnnnn

n BABABA

.3)2()32()3)(32()32( 1nnnnnn

n BABABA

.232

)32()32( 11

1 nn

nn

n BAa

Vậy 1)2(212 1 nnnnn BABaa = 222 342 nnnnn BABBA = (An + Bn)2.

Do đó 12 1 nnaa là số chính phương.

Bài 4 (Balkan MO 2002).

Cho dãy an xác định như sau:

a1 = 20, a2 = 30

an+1 = 3an - an-1, n 3.

Tìm n sao cho 15 1 nn aa là số chính phương.

Bài giải

Ta sẽ chứng minh 15 1 nn aa = 21)( nn aa + 501 bằng phương pháp quy nạp.

- Với n = 1 ta có 5a2a1 + 1= 3001 = (20 +30)2 + 501 = (a1 + a2)2 + 501.

Vậy mệnh đề đúng với n = 1.

- Giả sử 501)(15 211 nnnn aaaa . Ta cần chứng minh

.501)(15 211 nnnn aaaa

www.VNMATH.com

Page 137: [Vnmath.com] Ky Yeu Trai He Hv 2012

135

Thật vậy:

Vì .2513 111 nnnnnn aaaaanaa

Xét )(5)15()15( 1111 nnnnnnn aaaaaaa = ))(2( 1111 nnnnn aaaaa

= 1 1 1 1(( ) ( ))(( ) ( ))n n n n n n n na a a a a a a a = ( .)() 21

21 nnnn aaaa

Thay ( 5005) 12

1 nnnn aaaa ; ta có đpcm.

Ta thấy m2<m2 + 501 < (m + 1)2m> 250 do đó m2 + 501 không thể là số chính phương.

Từ công thức truy hồi của dãy ta có )(2 11 nnnn aaaa đồng thời vì a2 > a1 nên dễ dàng suy ra

an+1 > an, n.

Lại có a3 = 70, a4 = 180 4,36021 naaa nnn do đó 501)( 21 nn aa không là số chính

phương, .4n

- Với n = 1 thì 300115 12 aa không là số chính phương.

- Với n = 2 thì 1050115 23 aa không là số chính phương.

- Với n = 3 thì 234 2516300115 aa .

Vậy với n = 3 thì thoả mãn 15 1 nn aa là số chính phương.

BÀI TẬP:

Bài 1: Cho dãy số un; n = 1, 2, 3, …là dãy các số nguyên dương, (un) là dãy số tăng; U2n=un+n ; un là số chính phương thì n là số chính phương Tính u2012

Bài 2: Cho dãy số (xn) xác định như sau: x1=1; x2=2011; và xn+2=4022xn+1-xn

Chứng minh rằng: 2012 1

2012

x là số chính phương

Bài 3: Dãy yn được xác định bởi:

.,24)54(

1

12

21

Nnkyyky

yy

nnn

Xác định tất cả các số nguyên k sao cho mọi số hạng của dãy đều là số chính phương.

Bài 4: (IMO 1989-LOnglisted Problems) Chứng minh rằng: Dãy an cho bởi [ 2na n ]nN

chứa vô số số chính phương.

Bài 5: Dãy an được xác định bởi

.,98

5

12

10

Nnaaa

aa

nnn

Chứng minh rằng 6

1nalà số chính phương.

Bài 6: Dãy an được xác định bởi

.,

1*

1

1

Nnaaa

a

nnn

www.VNMATH.com

Page 138: [Vnmath.com] Ky Yeu Trai He Hv 2012

136

Chứng minh rằng: an là số chính phương khi và chỉ khi n = 2k + k – 2, k N*.

Bài 7: Cho a, b Z, dãyan được xác định như sau

.0,33

22,,

123

210

naaaa

ababaaa

nnnn

Tìm a, b để an là số chính phương, n 1996.

Bài 8: (China TST for IMO 2004) Dãy cn được xác định bởi

*

12

210

,200843

2005,0,1

Nnccc

ccc

nnn

Đặt an = 5 (cn+2 – cn)(502 – cn-1 – cn-2) + 4n.2004.501, n2.

Hỏi an có phải là số chính phương, n3 hay không?

Bài 9: Dãy an được xác định bởi

.,22

),,(,,

123

210

Nnaaaa

Zcbacabaaa

nnnn

Tìm k Z sao cho 4anan+1 + k là số chính phương, Nn

Bài 10: Dãy an được xác định bởi

.,24)54(

1

12

10

Nnkaaka

aa

nnn

Tìm tất cả k sao cho an là số chính phương, Nn .

Tài liệu tham khảo

1. Một số lớp các bài toán về tính chất số học của dãy số, Ngô Thị Hải

2. Tuyển tập các đề thi 30.4

3. Tuyển tập các đề thi quốc gia môn toán từ năm 2006 đến nay

4. Các trang Web: Mathscope.org

www.VNMATH.com

Page 139: [Vnmath.com] Ky Yeu Trai He Hv 2012

www.VNMATH.com

Page 140: [Vnmath.com] Ky Yeu Trai He Hv 2012

Tô Toan trường THPT chuyên Hoa ng Văn Thu - Tỉnh Hòa Bình

137

CÁC PHƯƠNG PHÁP CƠ BẢN KHI GIẢI HỆ PHƯƠNG TRÌNH

Nguyễn Ngọc Xuân, Bùi Thị Hương Trường THPT Chuyên Hoàng Văn Thụ, Hoà Bình

1. Phương pháp thế trong giải hệ phương trình Ví dụ 1. Giải các hệ phương trình sau:

a. 2 2

2 2

1 1

3 1

y x x y

x y

b.

2

2

1 4

1 2

x y y x y

x y x y

c. 3 3

2 2

8 2

3 3 1

x x y y

x y

d.

2 2

2

21

xyx y

x y

x y x y

e. 2 2

2

1 1 3 4 1

1

x y x y x x

xy x x

f.

3 2

2 2

3 49

8 8 17

x x y

x xy y y x

g.4 3 2 2

2

2 2 9

2 6 6

x x y x y x

x xy x

h.

3 2 3

3

4 12 9 6 5

xy x y

x x x y y

Hướng dẫn:

a. 2 2

2 2

1 1 (1)

3 1 (2)

y x x y

x y

Biến đối phương trình (1) ta có: 1 1 01

x yx y xy

xy

nếu 2 14 1

2x y x x y .

nếu 4 213 1 0x y y

y phương trình vô nghiệm.

b.

2

2

1 4 (1)

1 2 (2)

x y y x y

x y x y

Từ (1) rút được: 2 1 4x y x y , thế vào (2) ta có:

2 04 2 3 0

3

yy x y x y y y x y

x y

nếu y=0 thay vào hệ có 2 1x vô nghiệm.

nếu 3 3x y y x , thế vào (1) ta có: 2 1 22 0

2 5

x yx x

x y

www.VNMATH.com

Page 141: [Vnmath.com] Ky Yeu Trai He Hv 2012

Tô Toan trường THPT chuyên Hoa ng Văn Thu - Tỉnh Hòa Bình

138

c. 3 3 3 3

2 2 2 2

8 2 8 2 (1)

3 3 1 3 6 (2)

x x y y x y x y

x y x y

nhân vế với vế của (1) và (2) cho nhau ta có:

3 3 2 2 3 2 26 3 8 2 2 2 24 0x y x y x y x x y xy

0

2 4 3 0 4

3

x

x x y x y x y

x y

nếu x=0 thay vào (2) có 2 2y vô ghiệm.

nếu x=-4y thay vào (2) có 2

6 64

6 13 1313 6 6

413 13

y xy

y x

nếu x=3y thay vào (2) có 2 1 31

1 3

y xy

y x

d.

2 2

2

21 (1)

(2)

xyx y

x y

x y x y

ĐK: 0x y

Biến đổi (1) ta có

2 2 2 2 2

2 2 1 0 1 1 0 1x y x y x y

x y x y x yx y x y

.

Thế 1y x vào (2) ta có 2 1 02 0

2 3

x yx x

x y

e. 2 2

2

1 1 3 4 1 (1)

1 (2)

x y x y x x

xy x x

Từ (2) rút ra được 1 1 1x y x x thay vào (2) ta có

1 1 1 3 1 1 0x x x x y x x 1 1 1 3 1 0x x x x y x

21 1 1 1 3 1 0x x x x x y x

221 1 1 1 3 1 0x x x x x x

3 2

1

1 2 2 4 0 0

2

x

x x x x x

x

www.VNMATH.com

Page 142: [Vnmath.com] Ky Yeu Trai He Hv 2012

Tô Toan trường THPT chuyên Hoa ng Văn Thu - Tỉnh Hòa Bình

139

nếu x=1 thay vào (1) có y=-1. nếu x=0 thay vào (1) có 0=1 vô ghiệm.

nếu x=-2 thay vào (1) có 5

2y .

f. 3 2

2 2

3 49 (1)

8 8 17 (2)

x x y

x xy y y x

Biến đổi (1) ta có: 2 23 49x x y , nhận thấy x=0 không là nghiệm của hệ nên ta có

22 49

3 3

xy

x , thay vào (2) ta có

22 3 2 249

8 8 17 2 51 49 24 243 3

xx xy y x x x x y xy

x

224 1 1 2 49 49xy x x x x 21 24 2 49 49 0x xy x x

2

1 4

2 49 49(3)

24

x y

x xy

x

thay (3) vào (2) có 22 2

2(1 ) 2 49 49 2 49 49

17 824 24

x x x x xx x

x x

2 21 4 4 49 0x x x 1 4x y .

g. 4 3 2 2

2

2 2 9 (1)

2 6 6 (2)

x x y x y x

x xy x

Từ (2) rút ra được 2 6 6

2

x xxy

thế vào (1) có phương trình: 3 0

4 04

xx x

x

x=0 thay vào (2) có 0=6 vô ghiệm.

x=-4 thay vào (2) có y=17

4.

h. 3 2 3

3 (1)

4 12 9 6 5 (2)

xy x y

x x x y y

Từ (1) rút ra được 1 3y x x thay vào (2) ta có:

3 33 34 1 3 3 6 0 4 1 3 1 6 0y x x y y x y x y

3 33 3 24 1 3 3 0 4 1 3 1 0y x y x y x y x

21 2 2 0y x y x

1

2 1

y x

y x

www.VNMATH.com

Page 143: [Vnmath.com] Ky Yeu Trai He Hv 2012

Tô Toan trường THPT chuyên Hoa ng Văn Thu - Tỉnh Hòa Bình

140

nếu y=-x-1 thay vào (1) có 2 3 4 0x x vô nghiệm.

nếu 2 1y x thay vào (1) có 22 3 1 0x x .

2. Phương pháp đặt ẩn phụ trong giải hệ phương trình Ví dụ 2. Giải các hệ phương trình sau:

a.2 1 1

3 2 1 0

x y x y

x y

b.

2

2

1 3 0

51 0

x x y

x yx

c.2 2 2

1 7

1 13

xy x y

x y xy y

d.

2 3 2

4 2

5

45

1 24

x y x y xy xy

x y xy x

e.

2 22

3 854 4

3

1 132

3

xy x yx y

xx y

Hướng dẫn:

a. ta có Biến đổi 3 2 1 2 1 2x y x y x y , đặt 2 1 0

0

x y u

x y v

và đưa về hệ

2 2

1

2

u v

u v

rút u=v+1 thế vào phương trình còn lại được phương trình 22 2 1 0v v .

b. ĐK: 0x . iến đổi về hệ

2

2

31

51

x yx

x yx

, đặt 1

u x y

vx

và đưa về hệ 2 2

3 1

5 1

u v

u v

. Rút

u=3v-1 thế vào phương trình cò lại được 2

1 24 6 2 0 1 1

2 2

v uv v

v u

từ đó tìm được

1 3; 1; ; 2;

2 2x y

.

c. Thay y=0 vào hệ có 1 0

1 0

x

vô nghiệm, từ đó suy ra y=0 khôg là nghiệm của hệ. iến đối hệ về

dạng

www.VNMATH.com

Page 144: [Vnmath.com] Ky Yeu Trai He Hv 2012

Tô Toan trường THPT chuyên Hoa ng Văn Thu - Tỉnh Hòa Bình

141

22

17

113

xx

y y

xx

y y

, đặt 1

uy và đưa về hệ

2 2

7

13

x u xu

x u uv

đây là hệ đối xứng với u và x. Giải

hệ này ta tìm được 1; 3;1 ; 1;3 ; 3;1 ; 1;

3x u x y

.

d.

2 3 2 2 2

24 2 2

5 5

4 45 5

1 24 4

x y x y xy xy x y xy xy x y

x y xy x x y xy

Đặt 2x y u

xy v

đưa về hệ

2

5

45

4

u v uv

u v

Rút 25

4v u và thế vào phương trình cò lại ta tìm được

3 25 5 50 0

4 4 4u u u u v 3 3

5 25; ;

4 16x y

e. B iến đổi hệ về dạng

2 22 22 2

22

3 85 3 854 4 3

3 3

1 13 1 132

3 3

1 673

3

1 13

3

xy x y x y x yx y x y

x x y x yx y x y

x y x yx y

x y x yx y

Đặt 1

,x y u x y vx y

đưa về hệ

2 2 673

313

3

u v

u v

Rút 13

3u v và thế vào phương trình còn lại.

3. Phươg pháp đánh giá trong giải hệ phương trình Ví dụ 3. Giải các hệ phương trình sau:

www.VNMATH.com

Page 145: [Vnmath.com] Ky Yeu Trai He Hv 2012

Tô Toan trường THPT chuyên Hoa ng Văn Thu - Tỉnh Hòa Bình

142

a. 3

3

2 6 2

3 4

x y y

y x x

b.

2

23

2

23

2

2 92

2 9

xyx x y

x xxy

y y xy y

c.2 2 2

3 2

2 0

2 3 6 12 13 0

x y x y

x x y x

Hướng dẫn:

a. Biến đổi đưa về hệ

2

2

2 1 2 (1)

2 2 1 2 (2)

y x x

x y y

nếu x>2 từ (1) có y<2 từ (2) có x<2 (vô lí) nếu x<2 từ (1) có y>2 từ (2) có x>2 (vô lí) Vậy x=y=2.

b. Ta có 23

23

2 9 2

2 9 2

x x

y y

từ hệ Ban đầu

22 2

22

x xy x yx y xy

y xy y x

. từ đó tìm được

nghiệm x=y=1 và x=y=0.

c. 2 2 2

3 2

2 0 (1)

2 3 6 12 13 0 (2)

x y x y

x x y x

Từ (1) rút ra được 22 2

02

1 1 1 1

xxy

x y y

Từ (2) có 23 22 3 12 13 6 6 2 7 1 0x x x y x x 1x y .

4. một số phươg pháp khác trong giải hệ phương trình a. Sử dụng tính chất nghiệm của phương trình Bậc hai ví dụ 4. Giải các hệ phương trình sau:

a. 2

2 2

6 3 1

1

x xy x y

x y

.

2

2 2

5 4 4

5 4 16 8 16 0

y x x

y x xy x y

Hướng dẫn:

a. 2

2 2

6 3 1 (1)

1 (2)

x xy x y

x y

Biến đổi phương trình (1): 26 1 3 1 0x x y y coi đây là phương trình ẩ x, y là tham số, tìm

được:

1 3 41; 0 ;

2 5 5

1 2

3 3

yx y x y x

x y

www.VNMATH.com

Page 146: [Vnmath.com] Ky Yeu Trai He Hv 2012

Tô Toan trường THPT chuyên Hoa ng Văn Thu - Tỉnh Hòa Bình

143

B. 2

2 2

5 4 4 (1)

5 4 16 8 16 0 (2)

y x x

y x xy x y

Coi (2) là phương trình ẩ y, x là tham số tìm được 5 4

4

y x

y x

sau đó thế vào (1).

B. Đưa về phươg trình đẳng cấp Ví dụ 5. giải các hệ phương trình sau:

a.4 4

3 3 2

4 4

1

x y x y

x y xy

b.

4 2 2 4

2 2

91

7

x x y y

x xy y

Hướng dẫn:

a. 4 4

3 3 2

4 4

1

x y x y

x y xy

nhân chéo hai vế của hai phươg trình trong hệ và đưa về phươg trình đẳng cấp:

4 4 3 3 2 2 24 4 4 3 0x y x y xy x y xy x xy y

3 3

0 1

0 1

1

1 33 ;

25 25

x y

y x

x y x y

x y x y

b. Biến đổi đưa về hệ

4 2 2 4

22 2

91

49

x x y y

x xy y

nhân chéo hai phương trình trong hệ đưa về phương trình:

. 4 3 2 2 3 46 26 32 26 6 0x x y x y xy y

BÀI TẬP LUYỆN TẬP TỔNG HỢP

GIẢI CÁC HỆ PHƯƠNG TRÌNH SAU

1.

www.VNMATH.com

Page 147: [Vnmath.com] Ky Yeu Trai He Hv 2012

Tô Toan trường THPT chuyên Hoa ng Văn Thu - Tỉnh Hòa Bình

144

G: dễ thấy y = 0 không thỏa mãn hệ phương trình.

Với 0y đặt 2 1

; x

a y x by

2

2 2

4 4 1 1

( 2) 1 (2 ) 1 3 3

1 23 1 2 0

2 5

a b b a a x y

a b a a b x y

x yx x x x

x y

Kết luận: Hệ phương trình có 2 nghiệm (1;2) và (-2;5)

2.

G: Trừ vế với vế hai phương trình của hệ ta được:

2

2

5 0

(1)

5 0 (2)

x y x x y

x y

x x y

4

3 2

(1) 5 6 0

1 6 0

1 1

2 2

x x

x x x x

x y

x y

(2) 2

3

x

x5y

=> 065

.52

34

x

xx

6525

24 x

xx

Xét 4 4 32 2 2

25 25 25 6253.

2 2 4x x

x x x

Mặt khác, từ (2) 65 x

=> 6525

24 x

xx

=>trường hợp này phương trình vô nghiệm Kết luận: Hệ phương trình có 2 nghiệm (1;1) và (-2;-2)

3.

www.VNMATH.com

Page 148: [Vnmath.com] Ky Yeu Trai He Hv 2012

Tô Toan trường THPT chuyên Hoa ng Văn Thu - Tỉnh Hòa Bình

145

G: ĐK

3

4

2 1 27 1

2 1 2

x y x

x y

3 0

5 4 0

x y

x y

Đặt 3 0 ; 5 4 0x y a x y b

22 2 2

2 2

2

b=5-a5

12 2 35 12 2 5 35

2 12(5 ) (25 10 ) 35

0 52 0 ( 2) 0

2 3

253 0 7) 5 4 25 75

7

a b

b a b b a a

a a a a

a ba a a a

a b

xx y

x yy

3 4 1 )

5 4 9 1

x y x

x y y

Kết luận: Hệ phương trình có 2 nghiệm và (1;1)

4.

G: ĐK 4 0

2 0

x y

x y

Đặt 4 0 ; 2 0x y a x y b

Có Kết luận Hệ phương trình có 1 nghiệm (4;-7)

5.

25 75;

7 7

2 22 2

2 2 2

44

32 3 4 (*)2

2

(*) 2 3 (16 8 ) 4 2 10 12 0

( 1)( 6) 0

1 <=> 3 4 9 4

6( ) 2 1 7

a ba b

b ab b ab

b b b b b b

b b

b a x y x

b L x y y

www.VNMATH.com

Page 149: [Vnmath.com] Ky Yeu Trai He Hv 2012

Tô Toan trường THPT chuyên Hoa ng Văn Thu - Tỉnh Hòa Bình

146

G: ĐK 0x y

Đặt 0 2

2

a bx

x y a

x y b a by

11

135

4.4)22.(8

2

2222

aba

a

baba

aab

aba

11

135

352

22

2

22

1 15. 3 1 13 0a a

a a

0101

.31

.52

2

aa

aa

Đặt ta

a 1

có 21

222

aat

2

12 4

8 6 20 05 1

( )8 o

t a aat t

t a vô na

6.

www.VNMATH.com

Page 150: [Vnmath.com] Ky Yeu Trai He Hv 2012

Tô Toan trường THPT chuyên Hoa ng Văn Thu - Tỉnh Hòa Bình

147

G: Đặt ; 0x y a x y b có

Kết luận Hệ phương trình có 2 nghiệm 5 1

;2 2

và 35 29

;8 8

7.

2 2 22 2 2

2 2 2 2

16 1 1716 1 17

4 1 7 2 16 8 1 49 4 28

x y yx y y

xy y x x y xy y x xy

2 2 2

2 2

2 2

2 2

2

2

2

16 17 1

4 2 7 1

32 4 2 28 14 4 0

16 2 1 14 7 2 0

4 26 32 0

( 8 )( ) 0

8 (1)

(2)

(1) 32 16 7 1

(2) 4 5 1 0

4 5 1 0

( 1)(4 1) 0

1 1

1

4

x y y

xy x y

y x xy y x

y x xy y x

x xy y

x y x y

x y

x y

y y y

x x

x x

x x

x y

x y

1

4

Kết luận Hệ phương trình có 4 nghiệm 1 1 1 11;1 ; 1; ; ; ; ;1

4 4 4 4

2 222

66

4( 1) 3

( 1) 39

32

( 3)(4 3) 0 38

4

3553

3 82) +) 42 1 29

82 8

baba

a aab

ab

a aba

xxx y x y

x yx yy y

(vô nghiệm)

www.VNMATH.com

Page 151: [Vnmath.com] Ky Yeu Trai He Hv 2012

Tô Toan trường THPT chuyên Hoa ng Văn Thu - Tỉnh Hòa Bình

148

8.

G: ĐK 22

23

x

2 2

2

2 2 2 2

2 2

2 2 2 2

2 2

2 2 2 2

2

( 1 )( 1 ) 1 (1)

4 2 22 3 8 (2)

(1) ( 1 )( 1 ) 1

1 1 1'

(1) ( 1 )( 1 ) 1

( 1 ) 1 2 '

1' 2 ' 1 1 1 1

4 2 22 3 8

4 2 8 22 3

x x y y

x x y

x x y y x x

y y x x

x x y y y y

x x y y

y y x x x x y y

x y

x x y

x

24 4

2 4 22 3 164. ( 2)( 2)

2 2 22 3 4

2

4 32 (*)

2 2 22 3 4

x x

x xx x

x x

x

xx x

Nếu 1x thì 12 x

3

1

4322

1

3

1

4322

3

9432225322

3

4

22

4322

x

x

xx

xx

=> VT < 1 => Vô nghiệm CMTT với 1x => Vô nghiệm Xét thấy PT (*) có nghiệm 1x Kết luận Hệ phương trình có 2 nghiệm (2;-2) và (-1;1)

9. 2 2

2

4 5 2

x y

x y x y xy

www.VNMATH.com

Page 152: [Vnmath.com] Ky Yeu Trai He Hv 2012

Tô Toan trường THPT chuyên Hoa ng Văn Thu - Tỉnh Hòa Bình

149

G:

10.

G:

044422

096422

224

yxyx

yyxx

Cộng vế với vế ta có: 4 2 2 2 2

2 2 2

4 6 9 2 2 4 44 0

( 1) 36 1 6....

x x y y x y x y

x y x y

2 4 2

6 4 2

5 4 3 2

19 ( 5 12)

5 12 19 0

( 1)( 6 6 18 19) 0

1 3 2

a a a a

a a a a

a a a a a a

a x y

Kết luận Hệ phương trình có 1 nghiệm (3;2)

11.

3

4

2 1 27 1

2 1 2

x y x

x y

2 2

2

2 2

2 2 2 2

2 2 2

2

4 4 5(2 ) 4 0

(2 ) 5(2 ) 4 0

(2 )(2 4 ) 0

4 42)

22

4 5 0 4 5 (2 ) (2 ) 0

4 10 5 4 4 0

10 14 4 0

x xy y x y xy xy

x y x y xy xy

x y xy x y xy

x xy y xyx y xy

y xx y

x xy y x x x x

x x x x x

xx x

2 2 2 2

2 2 2 2 2

2

1

2

5

2) 4 4 16 4 20 0

2

4 20 (2 ) (2 ) 0 4 40 20 4 4 0

22 8 6

2525 44 4 022 8 6

25

x

x y xyx xy y xy x xy y

y x

x x x x x x x x x

xx x

x

www.VNMATH.com

Page 153: [Vnmath.com] Ky Yeu Trai He Hv 2012

Tô Toan trường THPT chuyên Hoa ng Văn Thu - Tỉnh Hòa Bình

150

G: ĐK 2, 1x y

4

2

2 ( 2) 1

( 2) 1

x y

x y

Thế vào pt (1) ta được: 2 3

3 2

2

2 ( 2) 27

2 4 31

2 19 (2 )( 6)

x x x

x x x x

x x x x

Đặt 02 ax ta có: 2 4 2

6 4 2

5 4 3 2

5 4 3 2

19 ( 5 12)

5 12 19 0

( 1)( 6 6 18 19) 0

1 0

6 6 18 19 0

1 3 2

a a a a

a a a a

a a a a a a

a

a a a a a

a x y

Kết luận Hệ phương trình có 1 nghiệm (3;2)

12.

Biến đổi phương trình thứ 2 của hệ ta được:

2 22 2

2 2

2 2

( )( 2 ) 3 2 4

9( 1) 9( 1)3 ( 1) 2 2 4 0

4 4

3( 1) 10 250

2 4

(2 3 3) ( 5) 0

4( 1)( 2 4) 0

1 (1)

2 4 (2)

x y x y x y

y yx x y y y

y y yx

x y y

x y x y

x y

x y

Biến đổi phương trình thứ nhất của hệ ta được: 2

2 2 2

( )2 1 2 1

2( 2 )

2 2 2 2 (2 1)(2 1) (*)4

x yx y

x y xyx y x y

Từ (1)(*)4

1681

4

)41(3424

222 yxxyxyxy

Đặt 4

34 axy ta có:

www.VNMATH.com

Page 154: [Vnmath.com] Ky Yeu Trai He Hv 2012

Tô Toan trường THPT chuyên Hoa ng Văn Thu - Tỉnh Hòa Bình

151

)(523

523

)(3

1

0)116)(3)(1(

0334264

1624216

4641124

16)1(216

)1(4

4

)1()3(4

4

)1(324

2

234

2432

2422

2

La

a

La

a

aaaa

aaaa

aaaaaa

a

aaa

a

aa

Với

1)1(4

1

1

14

yy

yx

yx

xy

2

1

2

10)12(

0144

2

2

xyy

yy

Với 523)1(45234 yyxy

0)523(44 2 yy (VNo)

Kết luận Hệ phương trình có 1 nghiệm 1 1

;2 2

13. 2 2

2 2

3 8 5

8 3 13

x y y y

x y x y

G: Đặt 2 23 0 ; 8 0x y a y x b có

www.VNMATH.com

Page 155: [Vnmath.com] Ky Yeu Trai He Hv 2012

Tô Toan trường THPT chuyên Hoa ng Văn Thu - Tỉnh Hòa Bình

152

2 2 2

22

2 2 22 2 2

22 2

22 2

5 5 2 3( 2)( 3) 0

3 213 25 10 2 13

93 9

) 9 36 72 36 0 9 36 1 038 4

(9 ) 72 36 0

6( 6 3)( 6 15) 0

a b a b b ab b

b aa b b b

xx x y

x x x x xy x

x x

x xx x x x

2

22

22 4

4 2 4 3 3 2 2

2

3 03 6

6 15 0( )

43 4

) 38 9

16 8 72 81

8 72 65 0 5 5 25 17 85 137 65 0

( 5) 5 ( 5) 17 ( 5) 13( 5) 0 ( 5)( 1)( 4 13)

o

xx x VN

xx x y

y xx x x

x x x x x x x x x x

x x x x x x x x x x x

1

05

x

x

14. 2

2

1 1 2 1 8

2 1 2 13

x y x

y y x x

G: ĐK 2

1x

Đặt 012 ax có 12 2 ax

12

8

012

8)1(2222 ayay

ayya

ayay

aya

Đặt tya ta có vay

)(12243

8

123

822 VNott

tv

vt

vt

15.

www.VNMATH.com

Page 156: [Vnmath.com] Ky Yeu Trai He Hv 2012

Tô Toan trường THPT chuyên Hoa ng Văn Thu - Tỉnh Hòa Bình

153

G: Đk:

23 1 2 4 2x x x x

+ Nếu 1x thì 3 1 2x và 2 1 3x VT

24 2 1 4x x x x x

Có 1 0

34 3

1

x xVT

x

VT VP x

iáo viên Toán. Trường THPT chuyên Hoàng Văn Thụ

www.VNMATH.com

Page 157: [Vnmath.com] Ky Yeu Trai He Hv 2012

www.VNMATH.com

Page 158: [Vnmath.com] Ky Yeu Trai He Hv 2012

154

SỬ DỤNG DÃY SỐ TRONG MỘT SỐ DẠNG BÀI TẬP VỀ PHƯƠNG TRÌNH HÀM TRÊN TẬP Z

TRẦN NGỌC THẮNG

GV THPT CHUYÊN VĨNH PHÚC

Phương trình hàm là một chuyên đề quan trọng trong chương trình chuyên toán trong các trường THPT chuyên. Các bài toán liên quan đến phương trình hàm cũng là những bài tập thương gặp trong các kì thi học sinh giỏi quốc gia, khu vực và quốc tế. Khi giải các bài tập về phương trình hàm thông thường chúng ta phải dự đoán được các tính chất cơ bản của hàm số cần tìm để từ đó đề ra phương hướng giải quyết, các hướng giải quyết thường là không tự nhiên và có nhiều kĩ thuật khó . Do đó việc giảng dạy chuyên đề phương trình hàm cho học sinh giỏi thường gặp rất nhiều khó khăn. Khó khăn ở đây theo chúng tôi thứ nhất là việc chọn lọc tài liệu, nội dung kiến thức, mức độ kiến thức và thời gian giảng dạy cho học sinh như thế nào là thích hợp? Thứ hai là việc tiếp thu của học sinh, cách học phần này cũng như thời gian học phần này không nhiều. Đối với chuyên đề phương trình hàm để học sinh học tốt, cũng như phát huy được năng lực của bản thân giáo viên cần trang bị cho các em các phương pháp giải cơ bản nhất và thường xuyên cập nhật các dạng toán mới. Với chuyên đề "sử dụng dãy số trong một số dạng bài tập về phương trình hàm trên tập ¡ " chúng tôi mong muốn được góp một phần nhỏ vào việc giảng dạy phần phương trình hàm đạt kết quả tốt nhất. Chuyên đề này được chia ra làm hai phần chính

Phần A: Trình bày một số kiến thức cơ bản về dãy số được sử dụng trong bài viết

Phần B: Phần bài tập, phần này được chia làm 2 dạng toán hay gặp (phương trình hàm Cauchy, phương pháp sử dụng dãy lặp) mà cách giải thường sử dụng dãy số.

A. PHẦN LÍ THUYẾT 1. Giới hạn của dãy số 1.1. Định nghĩa. Cho dãy số thực 1n n

u

. Số a được gọi là giới hạn của dãy

1n nu

nếu với mọi số 0 cho trước tồn tại một số tự nhiên N (phụ thuộc

vào ) sao cho ,nu a n N . Khi đó ta kí hiệu lim nn

u a

hoặc

lim nu a . Một dãy số có giới hạn còn được gọi là hội tụ. 1.2. Các tính chất của dãy hội tụ. 1.2.1. Mọi dãy hội tụ đều có giới hạn duy nhất

www.VNMATH.com

Page 159: [Vnmath.com] Ky Yeu Trai He Hv 2012

155

1.2.2. Mọi dãy con của dãy hội tụ là một dãy hội tụ 1.2.3. Mọi dãy hội tụ là dãy bị chặn 1.2.4. Cho hai dãy hội tụ 1n n

u

và 1n nv

. Giả sử tồn tại số tự nhiên N sao

cho ,n nu v n N . Khi đó lim limn nu v 1.2.5. Nguyên lý kẹp. Giả sử các dãy 1n n

u

, 1n nv

, 1

w n n thỏa mãn các

bất đẳng thức wn n nu v với mọi n , đồng thời lim lim wn nu a . Khi đó lim nv a . 1.2.6. Dãy số 1n n

u

được gọi là một dãy cơ bản hay dãy Cauchy nếu với mọi

0 cho trước tồn tại một số tự nhiên N (phụ thuộc vào ) sao cho , ,m nu u m n N .

Dãy số thực 1n nu

hội tụ khi và chỉ khi nó là dãy cơ bản.

1.2.7. Một dãy tăng (tăng thực sự) và bị chặn trên thì hội tụ. Một dãy giảm (giảm thực sự) và bị chặn dưới thì hội tụ. 2. Cận trên, cận dưới 2.1. Định nghĩa. Cho tập A ¡ . 2.1.1. Số a được gọi là một cận trên của tập A nếu ,x a x A . Cận trên nhỏ nhất (nếu có) của tập hợp A được gọi là cận trên đúng của A và kí hiệu là sup A . 2.1.2. Số a được gọi là một cận dưới của tập A nếu ,x a x A . Cận dưới lớn nhất (nếu có) của tập hợp A được gọi là cận dưới đúng của A và kí hiệu là infA . 2.2. Tính chất 2.2.1. Mọi tập khác rỗng và bị chặn đều tồn tại cận trên đúng và cận dưới đúng. 2.2.2. Nếu tập A có cận trên đúng thì tồn tại một dãy 1n n

u

trong A sao cho

lim supnu A . Nếu tập A có cận dưới đúng thì tồn tại một dãy 1n nu

trong

A sao cho lim infnu A . 3. Tập trù mật 3.1. Định nghĩa. Cho hai tập ,A B ¡ . Tập hợp A được gọi là trù mật trong tập hợp B nếu với mọi ,x y B , x y đều tồn tại a A sao cho x a y . 3.2. Tập hợp ¤ trù mật trong ¡ .Từ đó suy ra với mỗi x ¡ luôn tồn tại một dãy hữu tỷ nx hội tụ về x.

3.3. Tập hợp ,2n

mm n

¢ ¥ trù mật trong ¡ . Từ đó suy ra với mỗi x ¡

luôn tồn tại một dãy hữu tỷ 2n

m

hội tụ về x.

www.VNMATH.com

Page 160: [Vnmath.com] Ky Yeu Trai He Hv 2012

156

4. Hàm số bị chặn, hàm số đơn điệu 4.1. Hàm số f xác định trên tập D được gọi là bị chặn trên nếu tồn tại số M sao cho ,f x M x D .

4.2. Hàm số f xác định trên tập D được gọi là bị chặn dưới nếu tồn tại số m sao cho ,f x m x D .

4.3. Hàm số f xác định trên tập D được gọi là bị chặn nếu nó vừa bị chặn trên và bị chặn dưới. 4.4. Hàm số f xác định trên tập D được gọi là tăng (tăng nghặt) trên D nếu với mọi 1 2 1 2, ,x x D x x thì 1 2f x f x ( 1 2f x f x )

4.5. Hàm số f xác định trên tập D được gọi là giảm (giảm nghặt) trên D nếu với mọi 1 2 1 2, ,x x D x x thì 1 2f x f x ( 1 2f x f x )

4.6. Hàm số tăng hoặc giảm được gọi chung là hàm đơn điệu. 5. Hàm số liên tục 5.1. Định nghĩa. Cho hàm số y f x xác định trên khoảng K và 0x K .

Hàm số y f x được gọi là liên tục tại 0x nếu 0

0limx x

f x f x

.

Hàm số y f x được gọi là liên tục trên một khoảng nếu nó liên tục tại mọi

điểm của khoảng đó. Hàm số y f x được gọi là liên tục trên đoạn ;a b nếu nó liên tục trên

khoảng ;a b và lim , limx a x b

f x f a f x f b

.

5.2. Định lí giá trị trung gian của hàm số liên tục 5.2.1. Định lí. Giả sử hàm số f liên tục trên đoạn ;a b . Nếu f a f b thì

với mỗi số thực M nằm giữa f a và f b , tồn tại ít nhất một điểm

;c a b sao cho f c M .

5.2.2. Hệ quả. Nếu hàm số f liên tục trên đoạn ;a b và . 0f a f b thì

tồn tại ít nhất một điểm ;c a b sao cho 0f c . Từ đó ta có nếu hàm số f

liên tục trên đoạn ;a b và . 0f a f b thì phương trình 0f x có ít

nhất một nghiệm thuộc khoảng ;a b .

5.2.3. Định lí. Nếu hàm số f liên tục trên đoạn ;a b thì nó đạt giá trị lớn nhất

và giá trị nhỏ nhất trên đoạn đó. Từ định lí này suy ra nếu hàm số f liên tục trên đoạn ;a b thì nó bị chặn trên

đoạn đó. 5.2.4. Định lí. Nếu một hàm số liên tục và đơn ánh trên khoảng ;a b thì nó

đơn điệu nghiêm ngặt trên khoảng đó. Chứng minh.

www.VNMATH.com

Page 161: [Vnmath.com] Ky Yeu Trai He Hv 2012

157

Ta chứng minh bằng phản chứng. Giả sử hàm số f không đơn điệu nghiêm ngặt khi đó tồn tại a x y z b sao cho f x f y f z hoặc

f x f y f z .

+) Nếu f x f y f z thì theo định lí giá trị trung gian ta có với mỗi

max ,f x f z M f y tồn tại x c y d z sao cho

M f c f d c d vô lý. Vậy trường hợp này không xảy ra.

+) Nếu f x f y f z chứng minh tương tự ta suy ra mâu thuẫn.

Vậy định lí được chứng minh. B. PHẦN BÀI TẬP I. PHƯƠNG TRÌNH HÀM CAUCHY Cho hàm số :f ¡ ¡ . Khi đó hương trình

; ,f x y f x f y x y ¡

được gọi là phương trình hàm Cauchy, dễ thấy hàm số f thỏa mãn phương trình hàm Cauchy có những tính chất cơ bản: (1) 0 0f và f là hàm số lẻ trên ¡ ;

(2) 1 ,f n nf n ¢ ;

(3) 1 ,f r rf r ¤ .

Cùng với các tính chất cơ bản trên và thêm điều kiện của hàm số f hoặc điều kiện về tập xác định của f ta sẽ chỉ ra được 1 ,f x xf x ¡ . Sau

đây với bài tập 1 chúng tôi nêu lại một số các điều kiện để nghiệm của phương trình hàm Cauchy có dạng 1 ,f x xf x ¡

Bài 1. Giả sử :f ¡ ¡ là một hàm số thỏa mãn điều kiện ; ,f x y f x f y x y ¡ .

Khi đó nếu hàm số f thỏa mãn một trong các điều kiện sau đây: (1) f là hàm đơn điệu trên ¡ (2) f là hàm liên tục trên ¡ (3) f là hàm liên tục tại một điểm trên ¡ (4) f không âm với mọi 0x (5) f bị chặn trong một lân cận của 0 (6) f bị chặn trên một đoạn hữn hạn (7) f có đạo hàm tại một điểm trên ¡ (8) , ,f xy f x f y x y ¡

(9) f bị chặn trên trên một đoạn hữu hạn (10) f bị chặn dưới trên một đoạn hữu hạn

www.VNMATH.com

Page 162: [Vnmath.com] Ky Yeu Trai He Hv 2012

158

(11) f bị chặn trên một đoạn hữu hạn. Thì f x ax , trong đó a là hằng số cho trước tùy ý.

Đây là các kết quả cơ bản về phương trình hàm Cauchy đã có trong nhiều tài liệu nên chúng tôi không nêu lại các chứng minh. Bài 2. Nếu :f ¡ ¡ là một hàm liên tục và với mọi ,x y ¡ ,

(1) .f x y f x f y . Khi đó xf x c

(2) f xy f x f y . Khi đó lnf x c x

(3) .f xy f x f y . Khi đó cf x x

(4) 2 2

f x f yx yf

. Khi đó f x ax b

Lời giải.

(1) Ta có 2

0,2 2 2

x x xf x f f x

¡ . Nếu tồn tại c ¡ sao cho

0 0,f c f x f x c c f x c f c x ¡ . Nếu 0,f x x ¡

thì ta đặt ln , ,g x f x g x y g x g y x y ¡ và g x liên tục

trên ¡ ; do đó theo phương trình Cauchy ta được ,ax xg x ax f x e c x ¡ .

(2), (3), (4) tương tự như phần (1). Bài 3. Tìm tất cả các hàm số : 0; 0;f thỏa mãn điều kiện

2 2f x y f x f y , (1)

với mọi 0, 0x y . Lời giải. Đặt ln , 0g x f x x . Khi đó từ (1) ta được

2 2ln ln ln , , 0f x y f x f y x y

2 2 2 2 , , 0g x y g x g y x y

Do đó theo bài tập 1 ta có , 0g x cx x , trong đó c là hằng số dương suy

ra 2

, 0xf x a x , trong đó a là hằng số dương.

Bài 4. Tìm tất cả các hàm số :f ¡ ¡ , liên tục trên ¡ và thỏa mãn điều kiện

, , .f xy f x y f xy x f y x y ¡ (1)

Lời giải Thay 1y vào (1) ta được:

1 2 1 ,f x f x f x f x ¡ (2)

www.VNMATH.com

Page 163: [Vnmath.com] Ky Yeu Trai He Hv 2012

159

1 2 1f x f x f x f

1 2 1 1 2 2 2f x f x f x f f x f x f x

2 2 2 2 ,f x f x f x f x x ¡ .

Do đó ta thu được:

2 2 ... 2 , 12 2 2 2k k

x x x xf x f x f f f f k

2 lim 2 2 02 2k kk

x xf x f x f f f f

. Từ đó suy ra

2 2 0 ,f x f x f f x ¡ (3)

Với n là số nguyên dương và đẳng thức (3) ta thu được: 2 2 2 2 0f x n f x n f f

2 2 2 4 2 0f x n f x n f f

.............. 2 2 0f x f x f f

Cộng từng vế các đẳng thức trên ta được: 2 2 0 , 1,f x n n f f f x n x ¡ (4)

Tương tự ta có: 2 1 2 0 1 , 1,f x n n f f f x n x ¡ (5)

Thay 2y n vào (1) và kết hợp với đẳng thức (4) ta được: 2 1 2 2 2 2 1 2 2 2f n x f n f nx f x n f n x f nx f x n f n

2 1 2 2 0 1 2 0 1f n x f nx n f f f x n f f f

2 1 2 0f n x f nx f x f (6)

Tương tự ta có đẳng thức: 2 2 1 1 1f nx f n x f x f (7)

Từ các đẳng thức (6) và (7) ta có: 2 2 1 1 1f nx f n x f x f

2 1 2 2 0f n x f n x f x f

.............. 2 1 1f x f x f x f

Cộng từng vế các đẳng thức trên ta được:

www.VNMATH.com

Page 164: [Vnmath.com] Ky Yeu Trai He Hv 2012

160

2 1 1 1 0f nx f x n f x f n f x f

2 1 1 1 0f nx n f x f x f n f . Kết hợp với đẳng thức (2) ta

được: 2 2 1 0 ,f nx nf x n f x ¡ 1 0 ,f nx nf x n f x ¡ (8)

Trong (8) thay 2, 1n x ta được: 2 2 1 0 2 1 0 2 0 0 1 1 2 0 1f f f f f f f f f f f f

Đặt 1 0 ; 0a f f b f . Khi đó với mỗi số nguyên dương n và từ đẳng

thức (8) ta được: 1 1 0f n nf n f an b

1 1 1 1. 1 0f n nf n f f a bn n n n

1 1 1 1. 1 0f n nf n f f a b

n n n n

Với mỗi số hữu tỷ r¤ luôn biểu diễn dưới dạng mr

n , trong đó

*,m n ¥ ¢ nên theo đẳng thức (8) và các đẳng trên ta được:

1 1. 1 0

mf r f m mf n f a b ar b

n n n

f r ar b (9)

Với mỗi x ¡ , tồn tại dãy số hữu tỷ nx hội tụ đến x nên từ đẳng thức (9)

và tính liên tục của f suy ra f x ax b . Thử lại thấy thỏa mãn.

Bài 5. Tìm tất cả các hàm số liên tục :f ¡ ¡ thỏa mãn điều kiện f x y f x f y , với mọi ,x y ¡

(trong đó t là số nguyên lớn nhất không vượt quá t và t t t )

Lời giải. Trước hết ta chứng minh bổ đề sau Bổ đề. Nếu lim n

nu a

, nu là dãy số nguyên thì tồn tại số nguyên dương m

sao cho ,nu a n m . Thật vậy, theo định nghĩa giới hạn thì tồn tại số nguyên dương m sao cho

1 1 1, , ,

2 2 2n n nu a n m a u a n m u a n m

Do đó bổ đề được chứng minh. Trở lại bài toán, từ giả thiết ta có

f x y f x f y , với mọi ,x y ¡

f x y f x y f x f y f x f y , với mọi ,x y ¡

f x y f x f y f x y f x f y , với mọi ,x y ¡

www.VNMATH.com

Page 165: [Vnmath.com] Ky Yeu Trai He Hv 2012

161

Suy ra f x y f x f y ¢ , với mọi ,x y ¡ (1)

và sử dụng tính liên tục của hàm số ta được:

1lim 0 0

nf x f x f f

n

Kết hợp với bổ đề trên tồn tại số nguyên dương k sao cho:

1 10 , ,f x f x f f m k x

m m

¡

11 0 , ,f x f x mf mf m k x

m

¡

10mf mf a m k

m

, trong đó 1 0a f f (2)

Dễ dàng chứng minh được ,m m

f x f x a n kn n

(3)

Với mỗi ,x y ¡ ¡ , tồn tại dãy p

p

m

n

hội tụ đến x với mọi pn k

Khi đó, kết hợp với (3) ta có

lim p p

pp p

m mf x y f x f y f y f f y

n n

lim 0 0p p

pp p

m mf y a f a f y f

n n

.

Từ đó suy ra 0 , ,f x y f x f y f x y ¡

Đặt 0g x f x f ta được g là hàm số liên tục trên ¡ và thỏa mãn

đẳng thức , ,g x y g x g y x y ¡

Do đó theo bài toán 1 ta được , 0 ,g x cx x f x cx f x ¡ ¡

Vậy f x cx b , trong đó ,c b là các hằng số và b¢ .

Bài 6 (Moldova 2009). Tìm tất cả các hàm : 0; 0;f thỏa mãn

điều kiện 2 2f x y z f xz f yz f x y z , (1)

với mọi , , 0;x y z , sao cho x y z .

Lời giải. Thay 0x y z vào (1) ta được 0 0f .

Thay 0x vào (1) ta được 2f y z f yz f y z , với mọi 0y z (2)

www.VNMATH.com

Page 166: [Vnmath.com] Ky Yeu Trai He Hv 2012

162

Đặt g x f x , kết hợp với (2) ta được:

22 22 2g y z g yz f y z f yz f y z g y z ,

với mọi 0y z . Suy ra

22 22g y z g yz g y z ,

với mọi 0y z . (3)

Với mọi , 0a b ta chọn ,2 2

a b a a b ay z

suy ra

22, 2y z a yz b ,

Kết hợp với (3) ta được: , , 0g a b g a g b a b

Do đó theo bài tập 1 ta được , 0g x cx x , trong đó c là hằng số không

âm suy ra 2f x cx . Vậy 2f x cx , trong đó c là hằng số không âm.

Bài 7 (IMO 2002). Tìm tất cả các hàm số :f ¡ ¡ thỏa mãn điều kiện f x f z f y f t f xy zt f xt yz , (1)

với mọi , , , .x y z t ¡ Lời giải. Thay 0x y z vào (1) ta được

2 0 0 2 0 ,f f f t f t ¡ (2)

Trong (2) ta thay 0t ta được

2

0 04 0 2 0 1

02

ff f

f

TH1. Nếu 10

2f thì theo (2) ta được

1,

2f t t ¡

TH2. Nếu 0 0f , thay 0z t ta được

, ,f xy f x f y x y ¡ (3)

Thay 1x y vào (3) ta được

2 1 01 1

1 1

ff f

f

+) Nếu 1 0f thì từ (3) thay 1y ta được 0,f x x ¡

+) Nếu 1 1f thì thay 1y t vào (1) ta được

www.VNMATH.com

Page 167: [Vnmath.com] Ky Yeu Trai He Hv 2012

163

2 , ,f x f z f x z f x z x z ¡ (4)

Thay 0x vào (4) ta được 2 , ,f z f z f z z f z f z z ¡ ¡

Do đó f là hàm chẵn nêu ta chỉ cần xét f trên 0; và kết hợp với (3)

ta được 0, 0f x x (5).

Thay ,t x z y vào (1) ta được

22 2f x y f x f y , với mọi ,x y ¡ (6)

Đặt 22, 0 , 0g x f x x g x g x x (7)

Từ (4) và (6) ta được

2 2 2 2 2 24 2 2f x f z f x z f x z f x z x z f x z

2 2 2 22 2 2 2 , , 0g x g z g x z x z (8)

Thay 0z vào (8) ta được 2 22 2 , 0g x g x x (9)

Từ (7), (8) và (9) ta có: , , 0g x y g x g y x y

Theo bài tập 1 ta có , 0g x cx x , trong đó c là hằng số không âm

2 2f x c x , kết hợp với 1 1 1f c . Do đó 2 , 0f x x x , kết hợp

với f là hàm số chẵn nên 2 ,f x x x ¡ .

Vậy các hàm số thỏa mãn bài toán là:

210, , ,

2f x f x f x x x ¡

Bài tập tương tự Bài 8 (IMO 1992). Tìm tất cả các hàm số f xác định trên tập hợp các số thực và nhận giá trị thực, sao cho với mọi ,x y ¡ ta có

22f x f y y f x

Bài 9. Tìm tất cả các hàm số f xác định trên tập hợp các số thực và nhận giá trị thực, sao cho với mọi ,x y ¡ ta có

nnf x f y y f x ,

trong đó n là số nguyên dương cho trước. Bài 10. Cho 2n là một số tự nhiên. Tìm tất cả các hàm số :f ¡ ¡ thỏa mãn điều kiện

nnf x y x f y , với mọi ,x y ¡

Bài 11 (Romania 1998) Tìm tất cả các hàm số : 0;f ¡ thỏa mãn

www.VNMATH.com

Page 168: [Vnmath.com] Ky Yeu Trai He Hv 2012

164

2 2 2 2 2f x y f x y f xy , với mọi ,x y ¡

Bài 12. Tìm tất cả các hàm số : 0; 0;f thỏa mãn

2 22 2f x y f x f y , với mọi , 0x y

Bài 13. Tìm tất cả các hàm số :f ¡ ¡ thỏa mãn 0 0f và

2 22 2

, , , 0, 02 2

f x f yx yf x y x y

xy f x f y

¡ .

Bài 14. Tìm tất cả các hàm số : 0; 0;f thỏa mãn điều kiện

f xy x y f xy f x y , với mọi , 0;x y

Bài 15. Tìm tất cả các hàm số đơn điệu :f ¡ ¡ thỏa mãn điều kiện , ,f x f y f x y x y ¡

Bài 16. Tìm tất cả các hàm số : 0; 0;f thỏa mãn

, , 0;f x y f x f y f xy f x f y x y

Bài 17. Tìm tất cả các hàm số liên tục : 0;f ¡ sao cho

,f x y f x f y f x f y

với mọi số thực ,x y . Bài 18. Tìm tất cả các hàm số đơn điệu :f ¡ ¡ thỏa mãn điều kiện

0 ,f f x y f x y f

với mọi số thực ,x y . Bài 19. Tìm tất cả các hàm số đơn điệu :f ¡ ¡ thỏa mãn điều kiện

,nf x f y f x y

với mọi số thực ,x y và n là một số nguyên dương. Bài 20. Tìm tất cả các hàm số :f ¡ ¡ sao cho tồn tại hàm số đơn điệu

:h ¡ ¡ thỏa mãn ,h x y h x f y h y

với mọi số thực ,x y . Bài 21. Tìm tất cả các hàm số :f ¡ ¡ thỏa mãn đồng thời các điều kiện sau (1) f liên tục tại 0; (2) f x y f x f y xy x y , với mọi số thực , .x y

Bài 22. Tìm tất cả các hàm số :f ¡ ¡ thỏa mãn đồng thời các điều kiện sau (1) f x y f x f y với mọi số thực ,x y ;

(2) f p x p f x với mọi đa thức p x x ¡ có bậc lớn hơn 1.

www.VNMATH.com

Page 169: [Vnmath.com] Ky Yeu Trai He Hv 2012

165

Bài 23. Tìm tất cả các hàm số :f ¡ ¡ thỏa mãn điều kiện f f x yz x f y f z ,

với mọi , ,x y z ¡ . Bài 24. Tìm tất cả các hàm số :f ¡ ¡ thỏa mãn điều kiện

2 2f f x y x f y ,

với mọi ,x y ¡ . Bài 25. Tìm tất cả các hàm số : 0; 0;f thỏa mãn

2, , 0;f xf x f y f x y x y

Bài 26 (Bulgaria 2004) Tìm tất cả các hàm số :f ¡ ¡ thỏa mãn điều kiện

x yf x f y f f x f y

x y

,

với mọi số thực .x y Bài 27 (India 2003) Tìm tất cả các hàm số :f ¡ ¡ thỏa mãn điều kiện

f x y f x f y f x f y f xy ,

với mọi số thực ,x y . Bài 28. Tìm tất cả các hàm số :f ¡ ¡ thỏa mãn điều kiện

f y zf x f y xf z ,

với mọi , ,x y z ¡ Bài 29. Tìm tất cả các hàm số :f ¡ ¡ thỏa mãn điều kiện

f xf z y zf x y ,

với mọi , ,x y z ¡ . Bài 30. Tìm tất cả các hàm số liên tục : ;f a b ¡ thỏa mãn điều kiện

f xyz f x f y f z ,

với mọi , , , ;xyz x y z a b , trong đó 31 .a b

Bài 31. Tìm tất cả các hàm số :f ¡ ¡ thỏa mãn điều kiện f xy xf y yf x ,

với mọi ,x y ¡ . Bài 32 (Russia 1993). Tìm tất cả các hàm số : 0; 0;f thỏa mãn

, , 0;f yyf x f x x y

Bài 33. Tìm tất cả các hàm số : 0; 0;f bị chặn trên một khoảng

nào đó và thỏa mãn điều kiện f xf y yf x ,

với mọi số thực dương , .x y

www.VNMATH.com

Page 170: [Vnmath.com] Ky Yeu Trai He Hv 2012

166

Bài 34. Tìm tất cả các hàm số : 1; 1;f bị chặn trên một khoảng

nào đó và thỏa mãn điều kiện f x f y xf y y f x yf x ,

với mọi , 1;x y .

Bài 35 (Bulgaria 2003). Tìm tất cả các hàm số :f ¡ ¡ thỏa mãn điều kiện

22 2f x y f y y f x ,

với mọi ,x y ¡ . II. CÁC BÀI TOÁN SỬ DỤNG DÃY LẶP

Cho hàm số :f D D . Khi đó dãy số nu được xác định bởi

...n

n

u f f f x1 4 4 2 4 43

gọi là dãy lặp ứng với hàm số f . Sau đây chúng ta xét một

số bài tập về phương trình hàm sử dụng dãy lặp.

Bài 1. Tìm tất cả các hàm số : 0; 0;f thỏa mãn điều kiện

6 , 0;f f x x f x x

Lời giải. Với mỗi 0x ta đặt ...n

n

u f f f x1 4 4 2 4 43

. Khi đó ta có 2 16 , 0n n nu u u n ,

xét phương trình đặc trưng của dãy: 2 26 0

3

tt t

t

Suy ra 1 22 3nn

nu c c , trong đó 1 2,c c thỏa mãn

1

1 2 0

1 2 1

2

3

52 3 2

5

x f xcc c u x

c c u f x x f xc

Do đó 3 22 3 , 0

5 5

nnn

x f x x f xu n

+) Nếu 2 2 0f x x x f x thì xét với n chẵn ta có lim nn

u

vô lí

+) Nếu 2 2 0f x x x f x thì xét với n lẻ ta có lim nn

u

vô lí.

Suy ra với mỗi 0x thì 2f x x . Vậy 2f x x , 0x .

Bài 2. Tìm tất cả các hàm số : 0; 0;f thỏa mãn điều kiện

www.VNMATH.com

Page 171: [Vnmath.com] Ky Yeu Trai He Hv 2012

167

2 , 0;f f x x x x

Lời giải. Từ tập xác định của , 0f f x x x . Đặt g x f x x , kết hợp với

phương trình hàm đã cho ta được: 3 2g g x f g x g x f f x x f x x x f x x g x

Do đó ta cần tìm hàm số g được xác định như sau: : 0; 0;g thỏa mãn điều kiện 2 , 0g g x g x x x .

Với mỗi 0x ta đặt ...n

n

u f f f x1 4 4 2 4 43

.

Khi đó ta có 2 1 2 0, 0n n nu u u n , xét phương trình đặc trưng của dãy:

2 12 0

2

tt t

t

Suy ra 1 21 2nn

nu c c , trong đó 1 2,c c thỏa mãn

1

1 2 0

1 2 1

2

2

32

3

x f xcc c u x

c c u f x x f xc

Do đó 21 2 , 0

3 3

nnn

x f x x f xu n

+) Nếu 0f x x x f x thì xét với n chẵn ta có lim nn

u

vô lí

+) Nếu 0f x x x f x thì xét với n lẻ ta có lim nn

u

vô lí.

Suy ra với mỗi 0x thì f x x . Vậy f x x , 0x .

Bài 3 (IMO Shortlist 1992). Tìm tất cả các hàm số : 0; 0;f thỏa

mãn điều kiện , 0f f x af x b a b x x ,

trong đó ,a b là các số thực dương cho trước. Lời giải. Với mỗi 0x ta đặt ...n

n

u f f f x1 4 4 2 4 43

.

Khi đó ta có 2 1 0, 0n n nu au b a b u n , xét phương trình đặc trưng của

dãy:

2 0

t bt at b a b

t a b

Suy ra 1 2

nnnu c b c a b , trong đó 1 2,c c thỏa mãn

www.VNMATH.com

Page 172: [Vnmath.com] Ky Yeu Trai He Hv 2012

168

1

1 2 0

1 2 1

2

2

2

a b x f xcc c u x a b

bc a b c u f x bx f xc

a b

Do đó , 03 3

nnn

a b x f x bx f xu b a b n

+) Nếu 0f x bx bx f x thì xét với n chẵn ta có lim nn

u

vô lí

+) Nếu 0f x bx x f x thì xét với n lẻ ta có lim nn

u

vô lí.

Suy ra với mỗi 0x thì f x bx . Vậy f x bx , 0x .

Bài 4. Tìm tất cả các hàm số liên tục :f ¡ ¡ thỏa mãn điều kiện 2 ,f f x f x x x ¡ (1)

Lời giải Nếu f x f y f f x f f y , kết hợp với (1) ta được x y suy ra

f là một hàm đơn ánh. Mặt khác f liên tục suy ra f là một hàm đơn điệu. Thay 0x vào (1) ta được 0 0 0 0f f f f .

Do f là một hàm số đơn điệu nên xảy ra hai khả năng hoặc f đơn điệu tăng hoặc f đơn điệu giảm. TH1. Giả sử f đơn điệu tăng. Do 0 0f suy ra với 0 0x f x và

0 0x f x . Ta có f là hàm số đơn điệu tăng và liên tục suy ra f là

song ánh liên tục nên f có hàm số ngược g . Dễ thấy g thỏa mãn 2 ,x g x g g x x ¡ ; với 0 0x g x và 0 0x g x .

Với mỗi 0x ta đặt ...n

n

u g g g x1 44 2 4 43

, trường hợp 0x làm tương tự.

Khi đó ta có 2 12 0, 0n n nu u u n , xét phương trình đặc trưng của dãy:

2

12 1 0 1

2

tt t

t

Suy ra 1 2

11

2

nn

nu c c

, trong đó 1 2,c c thỏa mãn

1

1 2 0

1 2 1

2

2 2

32 2 2 2

3

x g xcc c u x

c c u f x x g xc

Do đó 2 2 211 , 0

3 2 3

nn

n

x g x x g xu n

www.VNMATH.com

Page 173: [Vnmath.com] Ky Yeu Trai He Hv 2012

169

+) Nếu 2 02

xg x x g x thì xét với n chẵn ta có lim n

nu

vô lí

+) Nếu 2 02

xg x x g x thì xét với n lẻ ta có lim n

nu

vô lí.

Suy ra với mỗi 0x thì 2

xg x . Vậy 2f x x , 0x .

TH2. Giả sử f là một hàm đơn điệu giảm. Từ 0 0f suy ra với

0 0x f x và 0 0x f x .

Với mỗi x ta đặt ...n

n

u f f f x1 4 4 2 4 43

. Khi đó ta có 2 1 2 0, 0n n nu u u n ,

xét phương trình đặc trưng của dãy:

2 12 0

2

tt t

t

Suy ra 1 21 2n n

nu c c , trong đó 1 2,c c thỏa mãn

1

1 2 0

1 2 1

2

2

32

3

x f xcc c u x

c c u f x x f xc

Do đó 21 2 , 0

3 3

n nn

x f x x f xu n

(2)

Từ (2) suy ra 22

22 , 0

3 3n

n

x f x x f xu n

(3)

Nếu 0 0, 0 0 2 2x f x f f x f f x x f f x x

Nếu 0 0, 0 0 2 2x f x f f x f f x x f f x x

Suy ra 2 2u x , kết hợp với (1) và bằng phương pháp quy nạp ta được

2 2 , 0nnu x n (4)

Từ (3) và (4) ta có:

2 22 2 2 2 2 2

22 2 2

2 3.2 2 3.2 2 3.2

nn nn n n n n n

xx f x x f x x f xu ux f x

(5)

Từ (5) và cho n ta được ,f x x x ¡ .

Vậy các hàm số thỏa mãn yêu cầu bài toán là: , 2 ,f x x f x x x ¡

Bài 5 (VMO 2012). Tìm tất cả các hàm số f xác định trên tập số thực ¡ , lấy giá trị trong ¡ và thỏa mãn đồng thời các điều kiện sau: (1) f là toàn ánh từ ¡ đến ¡ ; (2) f là hàm số tăng trên ¡ ;

www.VNMATH.com

Page 174: [Vnmath.com] Ky Yeu Trai He Hv 2012

170

(3) 12f f x f x x với mọi số thực x .

Lời giải Từ điều kiện thứ 3, ta có nếu

12 12f x f y f f x f f y f x x f y y x y

Suy ra f là đơn ánh, kết hợp với (1) ta được f là một song ánh trên ¡ suy ra f có hàm số ngược g . Do f là hàm số tăng trên ¡ nên g cũng là một hàm số tăng trên ¡ . Từ (3), thay 0x ta được 0 0 0 0 0 0f f f f g . Do đó

0 0 0x g x g và 0 0 0x g x g .

Với mỗi 0x ta đặt ...n

n

u g g g x1 44 2 4 43

, trường hợp 0x làm tương tự.

Khi đó ta có 2 112 0, 0n n nu u u n , xét phương trình đặc trưng của dãy:

2

1

412 1 01

3

tt t

t

Suy ra 1 2

1 1

4 3

n n

nu c c

, trong đó 1 2,c c thỏa mãn

1

1 2 0

1 2 1

2

4 12

73 4 12 12 3 12

7

x g xcc c u x

c c u g x x g xc

Do đó 4 12 3 121 1, 0

7 4 7 3

n n

n

x g x x g xu n

+) Nếu 3 12 04

xg x x g x thì xét với n chẵn ta có lim n

nu

vô lí

+) Nếu 3 12 04

xg x x g x thì xét với n lẻ ta có lim n

nu

vô lí.

Suy ra với mỗi 0x thì 4

xg x . Vậy 4f x x , 0x .

Vậy hàm số cần tìm là 4 ,f x x x ¡ .

Nhận xét. Từ điều kiện f là toàn ánh từ ¡ đến ¡ và f là hàm số tăng trên ¡ ta có thể chỉ ra được f là một hàm số liên tục. Do đó bài 5 sẽ được giải tương tự như bài tập 4 ở trên. Sau đây ta đưa ra cách chứng minh nhận xét trên dưới dạng một bổ đề như sau: Bổ đề. Nếu f là toàn ánh từ ¡ đến ¡ và f là hàm số tăng trên ¡ thì

www.VNMATH.com

Page 175: [Vnmath.com] Ky Yeu Trai He Hv 2012

171

f là một hàm số liên tục trên ¡ . Thật vậy, với mỗi dãy nx sao cho lim n

nx a

ta sẽ chứng minh

lim nn

f x f a

. Đặt 1 1sup , ,... , inf , ,...n n n n n nu u u v u u , khi đó ta có

lim , limn nu v lần lượt là giới hạn trên và giới hạn dưới của dãy nu suy ra

lim limn nu v a . Ta dễ thấy nu là một dãy số giảm, còn dãy nv là một dãy số tăng và bất

đẳng thức sau: *,n n nv x u n ¥ (1)

Do f là một hàm số tăng nên ta có *,n n nf v f x f u n ¥ (2)

Do dãy nu là dãy giảm bị chặn và f là hàm số tăng nên dãy số nf u

là dãy số giảm và bị chặn, nên tồn tại lim nf u b . Mặt khác do f là toàn

ánh nên tồn tại z sao cho f z b . Từ đó ta đươc:

lim ; , 1n n nf u f z f u f z u z n (3)

Do dãy nv là dãy tăng, bị chặn và f là hàm số tăng nên dãy số nf v là

dãy số tăng và bị chặn, nên tồn tại lim nf v b . Mặt khác do f là toàn ánh

nên tồn tại t sao cho f t c . Từ đó ta đươc:

lim ; , 1n n nf v f t f v f t v t n (4)

Mặt khác ,n nf v f u n ¥ nên chuyển qua giới hạn ta được

f z f t z t (5)

Từ (3), (4) và (5) ta thu được ,n nv t z u n ¥

Chuyển qua giới hạn ta được z t a suy ra lim limn nf u f v f a (6)

Từ (2), (6) và theo nguyên lí kẹp ta được: lim nf x f a

Do đó f là hàm số liên tục hay bổ đề được chứng minh. Bài 6 (APMO 1989). Tìm tất cả các hàm song ánh, tăng :f ¡ ¡ thỏa mãn điều kiện

1 2 ,f x f x x

với mọi số thực x . Kí hiệu 1f chỉ hàm ngược của hàm số f . Lời giải. Từ giả thiết ta được thay x bởi f x ta được

2 0,f f x f x x x ¡

www.VNMATH.com

Page 176: [Vnmath.com] Ky Yeu Trai He Hv 2012

172

Với mỗi x ¡ ta đặt ... nn

n

u f f f x f x 1 4 4 2 4 43

.

Khi đó ta có 2 12 0, 0n n nu u u n , xét phương trình đặc trưng của dãy: 2 2 1 0 1t t t

Khi đó 1 2nu c n c , trong đó 1 2,c c thỏa mãn

1 2 0 1

1 2 1 2

.0c c u x c f x x

c c u f x c x

Do đó , 0 0 0n n n

nu n f x x x f x n f x f n f x x f x 0

0n nf x f x

f x x fn n

(1)

Do f là hàm số tăng trên ¡ nên 0n nf x f nếu 0x và 0n nf x f nếu 0x (2)

Do đó trong (1) cho n và kết hợp với (2) ta được 0 0f x x f với 0x và 0 0f x x f với 0x (3)

Tiếp theo từ giả thiết thay x bởi 1f x ta được

1 1 12 0,f f x f x x x ¡

Với mỗi x ¡ ta đặt 1 1 1... nn

n

v f f f x f x 1 4 4 4 2 4 4 43

.

Khi đó ta có 2 12 0, 0n n nv v v n , xét phương trình đặc trưng của dãy: 2 2 1 0 1t t t

Khi đó 1 2nv c n c , trong đó 1 2,c c thỏa mãn

1 2 0 1

11 2 1 2

.0

2

c c v x c x f x

c c v f x x f x c x

Do đó , 0 0 0n n n

nv n x f x x f x n f x f n f x x f x 0

0n nf x f x

f x x fn n

(4)

Do f là hàm số tăng trên ¡ suy ra 1f cũng là hàm tăng suy ra 0n nf x f nếu 0x và 0n nf x f nếu 0x (5)

Do đó trong (4) cho n và kết hợp với (5) ta được 0 0f x x f với 0x và 0 0f x x f với 0x (6)

Do đó từ (3) và (6) ta được 0 ,f x x f x ¡ .

www.VNMATH.com

Page 177: [Vnmath.com] Ky Yeu Trai He Hv 2012

173

Do đó hàm số cần tìm là ,f x x a x ¡ , trong đó a là hằng số cho trước.

Bài tập tương tự

Bài 7. Tìm tất cả các hàm số : 0; 0;f thỏa mãn điều kiện

12 ,f f x f x x với mọi 0;x

Bài 8. Tìm tất cả các hàm số liên tục :f ¡ ¡ thỏa mãn điều kiện f f x f x x , với mọi x ¡

Bài 9. Tìm tất cả các hàm song ánh, tăng :f ¡ ¡ thỏa mãn điều kiện 14 4f x f x x ,

với mọi số thực x . Kí hiệu 1f chỉ hàm số ngược của hàm số f . Bài 10. Cho là một số cố định khác 0. Tìm tất cả các hàm số liên tục

:f ¡ ¡ sao cho

2

f xf x x

,

với mọi số thực x . Bài 11. Cho hai hàm số liên tục : ; ; ; : ; ;f a b a b g a b a b thỏa mãn

điều kiện , ;f g x g f x x a b . Biết rằng f là hàm số tăng. Chứng

minh rằng tồn tại ;c a b sao cho f c g c c .

Bài 12. Tìm tất cả các hàm số tăng : 0; 0;f thỏa mãn

2

, 0;x

f x xf x

Bài 13. Cho n là một số nguyên dương cho trước. Tìm tất cả các hàm số liên tục :f ¡ ¡ thỏa mãn điều kiện

...

n

f f f x x 1 4 4 2 4 43

, với mọi x ¡ .

Bài 14. Tìm tất cả các hàm số : 0; 0;f thỏa mãn điều kiện

2 5,f f f x f f x x với mọi 0;x .

Bài 15. Cho ,f g : ¡ ¡ là hai song ánh, tăng thỏa mãn điều kiện

1 1 2 ,f g x g f x x

với mọi x ¡ . Giả sử tồn tại a ¡ sao cho f a g a . Chứng minh rằng

f x g x với mọi số thực x .

www.VNMATH.com

Page 178: [Vnmath.com] Ky Yeu Trai He Hv 2012

www.VNMATH.com

Page 179: [Vnmath.com] Ky Yeu Trai He Hv 2012

TRẠI HÈ HÙNG VƯƠNG LẦN THỨ VII - LÀO CAI

KỲ THI CHỌN HỌC SINH GIỎI TRẠI HÈ HÙNG VƯƠNG LẦN THỨ VII - NĂM 2011

MÔN THI: TOÁN Thời gian: 150’ không kể thời gian giao đề

(Đề thi gồm 05 câu in trong 01 trang)

Câu 1 (4,0 điểm). Giải hệ phương trình

Câu 2 (4,0 điểm). Kí hiệu chỉ tập hợp các số tự nhiên. Tìm tất cả các hàm số

thỏa mãn các điều kiện và

với mọi .

Câu 3 (4,0 điểm). Cho tam giác nhọn có các góc thoả mãn nội tiếp

trong đường tròn , ngoại tiếp đường tròn . là điểm chính giữa cung nhỏ

là trung điểm cạnh . Điểm đối xứng với qua . Đường thẳng cắt

đường tròn tại điểm thứ hai . Chứng minh rằng:

. Điểm thuộc cung nhỏ của đường tròn .

.

Câu 4 (4,0 điểm). Tìm tất cả các số tự nhiên sao cho là số chính

phương.

Câu 5 (4,0 điểm). Một hội nghị Toán học quốc tế có 2011 nhà toán học tham dự.

Biết rằng một nhà toán học bất kỳ trong số đó quen biết ít nhất với nhà toán

học khác. Hỏi có thể lập ra một tiểu ban gồm nhà toán học mà người bất kỳ trong

người đó đều quen biết những người còn lại của tiểu ban đó hay không?

---------------- Hết----------------

Giám thị số 1:………………………… Họ tên thí sinh:……………………. Giám thị số 2:………………………… SBD:………………………

ĐỀ CHÍNH THỨC

www.VNMATH.com

Page 180: [Vnmath.com] Ky Yeu Trai He Hv 2012

www.VNMATH.com

Page 181: [Vnmath.com] Ky Yeu Trai He Hv 2012

www.VNMATH.com

Page 182: [Vnmath.com] Ky Yeu Trai He Hv 2012

Page 1

TRẠI HÈ HÙNG VƯƠNG LẦN THỨ VII - LÀO CAI

KỲ THI CHỌN HỌC SINH GIỎI TRẠI HÈ HÙNG VƯƠNG LẦN THỨ VII - NĂM 2011

HƯỚNG DẪN CHẤM MÔN THI:TOÁN (Hướng dẫn chấm được in trong 05 trang)

Câu 1 (4 điểm).Giải hệ phương trình

Hướng dẫn chấm

Nội dung trả lời Điểm

Điều kiện: hoặc Với ta có

Kết hợp với ta được

Cộng và ta được , thế vào ta có

Từ , nhận xét . Do đó

Đặt (từ đây chú ý là ), ta có

0.25

0.25

0.5

0.5

1

0.5

HDC ĐỀ CHÍNH THỨC

www.VNMATH.com

Page 183: [Vnmath.com] Ky Yeu Trai He Hv 2012

Page 2

Với , ta có

Với , ta có

0.5

0.25

0.25

Câu 2 (4 điểm). Kí hiệu chỉ tập hợp các số tự nhiên. Tìm tất cả các hàm số

thỏa mãn các điều kiện và

với mọi .

Hướng dẫn chấm

Nội dung trả lời Điểm

-Trong : +) cho ta được (do ) +) cho ta được (do ) +) cho ta được Hơn nữa, với thì Nên:

với mọi .

Từ đó cho ta được (do ). Do vậy, với . Ta sẽ chứng minh bằng quy nạp với mọi .

Thật vậy:

-Giả sử đúng đến ta chứng minh cũng đúng đến .

Ta có với mọi

nên

0.25

0.25

0.5

0.5

0.5

www.VNMATH.com

Page 184: [Vnmath.com] Ky Yeu Trai He Hv 2012

Page 3

với mọi .

Do nên

Hay . Do đó,

Theo nguyên lý quy nạp, ta có . với mọi

Thử lại, đúng.

1

0.75 0.25

Câu 3 (4 điểm). Cho tam giác nhọn có các góc thoả mãn nội tiếp trong đường tròn

, ngoại tiếp đường tròn . là điểm chính giữa cung nhỏ là trung điểm cạnh . Điểm

đối xứng với qua . Đường thẳng cắt đường tròn tại điểm thứ hai . Chứng minh rằng:

. Điểm thuộc cung nhỏ của đường tròn .

.

Hướng dẫn chấm (1 điểm).

Nội dung trả lời Điểm

Ta có Do đó E nằm trong góc

Do nên và nằm về cùng một phía đối với . Do đó và nằm về cùng một phía đối với Từ và suy ra nằm trong góc . Vậy thuộc cung nhỏ .

0.25

0.5

0.25

(3 điểm).

Nội dung trả lời Điểm

Gọi kà điểm trên đoạn sao cho .

0.25

www.VNMATH.com

Page 185: [Vnmath.com] Ky Yeu Trai He Hv 2012

Page 4

Khi đó ta có cân tại và

. Do đó

Ta lại có

Từ và suy ra .

Do đó, bốn điểm đồng viên.

-Mặt khác (do đồng viên)

Hay:

Gọi cắt tại khi đó là hình bình hành. Do đó là trung điểm Vậy là hình bình hành Mặt khác ta có và suy ra Từ và suy ra Từ và ta có . Nhận xét: Nếu học sinh làm theo phương pháp không dùng góc định hướng thì phải xét hai trường hợp như 2 hình vẽ trong đáp án. Nếu chỉ làm đúng được một trường hợp, cho điểm.

0.25

0.25

0.25

0.25

0.25

0.5

0.25

0.25

0.25

0.25

Câu 4 (4 điểm). Tìm tất cả các số tự nhiên sao cho là số chính phương.

Hướng dẫn chấm

www.VNMATH.com

Page 186: [Vnmath.com] Ky Yeu Trai He Hv 2012

Page 5

Nội dung trả lời Điểm

Dễ thấy thì không là số chính phương. Xét Nếu lẻ , đặt . Ta có , suy ra không là số chính phương Nếu chẵn , đặt . Giả sử tồn tại số nguyên sao cho :

thì , đặt . Khi đó

Vậy khi thì là số chính phương

0.25

1

0.25

0.5

1

1

Câu 5 (4 điểm). Một hội nghị Toán học quốc tế có 2011 nhà toán học tham dự. Biết rằng một nhà toán

học bất kỳ trong số đó quen biết ít nhất với nhà toán học khác. Hỏi có thể lập ra một tiểu ban

gồm nhà toán học mà người bất kỳ trong người đó đều quen biết những người còn lại của tiểu ban

đó hay không?

Hướng dẫn chấm

Nội dung trả lời Điểm

Gọi là tập hợp các nhà Toán học trong hội nghị, thì . Giả sử , ta quy

ước nếu quen biết nhau thì viết . Xét mà . Đặt

Thì .

Suy ra .

Do đó tồn tại , tức là tồn tại nhà toán học đôi một quen biết nhau.

Xét thì và . Suy ra

.

Do đó tồn tại , chứng tỏ có nhà toán học đôi một quen biết nhau.

Tiếp tục, xét thì và

. Suy ra

. Do đó

tồn tại , chứng tỏ có nhà toán học đôi một quen biết nhau.

Vậy có thể lập ra một tiểu ban gồm nhà toán học mà bất kỳ trong người đó đều quen

0.5 0.5

0.5 0.5 0.5 0.5 0.5

Ghi chú: Thí sinh làm cách khác đáp án vần cho điểm tối đa theo từng phần. ------------Hết------------

www.VNMATH.com

Page 187: [Vnmath.com] Ky Yeu Trai He Hv 2012

Page 6

biết những người còn lại của tiểu ban đó. 0.5

www.VNMATH.com

Page 188: [Vnmath.com] Ky Yeu Trai He Hv 2012

www.VNMATH.com

Page 189: [Vnmath.com] Ky Yeu Trai He Hv 2012

TRẠI HÈ HÙNG VƯƠNG LẦN THỨ VII - TẠI LÀO CAI

KỲ THI CHỌN HỌC SINH GIỎI TRẠI HÈ HÙNG VƯƠNG LẦN THỨ VII - NĂM 2011

HƯỚNG DẪN CHẤM MÔN THI:TOÁN (Hướng dẫn chấm được in trong 04 trang)

Câu 1 (4 điểm). Giải hệ phương trình : 3 3 2

2 3

3 4 2

5

x y y x y

x y x y

Hướng dẫn chấm

Nội dung trả lời Điểm

Ta có : 3 3 2 3 3

2 3 2 3

3 4 2 ( 1) ( 1)

5 5

x y y x y x x y y

x y x y x y x y

Hàm số f(x) = x3 + x là hàm số đồng biến trên R nên phương trình f(x) = f(y - 1) x = y - 1

Do đó 3 3

2 3 2 32 3

1 1( 1) ( 1)

5 ( 1) 4 55

x y x yx x y y

x y x y y y yx y x y

Ta có 2 3

3 2

52

( 1) 4 5 412

15 38 24 0

y yy y y

yy y y

Vậy hệ có 2 nghiệm : 1 11

;2 12

x x

y y

1

1

1

1

Câu 2 (4 điểm). Có tồn tại hay không một hàm số * *:f N N sao cho:

1 1f f n f n f n với mọi 2n .

Hướng dẫn chấm

Nội dung trả lời Điểm

Ta chứng minh bằng phản chứng là không tồn tại hàm số f thỏa mãn yêu cầu.

Vì * *:f N N nên 1f n , với mọi *n N .

Từ giả thiết có 1 1 1, 2f n f n f f n n , suy ra f là hàm đồng biến

0.5

0.5

HDC ĐỀ DỰ BỊ

www.VNMATH.com

Page 190: [Vnmath.com] Ky Yeu Trai He Hv 2012

trên tập 2n , do đó

1 1 ... 2 2 1 2, 2f n f n f n n n

1, 2f n n n . (1)

Từ giả thiết ta lại có 1 1 , 2f f n f n n hay

2 , 1f f n f n n , do f là hàm đồng biến trên tập 2n nên có

2, 2f n n n hay 1, 2f n n n (2)

Từ (1) và (2) suy ra

1 2 1 3, 2f n f n n n n (3)

Mặt khác từ (1) ta có:

1 1 1 2 1 3 3, 7f f n f n n n n (4)

Từ (3) và (4) suy ra điều mâu thuẫn. Vậy không tồn tại hàm số nào thỏa mãn yêu cầu.

0.5

1

0.5

0.5

0.5

Câu 3 (4 điểm). Cho tam giác nhọn ABC, trên cạnh BC lấy các điểm E, F sao cho góc · ·BAE CAF ,

gọi M, N lần lượt là hình chiếu vuông góc của F trên các đường thẳng AB và AC, kéo dài AE cắt

đường tròn ngoại tiếp tam giác ABC tại D. Chứng minh rằng tứ giác AMDN và tam giác ABC có diện

tích bằng nhau.

Hướng dẫn chấm

Nội dung trả lời Điểm

Đặt · · ·, .BAE CAF EAF

N M

D

O

A

B C E F

www.VNMATH.com

Page 191: [Vnmath.com] Ky Yeu Trai He Hv 2012

Ta có 1 1. .sin . .sin

2 2ABCS AB AF AC AF = . .4

AFAB CD AC BD

R

(R-là bán kính đường tròn ngoại tiếp tam giác ABC) (1) Diện tích tứ giác ADMN là

1 1. .sin . .sin( )

2 2AMDNS AM AD AD AN =

= 1. .cos .sin .cos .sin

2AD AF AF

1. .sin 2 . .

2 4

AFAD AF AD BC

R . (2)

Vì tứ giác AMDN nội tiếp trong đường tròn nên theo định lí Ptoleme ta có AB.CD +

AC.BD = AD.BC (3).

Từ (1), (2), (3) ta có điều phải chứng minh

1

1

1

1

Câu 4 (4 điểm). Chứng minh rằng phương trình sau đây không có nghiệm nguyên dương

Hướng dẫn chấm

Nội dung trả lời Điểm

32 21 1 2 1x x y . Lập luận x chẵn, do đó 2 21; 1 1x x

khi đó 2 3

2 3

1

1

x a

x b

trong đó a, b là hai số nguyên dương nguyên tố cùng nhau thoả mãn

2 1ab y .

2 2 3 3 2 21 1 2x x b a b a a ab b

khi đó 2 2a ab b là ước nguyên dương của 2 và phải lớn hơn hoặc bằng 3, vô lý; phương trình vô nghiệm.

1

1

1

1

Câu 5 (4 điểm). Trên đường tròn (C) cho 2011 điểm. Hỏi có bao nhiêu cách xóa đi 11 điểm sao cho

không có hai điểm bị xóa nào cạnh nhau.

Hướng dẫn chấm

Nội dung trả lời Điểm

Ta gọi một trong số 2011 điểm là A. Có hai trường hợp sau: Trường hợp 1. Điểm A không bị xóa. Sau khi xóa đi 11 điểm còn lại 2000 điểm. Xen kẽ xen kẽ giữa 2000 điểm này có 2000

www.VNMATH.com

Page 192: [Vnmath.com] Ky Yeu Trai He Hv 2012

khoảng trống. Mười một điểm bị xóa tương ứng với 11 trong số 2000 khoảng trồng nói trên.

Do đó số cách thực hiện trong trường hợp này bằng 11

2000

2000!

11!.1989!C

Trường hợp 2. Điểm A bị xóa. Sau khi xóa tiếp 10 điểm, còn lại 2000 điểm. Xen kẽ xen kẽ giữa 2000 điểm này có 1999 khoảng trống không kề với vị trí của điểm A. 10 điểm bị xóa (không kể điểm A) tương ứng với 10 trong số 1999 khoảng trống nói trên.

Do đó số cách thực hiện trong trường hợp này bằng 10

1999

1999!

10!.1989!C

Theo quy tắc cộng số cách xóa cần tìm thỏa mãn đề bài là: 11

2000C + 10

1999C = 2000!

11!.1989!+

1999!

10!.1989!

1,5 1,5 0,5

Ghi chú: Thí sinh làm cách khác đáp án vần cho điểm tối đa theo từng phần.

------------Hết------------

www.VNMATH.com

Page 193: [Vnmath.com] Ky Yeu Trai He Hv 2012

www.VNMATH.com

Page 194: [Vnmath.com] Ky Yeu Trai He Hv 2012

TRẠI HÈ HÙNG VƯƠNG LẦN THỨ VII - TẠI LÀO CAI

KỲ THI CHỌN HỌC SINH GIỎI TRẠI HÈ HÙNG VƯƠNG LẦN THỨ VII - NĂM 2011

MÔN THI: TOÁN Thời gian: 150’ không kể thời gian giao đề

(Đề thi gồm 05 câu in trong 01 trang)

Câu 1 (4,0 điểm). Giải hệ phương trình :

3 3 2

2 3

3 4 2

5

x y y x y

x y x y

Câu 2 (4,0 điểm). Có tồn tại hay không một hàm số * *:f N N sao cho:

1 1f f n f n f n với mọi 2n .

Câu 3 (4,0 điểm). Cho tam giác nhọn ABC, trên cạnh BC lấy các điểm E, F sao cho

góc · ·BAE CAF , gọi M, N lần lượt là hình chiếu vuông góc của F trên các đường

thẳng AB và AC, kéo dài AE cắt đường tròn ngoại tiếp tam giác ABC tại D. Chứng

minh rằng tứ giác AMDN và tam giác ABC có diện tích bằng nhau.

Câu 4 (4,0 điểm). Chứng minh rằng phương trình sau đây không có nghiệm nguyên

dương

Câu 5 (4,0 điểm). Trên đường tròn (C) cho 2011 điểm. Hỏi có bao nhiêu cách xóa đi

11 điểm sao cho không có hai điểm bị xóa nào cạnh nhau.

---------------- Hết----------------

Giám thị số 1:………………………… Họ tên thí sinh:…………………….

Giám thị số 2:………………………… SBD:………………………

ĐỀ DỰ BỊ

www.VNMATH.com

Page 195: [Vnmath.com] Ky Yeu Trai He Hv 2012

www.VNMATH.com